Сохранен 660
https://2ch.hk/spc/res/182891.html
24 декабря Архивач восстановлен после серьёзной аварии. К сожалению, значительная часть сохранённых изображений и видео была потеряна. Подробности случившегося. Мы призываем всех неравнодушных помочь нам с восстановлением утраченного контента!

Тред гениальных вопросов и каких получится ответов. #26

 Аноним 28/05/15 Чтв 17:33:56 #1 №182891 
14328236362510.jpg
Тред вопросов о жизни, Вселенной и всем таком.

Спрашиваем то, за что в других местах выдают путёвку в биореактор. Здесь анонимные ученые мирового уровня критически рассмотрят любые гениальные идеи и нарисованные в Paint схемы.

Прошлый тред тут: https://2ch.hk/spc/res/176350.html
Аноним 28/05/15 Чтв 17:40:47 #2 №182894 
С чего решили, что Хаумеа овальная?
Аноним 28/05/15 Чтв 17:48:56 #3 №182898 
>>182894
По кривой блеска. Очень сильные колебания получаются.
Аноним 28/05/15 Чтв 18:06:38 #4 №182906 
Есть ли цветные фоточки поверхности венеры? Не те 1.5, что в википедии. Неужто никто кроме дидов поверхность не фоткал?
Аноним 28/05/15 Чтв 18:25:07 #5 №182909 
>>182906
Поверхность скрыта под плотным слоем облаков, так что нихуя нет. Только радиолокация.
>Неужто никто кроме дидов поверхность не фоткал?
Представьте себе.
Аноним 28/05/15 Чтв 18:44:55 #6 №182912 
http://tass.ru/kosmos/1999276
Не стал делать новый тред, реквестирую видео
Аноним 28/05/15 Чтв 18:59:03 #7 №182916 
>>182912
https://www.youtube.com/watch?&v=J1QuGKpFnlg
Аноним 28/05/15 Чтв 19:00:10 #8 №182917 
>>182909
Почему тогда больше посадочные модули не отпарвляют? Соседка же земли
Аноним 28/05/15 Чтв 19:00:36 #9 №182918 
Сколько надо бабла чтобы слетать на марс с возвратом в одно рыло?
Аноним 28/05/15 Чтв 19:02:23 #10 №182920 
>>182917
Потому что ещё не изобрели мифрил и адамантий.
невозможно сделать годную систему охлаждения для тех условий
Нет там нифига интересного
Аноним 28/05/15 Чтв 19:05:27 #11 №182922 
14328291277020.jpg
>>182912
Анимация того, как это происходило:
http://www.youtube.com/watch?v=so_e0RZqnCE
Запись трансляции:
http://www.youtube.com/watch?v=D_GJ_OmLiZM
Таймлапс:
https://twitter.com/AstroTerry/status/603618564491214848
Пикрилейтед - фото Саманты Кристофоретти в процессе перемещения.


Алсо, традиционно охуительная статья с анализом ситуации и вариантов на nasaspaceflight:
http://www.nasaspaceflight.com/2015/05/iss-program-station-reconfiguration-future-crew-vehicles/
Аноним 28/05/15 Чтв 19:10:25 #12 №182923 
14328294254990.jpg
>>182918
> Сколько надо бабла чтобы слетать на марс с возвратом в одно рыло?
Обязательно/ни в коем случае.
Аноним 28/05/15 Чтв 19:13:59 #13 №182925 
>>182918
> Сколько надо бабла чтобы слетать на марс с возвратом в одно рыло?
Вопрос некорректный. Организация первичнее ресурсов.
Сколько нужно бабла, чтобы ублюдки не кидались повсюду бычками? Ответ: нисколько. Просто не кидай.
Аноним 28/05/15 Чтв 19:17:46 #14 №182926 
>>182925
Ну если например Билл Гейтс (76 миллиардов $) захочет слетать на марс - потянет? Ну кредит ещё возьмёт.
Аноним 28/05/15 Чтв 19:20:16 #15 №182927 
>>182918
Мы тут с диваном посовещались, и пришли к выводу, что все разработки, тесты, производство и сопутствующие расходы обойдутся лярдов в 200-300.
Аноним 28/05/15 Чтв 19:32:57 #16 №182929 
Собираются ли к нящам типа Седны и Эриды отправлять зонды?
Аноним 28/05/15 Чтв 19:45:50 #17 №182933 
>>182929
Нет, даже в планах пока ничего такого нет.
Аноним 28/05/15 Чтв 19:51:06 #18 №182934 
>>182933
Жаль. На потешную Цереру же выделили средства
Аноним 28/05/15 Чтв 19:55:03 #19 №182936 
>>182934
Церера сильно ближе.
Аноним 28/05/15 Чтв 20:03:13 #20 №182939 
Почему мурриканские спутники живут очень долго (особенно АМС), а сделанные в роисси-матушке - от силы пару лет? Это уже традиция со времён Никиты Кукурузки как минимум.
От чего зависит время жизни спутника? Развёрнуто если можно, ибо коротко и сам знаю.
Аноним 28/05/15 Чтв 20:08:05 #21 №182940 
>>182939
Спутники и АМС - разные вещи.
Спутник - просто банка с антенной и солнечными панелями, дешевая, можно быстро заменить, благо болтается под боком.
АМС - целая лаборатория, заточенная под полет, длящийся десятилетиями, заменить ее долго - те самые десятки лет.
Аноним 28/05/15 Чтв 20:12:29 #22 №182943 
>>182940
Какое отношение это имеет к вопросу? Факт в том, что и коммерческие спутники, и тысячу раз перепроверенные АМС, живут гораздо дольше если сконструированы не в России.
Аноним 28/05/15 Чтв 20:25:58 #23 №182948 
>>182943
Причём это такая давняя традиция, что ТАЙНА ЛАЙФТАЙМА занимает всё моё сознание. Когда был совок можно было ещё спихнуть на недоступность нормальной элементной базы в стране с загнивающей электронной промышленностью (хотя мне кажется, что где нужно таки юзали добытое через серые каналы, как в других отраслях), но сейчас-то всё можно купить (и милитари класс, и космический), и таки покупают. Но спутники по-прежнему живут охуеть как мало.
Аноним 28/05/15 Чтв 20:29:10 #24 №182949 
>>182940
Лiл. Спутник - это аппарат, находящийся на орбите планеты. АМС может быть спутником, если она не посадочная и не пролетная. Спутник может быть намного сложнее простой посадочной АМС,
Аноним 28/05/15 Чтв 20:29:32 #25 №182950 
>>182891
Вопрос века.
ЧТО ЖЕ БЫЛО ДО БОЛЬШОГО ВЗРЫВА??????????????????????
И ЧТО БУДЕТ ПОСЛЕ СХЛОПЫВАНИЯ???????????????
Аноним 28/05/15 Чтв 20:40:16 #26 №182953 
>>182949
>Какое отношение это имеет к вопросу?
Ты дебил?
Аноним 28/05/15 Чтв 20:40:53 #27 №182954 
>>182953
Ойвей, сорян чувак, попутал рамсы в натуре кароч.
Аноним 28/05/15 Чтв 20:42:25 #28 №182955 
>>182950
>ЧТО ЖЕ БЫЛО ДО БОЛЬШОГО ВЗРЫВА
Хватит толстить.
>>182948
>на недоступность нормальной элементной базы
Сейчас эта проблема тоже существует. В последние годы, сроки активного существования удалось увеличить, но до европейских и американских еще далеко. Короче, мы работаем над этим.
Аноним 28/05/15 Чтв 20:43:52 #29 №182957 
>>182940
Шоб на тебя свалился дорогущий геостационарный спутник для трансляции порнухи, с более чем 9000 транспондеров и системами развёртывания антенн из сухожилий енотов-альбиносов.
Аноним 28/05/15 Чтв 20:43:58 #30 №182958 
14328350388400.jpg
>>182891
Человечество ведь никогда не выберется из этой сраной системы верно?
Если до ближайшей звезды 4 световых года а это расстояние которое человеческий мозг неспособен осознать то мы даже до неё никогда не доберёмся.
Я прав или я прав?
Аноним 28/05/15 Чтв 20:45:06 #31 №182960 
>>182955
Т.е. все проблемы от элементной базы онли (и отсутствия людей, которые могут грамотно её юзать)?
Аноним 28/05/15 Чтв 20:46:52 #32 №182961 
Что будет если добраться до края вселенной? Скорость света похуй, есть движок Альбукеркекубьерре
Аноним 28/05/15 Чтв 20:48:08 #33 №182962 
>>182961
Ты сначала до транснептуновых объектов добери. Кто знает, что там. Может там 3 черепахи, а ты про край вселенной
Аноним 28/05/15 Чтв 20:49:24 #34 №182964 
>>182948
Байка с факультета радиофизики БГУ.
Пилят антенну для некой венерианской АМС (подробностей нет, к сожалению, и уже не будет). Ок.
Поднимаем на вертолёте повыше, делаем замеры, опускаем, с помощью высокотехнологичного молотка изменяем форму, поднимаем повыше...
Аноним 28/05/15 Чтв 20:50:51 #35 №182965 
>>182960
Там комплексная проблема. От микроэлектроники, до энергообеспечения. Как показывает пример Решетнева, с людьми все может быть вполне окей.
Аноним 28/05/15 Чтв 20:54:50 #36 №182967 
>>182958
>Человечество ведь никогда
Может когда-нибудь, но тебя это волновать уже не будет.
>4 световых года а это расстояние которое человеческий мозг неспособен осознать
Способен, если его развивать в эту сторону. И осознание расстояния вовсе необязательно, для его покорения. До Луны тоже не близко, но летали же.
Аноним 28/05/15 Чтв 20:59:23 #37 №182968 
>>182964
Как байка - может быть, но у нас ИСС давно уже умеет делать рефлекторы антенн с контурными диаграммами направленности.
Аноним 28/05/15 Чтв 21:00:28 #38 №182969 
>>182965
>энергообеспечения
У муриканцев хигх-текх акумуляторы штоле?
С электроникой понятно: если не поставить правильный чип или хуёво спроектировать - посыпется.
>>182968
Может и байка, а может хуй его знает что они там меряли и чего хотели.
Аноним 28/05/15 Чтв 21:05:48 #39 №182972 
>>182969
>У муриканцев хигх-текх акумуляторы штоле?
И аккумуляторы и панели лучше, да. По крайней мере срок деградации у них больше.
>>182961
>Что будет если добраться до края Земли? Скорость лошади похуй, есть парусник Виктория.
Аноним 28/05/15 Чтв 21:08:45 #40 №182974 
Правда ли что обьект вьебавшийся в планету со скоростью близкой к световой пройдёт сквозь неё без каких-либо последствий для себя и планеты?
Аноним 28/05/15 Чтв 21:09:06 #41 №182975 
>>182972
Я не думаю, что туда ставят что-то, чего нельзя купить. И также не думаю, что купить это будет дороже чем запустить скажем 2 спутника вместо одного, пока отечественный производитель пытается что-то годное, простите, высрать.
Аноним 28/05/15 Чтв 21:10:02 #42 №182977 
>>182958
Теоретических ограничений для полетов к другим звездам нет. В далеком-предалеком будущем мы сможем летать к соседним звездам со скоростью, близкой к световой. Впрочем, и в Солнечной системе более чем достаточно интересных мест, здесь то еще веселье можно устроить.
Аноним 28/05/15 Чтв 21:12:52 #43 №182980 
>>182975
Это проблема технологий разработки и производства. ИСС делает успешно, потому что умеет. Остальные разучились делать.
Аноним 28/05/15 Чтв 21:14:00 #44 №182981 
>>182977
Нихуя туи интересного нету, говно одно. Еще и проезд подорожал.
Аноним 28/05/15 Чтв 21:16:01 #45 №182983 
>>182975
Почитай, про микросхемы, для космоса, их не так просто купить. К тому же не все можно купить, потому что мы долго шли своим путем и некоторые вещи просто могут быть несовместимыми.
>>182980
ИСС тоже пока не вышли на уровень, но они стараются и результаты у них уже есть.
>Остальные разучились делать.
Не разучились, просто в какой-то момент развитие остановилось, не до жиру было, и все ушли вперед, а у нас все осталось в жопе.
>>182974
Нет, не правда.


Аноним 28/05/15 Чтв 21:16:09 #46 №182984 
>>182975
Ну купи сейчас американский или хранцузский спейс-грейд, находясь в россии, попробуй. Да хотя бы индастриал-грейд. Алсо, даже если забыть про экспортные ограничения - например практичная наса не гнушается международными подрядчиками, а мы жи гордые, нам западло.
Аноним 28/05/15 Чтв 21:17:26 #47 №182985 
>>182981
Проезд только дешевеет. А интересного много. Если ты хотел Земли-2, то ее нет. Но при наличии дешевого вывода в космос можно наклепать кучу кораблей (и устроить космическую войнушку), баз на всяких астероидах, колоний на других планетах и т. д. Создание самоподдерживающейся автономной базы - не такая сложная задача, какой она кажется на первый взгляд. На нынешнем уровне развития технологий это вполне доступно.
Аноним 28/05/15 Чтв 21:19:43 #48 №182986 
>>182983
>ИСС тоже пока не вышли на уровень, но они стараются и результаты у них уже есть.
Суть в том, что их спутники работают. Пусть и с 70% импорта.
>Не разучились, просто
Нет, именно разучились. Не могут сделать систему, которая будет нормально работать заявленный срок. Могут сделать кое-как то, что не пролетает и года - как любой, кто пытается в первый раз.
Аноним 28/05/15 Чтв 21:21:26 #49 №182988 
>>182986
Дешевле запустить много дешёвых спутников, это всем известно.
Аноним 28/05/15 Чтв 21:33:40 #50 №182995 
>>182985
Откуда будешь брать воздух, мань?
Аноним 28/05/15 Чтв 21:37:44 #51 №182996 
>>182995
есть два варианта
1. Китайцы устроили успешный эксперимент с замкнутой биосферой год назад. Воздух генерировался водорослями, вроде, или другой растительностью. Вся необходимая еда тоже выращивалась.
2. В Солнечной системе воды дофига, она почти везде есть. Вода легко превращается в кислород.
Аноним 28/05/15 Чтв 21:37:49 #52 №182997 
>>182995
Воды в космосе - залейся, выделяй кислород, регенерируй.
Аноним 28/05/15 Чтв 21:38:51 #53 №182998 
>>182996
>Китайцы устроили успешный эксперимент с замкнутой биосферой год назад.
Дай почитать.
Аноним 28/05/15 Чтв 21:39:11 #54 №182999 
>>182996
>>182997
Как вы будете ловить эту воду, находясь на астероиде?
Аноним 28/05/15 Чтв 21:44:01 #55 №183000 
>>182999
Добывать из астероида, лалка
Аноним 28/05/15 Чтв 21:44:16 #56 №183001 
>>182999
>Подразумевает, что на астероидах нет воды.
Аноним 28/05/15 Чтв 21:47:39 #57 №183003 
>>183000
>>183001
>Подразумевают, что вода на астероиде не кончится через год-два
Аноним 28/05/15 Чтв 21:49:48 #58 №183005 
>>183003
Таки не кончится
Аноним 28/05/15 Чтв 21:50:13 #59 №183006 
14328390140010.jpg
>>182961
Авотхуйзнает. Послали один кораблик, так и летит до сих пор.
Аноним 28/05/15 Чтв 21:50:42 #60 №183007 
>>183003
>Подразумевает, что за 2 года из астероида всю воду выпьют жиды и выссут в космос.
Аноним 28/05/15 Чтв 22:02:18 #61 №183008 
>>182998
щас найду, не расходись
Аноним 28/05/15 Чтв 22:03:45 #62 №183009 
>>183008
Ок. Жду.
Аноним 28/05/15 Чтв 22:05:22 #63 №183010 
>>182998
Была статья в "Новостях космонавтики". Не нажел, но она 100% висела в открытом доступе в виде пдфки. А вот в английской вики: http://en.wikipedia.org/wiki/Yuegong-1
Аноним 28/05/15 Чтв 22:06:07 #64 №183012 
>>182999
На астероиде - генерировать искусственной биосферой. В тех местах, где есть доступная вода - использовать ее.
Аноним 28/05/15 Чтв 22:11:46 #65 №183013 
>>183010
105 дней - маловато, на мой взгляд. И жрать мучных червей - ну пиздец, пиздоглазым-то похуй, но европеоиды блеванут от одной мысли.
Аноним 28/05/15 Чтв 22:13:36 #66 №183014 
>>183013
105 дней, 210 или 1050 - без разницы, если им сгенерированного кислорода и еды хватало.
Аноним 29/05/15 Птн 01:42:35 #67 №183044 
Зачем вселенная такая большая?
Аноним 29/05/15 Птн 01:43:37 #68 №183045 
>>183044
Спроси у своей мамки.
Аноним 29/05/15 Птн 01:45:10 #69 №183047 
>>183044
Это просто ты маленький, а Вселенная в самый раз.
Аноним 29/05/15 Птн 02:52:48 #70 №183073 
>>183044
Чтоб было интересно жить.
Аноним 29/05/15 Птн 06:21:09 #71 №183085 
>>183073
Интересно? У меня депрессия из-за этой хуйни всё жизнь.Я умру так толком и не узна ничего о ней.Если тебе это "интересно" то ты довольно странный
>>183044
Да это просто такое издевательство.Тонкий намёк на ничтожность человека
Аноним 29/05/15 Птн 12:00:56 #72 №183102 
>>182920
>невозможно сделать годную систему охлаждения для тех условий
воувоу ты полегче с такими словами - невозможно, 400 по цельсию(или скока там) это не мало конечно, но работать можно - система охлаждения для этих условий вполне реальна. пачка других обстоятелсьт обоссывает эту идею в текущих условиях. но я заметил слово годную - такчто это больше в контексте нациграмматрон
Аноним 29/05/15 Птн 12:03:00 #73 №183103 
>>182918
подвезу за лям баксов
Аноним 29/05/15 Птн 12:11:49 #74 №183105 
>>182926
теоретически пейсх вроде стартовал с меньшего, марс объявлен некой целью - такчто выходит у билли был бы шанс, но он старикан уже, может не докоптить до результатов
Аноним 29/05/15 Птн 12:43:09 #75 №183112 
>>182984
>мы жи гордые, нам западло.
как будто это плохо, сейчас все играют в сосницкого в космосе, пока. и участники бегут пока не особенно напрягаясь, отдыхая от предыдущих спринтов, и в принципе это неплохо. для нас главное чтобы у нас к моменту Х было все как надо. следи за индусами китаезами - они еще дадут просраться и пиндосам и нам, европка не в счет но хз, а япошки мутные хз, но чето нет ощущений что они сделают домашнюю работу.

>>182986
>Суть в том, что их спутники работают. Пусть и с 70% импорта.
линку ссылку что еще обосновать могешь ? мне просто любопытно, где они эти 70% гребут, в каких таких заморских странах.

>>182988
было бы неплохо, http://tass.ru/kosmos/2000889 боюсь что попиздят и воз кинут, так то вообщето забавная тема, не только из-за самих спутников, сколько из-средне серийного производства космического стаффа

>>182995 >>182999 >>182999
в чем проблема с воздухом - куда не плюнь везде окислы то железа то кремния, подход к энергетическому вопросу вот это проблема.

>>183013
))) охохо о такой форе у азитов я даже не думал, пасибо ананас

>>183014
>Восемь человек (четыре женщины и четверо мужчин) пробыли в «Биосфере-2» около двух лет, поддерживая связь с внешним миром только через компьютер.
>Через пару недель одна из участниц эксперимента отрезала себе палец при работе на сельскохозяйственном оборудовании.
>Довольно быстро команда разбилась на две противоборствующие группы. Это сильно мешало нормальному ходу исследований. Даже спустя 20 лет группы избегают встреч
ржакак как по моему ))
Аноним 29/05/15 Птн 12:47:44 #76 №183113 
>>183085
ты уже купил себе телескоп, да ? обчитался астрофизическими теориями? обмазался матаном и симуляциями?
уверяю, все это поможет тебе, если ты этого еще не сделал.
Аноним 29/05/15 Птн 14:17:18 #77 №183129 
Мне кажется, пришло вскрыть эту тему, которая хотя и немного нерелейтед, но напрямую связана с космосом, а именно с ролью космоса в вашей социальной жизни.
.
1)Итак, диванные астрономы, какое влияние ваше увлечение астрономией оказывает на социальную жизнь? Вы сычуете в одиночку за телескопами, или собираетесь с кем-то, имеете свой круг таких же мамкиных астрономов?

2)Самый важный вопрос. Как относятся к увлечению космосом тян? Как относятся к этому ваши тян которых нет?Есть ли тян-астролюбы, как тян от реагирует, если я её приглашу сходить на задний двор и посмотреть в мой телескоп на звёзды - посчитает меня мудаком-задротом или же ей понравится? Будем считать, что тян - не сферическая ТП, а хоть мало-мальски нормальная.

Добра всем
Аноним 29/05/15 Птн 14:21:35 #78 №183131 
>>183129
>какое влияние ваше увлечение астрономией оказывает на социальную жизнь?
Никакого
>Как относятся к увлечению космосом тян?
Да похуй, я не спрашивал.
>Как относятся к этому ваши тян
Положительно: спит в машине, время от времени просыпается и выбирается на что-нибудь позырить.
Аноним 29/05/15 Птн 14:58:34 #79 №183141 
да они все радосно бегут с тобой на задний двор
подержать твоу упругий телескоп!

они так еше характерно становяцца к тебе задом и немного наклоняюцца
Аноним 29/05/15 Птн 15:37:06 #80 №183153 
>>183085
>Я умру так толком и не узна ничего о ней
Ты просто слишком наивен. Смерть это конец одного приключения и начало следующего.
Аноним 29/05/15 Птн 15:43:15 #81 №183155 
>>183129
Ты что с тян никогда не разговаривал? Тян это такой же человек как ты, но только отличается немного физиологически. У тян такие же интересы как и у тебя. Если тян будет не интересно смотреть в твой телескоп на заднем дворе, то она не тян вовсе и скорее всего даже не человек.
Аноним 29/05/15 Птн 15:51:22 #82 №183157 
>>183085
>Тонкий намёк на ничтожность человека
Это так в Библии написали? На самом деле это величие разума. Потому что мы сами можем быть создателями вселенной и всего всей, играющими в свое детище, временно забыв обо всем.
Аноним 29/05/15 Птн 15:56:47 #83 №183158 
>>182958
Индивидуумы пораженные релятивизмом головного мозга правда никогда не долетят. А вот эфиробоги долетят уже совсем скоро, причем на скорости выше световой.
Аноним 29/05/15 Птн 15:57:27 #84 №183159 
>>183155
> У тян такие же интересы как и у тебя
Ох, лол. Просто съеби, пожалуйста.
Аноним 29/05/15 Птн 16:08:24 #85 №183161 
>>183157
>>183158
>>183153

В Кащенко интернет провели
Аноним 29/05/15 Птн 16:22:37 #86 №183162 
>>183159
>Будем считать, что тян - не сферическая ТП, а хоть мало-мальски нормальная.
Сначала сиси с супом покаж и документ об образовании :3

Вот яркий педставитель нормальной тян.
https://ru.wikipedia.org/wiki/Гипатия
Аноним 29/05/15 Птн 16:33:20 #87 №183163 
Что за набег религиоблядей, кефирщиков и биопроблемников ITT?
Аноним 29/05/15 Птн 18:47:40 #88 №183179 
>>183102
Нихуя не понял, ты что аутист? Донеси свои мысли как подобает разумному существу.
Аноним 29/05/15 Птн 18:57:53 #89 №183181 
>>183163
Да хули я уже сам хоть и атеист хотел бы верить в бога.
Так хотя бы мысль о собственной ничтожности немного отпускает
Аноним 29/05/15 Птн 19:07:39 #90 №183184 
Пацаны, последнее время меня интересует один вопрос. Возьмем господствующее представление о расширяющейся вселенной. Я так понимаю расширяется именно пространство, а не вещество. Выходит оно или берется откуда-то(преобразуется из чего-либо) или растягиваясь истончается. Дальше - гравитация. Я понимаю ее как воздействие материи на пространство посредством массы. Чем больше масса, тем сильнее объект давит на пространство. Выходит, что со временем понадобится все меньшая масса для рождения черных дыр? А может дойти до того, что сначала все звезды сколлапсируют, а потом и планеты и прочите объекты? Дойдет ли до того, что наша реальность просто порвется? Или я сторчался уже?
Аноним 29/05/15 Птн 19:13:35 #91 №183185 
>>183184
Пространство не материально, оно может расширяться сколько угодно.
Материально оно вблизи материальных обьектов - там оно стягивается.
Т.е. материальные обьекты удаляются друг от друга, не меняясь внутри.
Аноним 29/05/15 Птн 19:19:45 #92 №183186 
>>183184
Так и будет, гугли про эпоху тьмы, в итоге все атомы распадутся
Аноним 29/05/15 Птн 19:25:52 #93 №183187 
>>183186
Необоснованные маняфантази
Аноним 29/05/15 Птн 19:30:50 #94 №183188 
>>183185
Не понял. Так материально или не материально?
Да и вопрос не в этом. Откуда оно берется? Ведь оно возникло с Большим взрывом и с тех пор расширяется
Аноним 29/05/15 Птн 19:32:02 #95 №183189 
>>183188
Не стоит вскрывать эту тему.
Аноним 29/05/15 Птн 19:35:48 #96 №183190 
>>183184
>Выходит оно или берется откуда-то(преобразуется из чего-либо) или растягиваясь истончается.
Ты сейчас описываешь пространство, как нечто материальное, что можно потрогать и интуитивно приписываешь ему свойства материи. Тебе надо уйти от этого.
Аноним 29/05/15 Птн 19:43:35 #97 №183192 
>>183190
Сможешь объяснить как его воспринимать? Я прекрасно понимаю, что это не материя, но в моделях демонстрирующих гравитацию, оно имеет некоторые свойства материи. Оно расширяется, оно прогибается, его можно прорвать.
Аноним 29/05/15 Птн 19:46:17 #98 №183193 
Без пары стаканов кефира не понять о чем там маняфантазируют релявы.
Аноним 29/05/15 Птн 19:48:11 #99 №183194 
>>183192
Могу объяснить, что модели- это модели. Условное изображение, для комфортного восприятия.
Аноним 29/05/15 Птн 21:05:37 #100 №183216 
>>164980
>ЗАЧЕМ НУЖЕН ТЕЛЕСКОП ЗА МИЛЛИАРД?
Очень медленный ответ на вопрос из позапрошлого треда.
http://tube.sfu-kras.ru/video/1967
Аноним 29/05/15 Птн 23:41:20 #101 №183254 DELETED
Напоминаем БВшникам, чтоб жизнь мёдом не казалась:

Вопросик из стареньких чёт всё не отвечают - вота мы видим Здеся скопление галлактик, ну, разлетается. А вот Тута - поближе и кучнее - неразлетается. Я грю - а чо так? А БВшники грят - ну, тип, Здеся расширение пространства, все дела, вот и разлетаются в этом пространстве ну тип того. А Тута другое дело - близко они скопления, за друг дружку гроветаццей держатся и неразлетаются. А я такой опять - а чё, а тип раньше, недалеко от БВ как вы грите, все были близко, все держались гроветаццей, чё такого произошло чё они прощёлкали момент и таки стали разлетаться? Раньше были в куче и держались, а теперь не в куче, чё за хуйня.

На это какая-та миморазь булькнула - "НУ ПОСЛИ БАЛШОВА ВЗРЫВА ПОЛМИЛЬЯРДА ЛЕТ ГОЛЛАКЕК НЕ БЫЛО, ВОТ И РАЗЛЕТЕЛОСЬ".
Аноним 29/05/15 Птн 23:50:09 #102 №183258 DELETED
>>183254
Пруфы есть, что галактики держались рядом после БВ?
Аноним 29/05/15 Птн 23:53:42 #103 №183259 DELETED
>>183258
Были рядом -> было гравитационное взаимодействие. Это сложно - моделировать то, что напрямую не написано в библии БВ, я понимаю, да.
Аноним 29/05/15 Птн 23:56:29 #104 №183261 DELETED
>>183259
Есть пруфы, что было гравитационное взаимодействие, а не сразу образовались несвязанные меж собой кластеры?
Аноним 29/05/15 Птн 23:58:43 #105 №183263 DELETED
>>183261
>Есть пруфы, что было гравитационное взаимодействие

Когда они были рядом? Было, отвечаю. Я тебя хоть раз обманывал?

>а не сразу образовались несвязанные меж собой кластеры

Фея наколдовала?
Аноним 30/05/15 Суб 00:04:39 #106 №183266 DELETED
>>183263
Неси пруфы, что взаимодействие было раньше
Аноним 30/05/15 Суб 00:11:24 #107 №183268 DELETED
Пересчитывали ли возраст вселенной с учётом эффекта "Тёмной Энергии"? Ведь разница должна быть не в разы, а на порядки.

То же с "Тёмной Массой" - если, как оказалось, в космосе "больше" собственно, массы(причём в разы), то ситуация с наблюдаемым состоянием вселенной(образование и эволюция структур и их составляющих) должна была произойти совсем за другое время.

Если всё уже корректировали, то корректировали ли Теорию Инфляции - ведь там, раз в исходных физ. данных вселенной(масса-энергия) разница в пару десятков раз, тоже должны были получиться совсем другие цифры.

А что тогда с Красным Смещением? Ведь, если массу\энергию\возраст надо корректировать, то и КС то же, ибо в его уравнении всё это фигурирует. Стало быть, значение, по которому измеряется сейчас размеры\расстояния тоже надо корректировать.

И последний вопрос. В 1947 году, согласно имеющимся на тот момент данным о массе\энергии\возрасте\размере вселенной было сделано предсказание существование Реликтового Излучения, и указана его предполагаемая температура С ТОЧНОСТЬЮ ДО ДЕСЯТОЙ ГРАДУСА. Как?
Аноним 30/05/15 Суб 00:13:44 #108 №183269 DELETED
>>183266
Пруф того, что гравитация была и раньше? Ну, хуй знает. Я конечно, попробую у друзей поспрашивать, мож кто её на мобилку заснял когда.
Аноним 30/05/15 Суб 03:19:10 #109 №183290 DELETED
>>183254
Петух не унимался, любит зайти говна поесть.
Покормлю петуха говном еще раз.
Первые звезды начали образовываться через 500 миллионов летплюс-минус лапоть, после БВ. Полмиллиарда лет прошло, прежде чем зажглись первые звезды, примерно тогда же начали формироваться первые галактики. Не те, невъебенные, которые сейчас объединены в скопления, а еще дрищеватые карланы. Нужно ли напоминать, что сила гравитационного взаимодействия напрямую зависит от массы? То есть, разорвать гравитационное взаимодействие двух легких тел гораздо легче, чем двух массивных. Но у петуха галактики возникли сразу, после БВ, в тот же момент и в том же виде. На все это у петуха ничего нет, кроме оскорблений и подрыва, нечего петуху на это возразить, он шипит в бессильной злобе и исходит на говно, кривляясь, думая, что это самый лучший аргумент в споре.
>>183268
>Ведь разница должна быть не в разы, а на порядки.
И тут петух должен бы выкатить что-нибудь в доказательства своих манязаключений на разы и порядки.
>должна была произойти совсем за другое время.
За какое?
>совсем другие цифры.
Какие?
>А что тогда с Красным Смещением?
С ним все заебись. Или ты про постоянную Хаббла вообще не слышал, или про ее коррекцию тебе тоже ничего не ведомо?
>С ТОЧНОСТЬЮ ДО ДЕСЯТОЙ ГРАДУСА. Как?
Жопой на косяк
ufn.ru/ufn94/ufn94_8/Russian/r948f.pdf
Аноним 30/05/15 Суб 07:48:37 #110 №183301 DELETED
>>183290
ЧСВ зашкаливает.
Аноним 30/05/15 Суб 08:37:32 #111 №183304 
>>183013
>но европеоиды блеванут от одной мысли
Ты просто с изнеженными пищевыми привычками человек. Ничего блевотного в самой концепции поедания червей нет. Вот если их вкусовые качества очень посредственны или даже отталкивающие, то да, может плохо зайти.
Аноним 30/05/15 Суб 09:00:56 #112 №183307 
Спутник ГСО неподвижен относительно поверхности земли, но крутится относительно звезд. А если тело беудет неподвижно относительно звезд, что тогда? Упадет?
Аноним 30/05/15 Суб 09:07:08 #113 №183309 
>>183307
Это невозможно. Звезды сами по себе движутся в галактической системе отсчета вокруг барицентра галактики, все с разной скоростью. А если еще и учесть, что дохуя объектов на небе это галактики и другие удаленные объекты, выходит вообще каша.

Если ты говоришь о неподвижности звездного неба с точки зрения спутника, то спутнику всего лишь надо повернуться в нужную точку и соблюдать соответствующую ориентацию (гироскопом или закруткой), никуда он не упадет.
Аноним 30/05/15 Суб 09:08:55 #114 №183310 
>>183309
Ну давай так - спутник неподвижен относительно центра земли. Или барицентра системы луна-земля. Ну ты понел, что я пытаюсь сказать.
Аноним 30/05/15 Суб 09:13:37 #115 №183313 
>>183310
Тогда пизданется, естественно.
Аноним 30/05/15 Суб 10:53:30 #116 №183325 DELETED
Объясните-ка, как можно обосновать отсутствие аккреционного диска у ЧД, если для наблюдателя время падения будет бесконечно?
Аноним 30/05/15 Суб 11:16:26 #117 №183328 
Что будет если в поясе Койпера пролетит небольшая ЧД?
Аноним 30/05/15 Суб 11:20:33 #118 №183330 
>>182891
А какие компуктеры используются в КА и на МКС? Интересует само железо(стоят ли там обычные хуи86 или нечто более экзотичное вроде PPC, MIPS или спарка?) и софт(какие ос используют?). Можно ли купить это железо и какая его цена для обычного васяна?
Аноним 30/05/15 Суб 11:26:06 #119 №183331 DELETED
>>183328
Чёрных дыр нет
Аноним 30/05/15 Суб 11:31:52 #120 №183332 DELETED
>>183331
Докажи с трех раз
Аноним 30/05/15 Суб 11:54:47 #121 №183333 DELETED
>>183332
Маньки кукарекают, что сверхмассивные чёрные дыры подчищают вокруг себя, однако они сейчас неактивных и не имеют аккреционного диска. Раз диска нет, значит он уже упал на сингулярность. Однако, мы знаем, что для стороннего наблюдателя падение объекта будет бесконечно долгим. Значит, ничто не падало на чд, раз аккреционного диска нет
Аноним 30/05/15 Суб 12:30:32 #122 №183337 
>>183129
Моя тян с интересом вместе со мной пялится на Луну и галилеевские спутники. Кольца Сатурна мой недотелескоп почти не показывает, да и живу я в городе-миллионике.
Аноним 30/05/15 Суб 12:31:42 #123 №183338 
>>183181
Слабак
Аноним 30/05/15 Суб 12:37:38 #124 №183340 
>>183330
http://geektimes.ru/post/241846/
Аноним 30/05/15 Суб 13:03:38 #125 №183342 
>>183181
Почему вам (нам) всё время нужны какие-то покровители, защитники, хранители?.. И как это отгоняет мысли о своём ничтожестве, если должно строго наоборот, их усиливать?.. Неужели так сложно принять, что за тобой не смотрит играющий в Симс бог?
Пардон за офф, просто с некоторых пор меня окружают отборные ПГМщики и прочие астралолюди.
Аноним 30/05/15 Суб 13:10:59 #126 №183343 
>>183330
Если ты имеешь в виду не ноуты, а компьютеры, стоящие в СУ, то там лютая мешанина разношерстной электроники и контроллеров. На российских модулях стоят свои БВК и контроллеры, на USOS свои. В фермах в приборных отсеках отдельная электроника - контроллеры заряда, поворота радиаторов, батарей, циркуляции теплоносителя и т.п. Некоторые контуры развязаны и автономны, некоторые связаны в общую сеть.
Аноним 30/05/15 Суб 13:21:05 #127 №183347 
>>183330
Если тебя интересует архитектура процов на СУ аппаратов - у нас в качестве БВК обычно стоят аргоновские изделия вроде ЦВМ-101 с кастомной системой команд. У американцев и ЕКА в БВК стоят обычно либо PowerPC и MIPS (например RAD750, который PPC), либо опять же кастомные всякие штуки, которых миллион и тележка разных бывает. Розетта та же использует кастомные Harris RTX2010, Dynex MAS31750 и ещё что-то. Дохуя их, в общем.
Аноним 30/05/15 Суб 13:22:13 #128 №183348 
>>183185
то есть закон сохранения на пространство не действует?
Аноним 30/05/15 Суб 13:24:32 #129 №183349 
>>183348
Закон сохранения чего?
Аноним 30/05/15 Суб 13:26:43 #130 №183350 
>>183349
Энергии конечно, балда!
Аноним 30/05/15 Суб 13:28:44 #131 №183351 
>>183342
Особенность человеческой психологии, идёт из детсткости восприятия мира.
Ничтожность по сравнению с создателем с лихвой компенсируется "фактом" того, что мы - его дети/создания.
Аноним 30/05/15 Суб 13:32:30 #132 №183353 
Можно ли как-то фигурально выебать вселенную ну или галактику там. Чтобы малафья полилася.
Аноним 30/05/15 Суб 13:37:04 #133 №183355 
>>183353
А чёрныю дыры, по-твоему, чем занимаются?
Аноним 30/05/15 Суб 13:46:47 #134 №183356 DELETED
>>183325
Тем, что аккреционный диск лежит вне горизонта событий. Может на экваторе сферы Шварцшильда должен присутствовать яркий поясок, но аккреционный диск с хуя ли?
>>183328
>пролетит небольшая ЧД?
Насколько небольшая?

Аноним 30/05/15 Суб 13:48:25 #135 №183357 
>>183350
Рассуждая о таких вещах, мы непременно будем прибегать к аналогиям, а значит, переносить наше восприятие мира на модель пространства. В представлении современной науки пространство - это метрика. Эдакая трёхмерная сеточка. Пространство не обладает энергией, энергией обладает вещество, располагающееся в этом пространстве. Вещество может влиять на метрику, искажая её, но не наоборот. При растяжении пространства вещество не "истончается", его не становится меньше, оно "разбегается". Увеличиваются расстояния между галактиками, например. Аналогия - точки на надуваемом воздушном шарике.
Что касается вопроса "откуда берётся пространство", то ты не там задаёшь вопросы, мы такие же дилетанты, как и ты =)

>Чем больше масса, тем сильнее объект давит на пространство. Выходит, что со временем понадобится все >меньшая масса для рождения черных дыр? А может дойти до того, что сначала все звезды сколлапсируют, >а потом и планеты и прочите объекты? Дойдет ли до того, что наша реальность просто порвется?
Почему меньше? Ведь рождающееся пространство однородно за исключением небольших флуктуаций. Оно ничем не отличается от пространства вокруг нас. Метрика неизменна. Во всяком случае, наука подтверждения "растягиванию" пока не нашла. И потом, Вселенная замкнута сама в себе, куда ей рваться?
sageАноним 30/05/15 Суб 14:15:31 #136 №183360 DELETED
>>183356
>аккреционный диск лежит вне горизонта событий
Но когда он будет проходить под горизонт, мы будем видеть его вечно, однако его нет. Шах и мат!
sageАноним 30/05/15 Суб 14:26:25 #137 №183362 DELETED
>>183360
>однако его нет
Лол, ты видел ЧД что ли, наркоман?
Аноним 30/05/15 Суб 14:43:00 #138 №183363 
>>182891
ААААААААААААААААААААААААААА ПОМОГИТЕ!
ПАДАЮ В ЮПИТЕРИАНСКУЮ АТМОСФЕРУ!
СЧЁТЧИК ГЕЙГЕРА ЗАШКАЛИВАЕТ НО Я В КАЧЕСТВЕННОМ СКАФАНДРЕ!
ЧТО ДЕЛАТЬ, АНОНЫ? ПИШУ С ТЕЛЕФОНА!
Аноним 30/05/15 Суб 14:48:08 #139 №183365 
>>183363
Тебе пизда, анон. Честью было раковать с тобой.
Аноним 30/05/15 Суб 14:48:35 #140 №183366 DELETED
>>183360
sageАноним 30/05/15 Суб 15:39:27 #141 №183377 DELETED
>>183362
Вот ты и упал в свою же яму. Если её не видели - значит её нет! Оправдывайся
Аноним 30/05/15 Суб 15:41:02 #142 №183378 DELETED
>>183377
Магнитных полей никто не видел, оправдывайся. Оправдывайся.
sageАноним 30/05/15 Суб 15:47:27 #143 №183379 DELETED
>>183378
А я и не говорил, что они есть
Аноним 30/05/15 Суб 15:52:45 #144 №183381 DELETED
Эфир тоже никто не видел но он есть
Аноним 30/05/15 Суб 15:54:43 #145 №183382 DELETED
>>183379
Я тебе привожу пример того, что твои выводы ущербны.
Аноним 30/05/15 Суб 15:56:08 #146 №183383 DELETED
>>183381
Да, но для веществ, вроде была отдельная доска.
Аноним 30/05/15 Суб 16:02:08 #147 №183384 
Вы никогда не задумывались что все эти способы полетов к другим системам просто придуманы фантастами, и на самом деле мы никогда не сможем это повторить.
Аноним 30/05/15 Суб 16:02:14 #148 №183385 
>>183347
>кастомные
Там нормальный ЦПУ со своей архитектурой или поделия на транзисторах/лампах и т.д? Какой софт? Можно ли эти PPC и мипсы где то купить?
Аноним 30/05/15 Суб 16:05:33 #149 №183386 DELETED
>>183378
Железные опилки тебе покажут магнитные поля.
Аноним 30/05/15 Суб 16:14:59 #150 №183387 DELETED
>>183381
Двачую адеквата
>>183382
Ты пруфы неси на черные дыры, а не маневрируй >>183386
Аноним 30/05/15 Суб 17:02:38 #151 №183394 
>>183385
Нормальные ЦПУ со своей архитектурой, естественно. Софт тоже кастомный, часто за основу берут какую-нибудь RTOS. Вообще вся космическая индустрия на 99% занимается выработкой кастомных решений, процент унификации мал по техническим причинам. Поэтому и стоит всё так.
> Можно ли эти PPC и мипсы где то купить?
Можно, если ты организация, что-то на них делающая, и регулятор не против.
Аноним 30/05/15 Суб 19:53:15 #152 №183437 DELETED
>>183381
>>183387
Эфироблядям тут не рады, уебывайте в свой саентач и несите свой бред там.
Аноним 30/05/15 Суб 20:02:24 #153 №183443 DELETED
>>183437
Не хуже бреда о ЧД
Аноним 30/05/15 Суб 20:24:21 #154 №183455 
>>183330
Читал, что на кубосаты ставят обычные тексасовские контроллеры. Просто иди и покупай.
Аноним 30/05/15 Суб 20:26:23 #155 №183458 DELETED
>>183437
Эфир не доказан - ИВО НЕТ УХОДИТИ!
ЧД не доказаны - ВРЁТИ, ЙЕСТЬ!
Аноним 30/05/15 Суб 20:31:48 #156 №183463 DELETED
>>183458
Напоминает пример далекого прошлого
> земля круглая не - НА КОСТЕР!
> доказательств существования бога нет - ВРЁТИ, ЙЕСТЬ!
Аноним 30/05/15 Суб 20:38:09 #157 №183465 DELETED
>>183458
А у эфироблядей:
ЧД не доказаны - ИХ НЕТ УХОДИТИ!
Эфир не доказан - ВРЁТИ, ЙЕСТЬ!
Аноним 30/05/15 Суб 20:48:17 #158 №183476 DELETED
>>183443
У ЧД есть хотя бы теория, с предсказательной силой и наблюдения косвенных признаков, у кефира нет нихуя, вообще, нуль без палки.
Аноним 30/05/15 Суб 20:57:55 #159 №183483 DELETED
>>183476
Какие ещё косвенные признаки?
Аноним 30/05/15 Суб 21:01:35 #160 №183486 DELETED
>>183476
Есть косвенные признаки что ты долбоеб а твоя мамаша шлюха.
Аноним 30/05/15 Суб 22:13:44 #161 №183503 
от чего зависит скорость вращения планеты?
Аноним 30/05/15 Суб 22:17:35 #162 №183505 
>>183503
Вес, размер, присутствие спутников, которые будут гасить скорость вращения
Аноним 30/05/15 Суб 22:21:07 #163 №183507 
>>183505

>>183505
>Вес
йоба_смеется.жпг
Аноним 30/05/15 Суб 22:34:28 #164 №183516 
Поясните, все эти голубые карлики и красные гиганты, реально ли мы видели бы их голубыми/красными, если бы смотрели на них глазами?
Алсо, какие задачи у коричневых карликов?
Аноним 30/05/15 Суб 22:34:40 #165 №183517 
14330144803730.png
>>183507
Но если вес - сила, с которой гравитация действует на объект, то вес планеты зависит от веса планеты!
Аноним 30/05/15 Суб 22:36:00 #166 №183518 DELETED
>>183516
Нет, они все светят желтым. Это всё маняобозначения ученых
Аноним 30/05/15 Суб 22:39:45 #167 №183520 
>>183517
>вес - сила, с которой гравитация действует на объект
Это сила притяжения.
Аноним 30/05/15 Суб 22:41:38 #168 №183522 
>>183517
У планеты нет веса.
У планеты есть масса и гравитация.
Аноним 30/05/15 Суб 22:49:56 #169 №183525 
Вес - это сила с которой тело действует на опору или наоборот, хуй знает.
мимо физику не учил
Аноним 30/05/15 Суб 23:05:22 #170 №183529 
Поясните, все эти голубые карлики и красные гиганты, реально ли мы видели бы их голубыми/красными, если бы смотрели на них глазами?
Алсо, какие задачи у коричневых карликов?
Аноним 30/05/15 Суб 23:09:05 #171 №183530 
>>183525
Силы без опоры не бывает - законы ньютона
Аноним 30/05/15 Суб 23:48:09 #172 №183537 
>>183384
ты про гипердрайв чтоли или про то что чертежей не приложили ?
Аноним 30/05/15 Суб 23:53:06 #173 №183538 
>>183394
> процент унификации мал по техническим причинам.
не по техническим наверно, просто отрасль находиться в детской фазе

>>183529
коричневые карлики это заначка для постхуманов
Аноним 30/05/15 Суб 23:59:38 #174 №183539 
>>183529
Нет конешно. Если бы мы смотрели на них глазами, мы умерли бы через несколько секунд.
Вообще нирикаминдую открывать глоза в открытом касмасе.

Людей посмотревших, сойдут с ума
Аноним 31/05/15 Вск 00:17:41 #175 №183543 
>>183194
Как же он хорош. Вот это его "you don't like it?" просто божественно.
Аноним 31/05/15 Вск 00:26:46 #176 №183547 
>>183538
Согласен. Везде, и в природе, и в развитых технологиях наблюдается один и тот же принцип: схожие проблемы рождают схожие решения.
Проблемы у ракетных технологий одни и те же, но вот решения - каждый лепит во что горазд. Со временем конечно хаос устаканиться, и девайсы. решающие одни и те же проблемы, станут унифицированными - как сейчас например у автомобилей унифицированны многие узлы: даже если они отличаются по форме и размерам, конструкция у них одна и та же, не смотря на разных производителей.
Аноним 31/05/15 Вск 01:41:15 #177 №183630 DELETED
>>183539
>Если ты долго смотришь в бездну, то бездна тоже смотрит в тебя
Аноним 31/05/15 Вск 01:42:27 #178 №183631 DELETED
>>183630
>звезда
>бездна
Ясно
Аноним 31/05/15 Вск 01:50:59 #179 №183645 DELETED
>>183631
Звезды это боги.
Аноним 31/05/15 Вск 01:51:19 #180 №183646 DELETED
>>183630
>>183631
Два дебила - это сила.
Аноним 31/05/15 Вск 02:38:12 #181 №183701 DELETED
>>183646
Сила подразумевает воздействие на опору, школьник. Бегом учить законы Ньютона!
Аноним 31/05/15 Вск 03:11:19 #182 №183765 DELETED
>>183701
>засрали и просрали весь пейсач
>воздействие на опору
Exactly.
Аноним 31/05/15 Вск 03:23:31 #183 №183772 DELETED
>>183701
Хорошо, тогда на чем основана материя?
Сила, поле и инерция - все это связано с энергией, сила меняет энергию, поле переносит энергию, а инерция хранит энергию.
Это все, или есть еще основы материи, то, из чего и благодаря чему устроен, существует, и работает этот мир?
Аноним 31/05/15 Вск 04:03:27 #184 №183795 
>>183537
И то и другое.
Аноним 31/05/15 Вск 04:40:43 #185 №183798 
>>183795
Ну подводные лодки тоже были фантастикой, так же как компьютеры, самолеты и ракеты.
Аноним 31/05/15 Вск 05:05:07 #186 №183802 
>>183798

Но голову под воду всегда можно было сунуть. И птички показывали, как летать. И реактивное движение можно посмотреть было.
Аноним 31/05/15 Вск 05:15:05 #187 №183803 
>>183802
Ну так и с гипердрайвом все тоже - космические тела нам показывают как искривлять пространство.
Аноним 31/05/15 Вск 09:46:48 #188 №183820 DELETED
>>183803
Твоя мамка такая жирная что испривляет пространство-время вокруг себя.
Теоретически искривлять могу ЧД. Но это ненаблюдаемые теоретические обьекты.
Аноним 31/05/15 Вск 10:01:55 #189 №183822 DELETED
>>183820
>Но это ненаблюдаемые теоретические обьекты.
А я то думал куда ты делся, думал тебя вместе с G2 облаком распидорасило.
Аноним 31/05/15 Вск 10:11:38 #190 №183823 DELETED
ЧД
-ненаблюдаемы
-не все модели ОТО допускают их существование
-скорее всего вовсе не могут образоваться
Аноним 31/05/15 Вск 10:44:18 #191 №183826 DELETED
>>183820
ЧД не существует, выяснили уже
Аноним 31/05/15 Вск 10:50:32 #192 №183828 DELETED
>>183823
Тогда что придаёт вращение нашей галактике?
Аноним 31/05/15 Вск 10:52:55 #193 №183829 DELETED
>>183828
Высокая плотность звёзд в центре
Аноним 31/05/15 Вск 13:11:36 #194 №183838 
>>182967
>4 световых года
>ничо, до Луны летали же
Аноним 31/05/15 Вск 13:13:45 #195 №183839 DELETED
>>183828
Скопление газа в центре, которое очень много весит
Аноним 31/05/15 Вск 13:19:48 #196 №183841 DELETED
>>183839
Много весит твоя мамаша, а у газа большая масса.
Аноним 31/05/15 Вск 13:22:38 #197 №183842 DELETED
>>183841
Очередная ОТО-блядь порвалась, как по нотам. Как можно верить в не существующую хуйню (ЧД), которая по всем законам логики не может существовать?
Аноним 31/05/15 Вск 13:25:26 #198 №183843 DELETED
>>183842
Эта маня и в Бога поверит.
Аноним 31/05/15 Вск 13:25:36 #199 №183844 DELETED
ефир ест
Аноним 31/05/15 Вск 13:26:20 #200 №183845 DELETED
>>183842
Вскрой тему. По каким ещё законам логики?
Аноним 31/05/15 Вск 13:32:28 #201 №183847 DELETED
Эфиробоги дали пососать чернодырщикам.
Аноним 31/05/15 Вск 13:47:31 #202 №183848 DELETED
>>183845
Я уже спрашивал, как ЧД могут толстеть для сторонних наблюдателей? Частенько говорят, что ЧД всё уже подчистили вокруг, и поэтому не излучают. То есть аккреционного диска у них нет, вокруг них тоже ничего нет. Куда делся диск, если падение на ЧД стороннему наблюдателю кажется бесконечным? Шах и мат
Аноним 31/05/15 Вск 13:58:22 #203 №183849 DELETED
>>183848
А тебе уже отвечали >>183358.
И кстати, как там ваша кефиродинамика объясняет гравитационное линзирование?
А где кефирные комментарии к https://2ch.hk/spc/res/148354.html
А про ответы в стиле пикрилейтед я вообще молчу, у кефироопущенок каникулы начались видимо.
Аноним 31/05/15 Вск 14:02:15 #204 №183850 
>>183849
А пруфов так и не принесли, что время выпадения конечно
Аноним 31/05/15 Вск 14:05:09 #205 №183853 DELETED
>>183849
Алсо, что не так с этим пульсаром и эфиром, мань? В эфире тоже могут быть складки, в которые мог нырнуть пульсар
Аноним 31/05/15 Вск 14:51:09 #206 №183858 DELETED
А теперь я поясню, почему существует эфир. Во вселенной, как известно, вакуум. И подтверждение существования эфира таково - эфир существует, т.к вакуум обладает свойствами, а ничто, как известно, свойствами обладать не может. Такой вот ход конём. А деформации в пространстве объяснимы сжатием/расширением эфира на каком-то участке
Аноним 31/05/15 Вск 14:54:35 #207 №183859 DELETED
>>183858
Гениально. Шах и мат эфирохейтеры.
Аноним 31/05/15 Вск 15:42:06 #208 №183868 
14330761264750.jpg
Анон, я угорел по столкновению Млечного Пути с Андромедой. Есть несколько вопросов:
1. Космос станет гуще? В смысле — уменьшится ли расстояние между звёздами путём увеличения их количества?
2. Будет такая хуйня, как на пике?
3. Ну, и почитать чего на эту тему есть?
Аноним 31/05/15 Вск 16:14:41 #209 №183874 DELETED
>>183868
НЕ-БУ-ДЕТ. ТВЁРДО И ЧЕТКО
Аноним 31/05/15 Вск 16:18:59 #210 №183875 DELETED
>>183874
ПОРАШНИК НЕ ПАЛИТСЯ!
Аноним 31/05/15 Вск 16:19:22 #211 №183876 DELETED
уфир ест всем ясно ето ? ?
Аноним 31/05/15 Вск 16:23:53 #212 №183877 DELETED
>>183876
Да. Ясно-понятно.
Аноним 31/05/15 Вск 16:32:28 #213 №183880 DELETED
Что тут происходит?
Аноним 31/05/15 Вск 16:33:29 #214 №183882 DELETED
>>183826
Ох, нихуя себе, ананимный ученый с двощей выяснил, что ЧД не существуют, астрофизики всей планеты ходят смурные с окей-фейсом.
>>183829
Надо же, а вокруг чего же раскручиваются звезды, в центре галактики?
>>183839
Кторый дохуя весит, скопился в самом центре и не отсвечивает, не иначе, как холодный, аж пиздец, потому и не излучает.
>>183842
Посмотрите, кефиропидор обвиняет кого-то в вере в несуществующую хуйню.
>>183858
Они говорят, что физический вакуум - это ничто, и тут же срут себе на голову, говоря, что вакуум обладает свойствами. Браво.
>>183880
У эфироблядей каникулы.


Аноним 31/05/15 Вск 16:38:33 #215 №183884 DELETED
>>183882
Там тёмная материя, суть сгущеный эфир.
Аноним 31/05/15 Вск 16:38:50 #216 №183885 DELETED
>>183858
Схуяли во вселенной вакуум? Во вселенной нихуя не чистый вакуум. Там много всякого дерьма в нем намешано. Мат и шах, шахматист.
Аноним 31/05/15 Вск 16:40:39 #217 №183887 
http://www.youtube.com/watch?v=fMNlt2FnHDg
>>183868
1. На какое-то время станет, когда если один диск будет напрямую проходить, через другой. Но сразу слияния не будет, какое-то количество звезд будет выброшена из галактик.
2. Будет
3. В интернете, по запросу galaxy collision.
Аноним 31/05/15 Вск 16:41:19 #218 №183888 DELETED
>>183884
А его на хлеб намазать можно?
Аноним 31/05/15 Вск 16:42:47 #219 №183889 DELETED
>>183888
Можно нанюхаться и писать на двощах, что релявы соснули, больше никакого применения эфиробляди своему кефиру не нашли.
Аноним 31/05/15 Вск 16:46:38 #220 №183890 DELETED
>>183889
уфир ест
Аноним 31/05/15 Вск 16:48:11 #221 №183891 DELETED
>>183885
>Во вселенной нихуя не чистый вакуум
Правильно, там эфир, чем и объясняются различные взаимодействия, гравитация и т.д. Те же маняискажения пространства ОТО блядей - обычные сжатия эфира на определённом месте
Аноним 31/05/15 Вск 16:53:30 #222 №183892 DELETED
>>183891
Это все промысел Божий, а не какой то там ефир.
Аноним 31/05/15 Вск 16:55:53 #223 №183893 DELETED
>>183892
Богоблядь закукарекала
Аноним 31/05/15 Вск 16:57:22 #224 №183895 DELETED
>>183884
>сгущеный эфир
По госту хоть сгущённый?
Аноним 31/05/15 Вск 17:00:16 #225 №183896 DELETED
>>183895
Ну он плотный очень и инвертирован по знаку потому плохо видится
Аноним 31/05/15 Вск 17:04:58 #226 №183898 DELETED
ефир ест норкоманы хотит те док-ва итт ? ?
Аноним 31/05/15 Вск 17:06:27 #227 №183899 DELETED
>>183898
Хотим
Аноним 31/05/15 Вск 17:07:37 #228 №183900 DELETED
>>183896
Что еще расскажешь?
>>183898
В этом итт треде?
Аноним 31/05/15 Вск 17:09:49 #229 №183901 DELETED
>>183898
давай
Аноним 31/05/15 Вск 17:13:04 #230 №183905 DELETED
>>183900
Ну он как бы в двух реальностях находится поэтому почти не могу заметить, вот увидели что-то и назвали это тёмной материе а это на самом деле разновидность эфира
Аноним 31/05/15 Вск 17:14:15 #231 №183907 DELETED
>>183905
Более плотный эфир. Двачую
Аноним 31/05/15 Вск 17:21:25 #232 №183910 DELETED
>>183907
Ну да. Он размазан между вселенными поэтому его плотность бесконечно велика, но одновременная конечна (в нашей вселенной).
Аноним 31/05/15 Вск 17:25:41 #233 №183912 DELETED
>>183910
А в другой вселенной тоже конечна?
Аноним 31/05/15 Вск 17:31:44 #234 №183913 DELETED
>>183899
>>183901
ефир ето я
я существуюю значит я ефир если ефир ест то он ест
ясно
Аноним 31/05/15 Вск 17:34:03 #235 №183914 DELETED
Эфиробляди обезумели, даже Эйнштейн не сможет с ними совладать!
Аноним 31/05/15 Вск 17:35:37 #236 №183915 
>>183887
Спасибо. Пойду обмазываться и дрочить.
Аноним 31/05/15 Вск 17:36:25 #237 №183916 DELETED
Нихуясе тут балаган. Эфир кефир и боженька. Что я пропустил?
Аноним 31/05/15 Вск 17:36:49 #238 №183917 DELETED
>>183916
Сегодня 31 мая.
Аноним 31/05/15 Вск 17:37:00 #239 №183918 DELETED
>>183914
Эйнштейн с ними давно совладал. Настолько, что эфиробляди могут кукарекать про свой эфир толькуо на двощах. Во всех приличных местах их гонят ссаными тряпками.
Аноним 31/05/15 Вск 17:38:10 #240 №183919 DELETED
>>183918
ефир ест
Аноним 31/05/15 Вск 17:38:12 #241 №183920 DELETED
>>183916
В школке занятия кончилось, ИТТ и в разделе второй день беснуются две эфиробляди.
Аноним 31/05/15 Вск 17:40:20 #242 №183921 DELETED
ефир ест ефир поел
есть
Аноним 31/05/15 Вск 17:41:30 #243 №183922 DELETED
>>183919
А скиньте чорную дыру
Аноним 31/05/15 Вск 17:41:36 #244 №183923 DELETED
>>183920
Нас минимум три. Один рассказывает про аккреционный диск, второй про уфир, а я доказываю ОТО блядям теорию на примере вакуума
Аноним 31/05/15 Вск 17:42:35 #245 №183924 DELETED
еир ест вы боитес
Аноним 31/05/15 Вск 17:43:39 #246 №183925 DELETED
>>183923
Петухи_взбунтовались.avi
Аноним 31/05/15 Вск 17:45:48 #247 №183927 DELETED
Вот МКС летит в вакууме. Что заставляет ее лететь не прямолинейно, а все время прижиматься в сторону нашей бренной? Откуда такая силища? Откуда столько энергии на постоянное изменение траектории? Кругом нее же только вакуум.
Аноним 31/05/15 Вск 17:46:13 #248 №183928 DELETED
>>183922
чоная дира их не бивает
вот
док-во нет ничего ест токо ефир
Аноним 31/05/15 Вск 17:46:32 #249 №183929 DELETED
>>183927
>Кругом нее же только вакуум.
А Земли типа нет? Он в карман спряталась?
Аноним 31/05/15 Вск 17:47:00 #250 №183930 DELETED
>>183928
а хде тют ефир?
Аноним 31/05/15 Вск 17:47:11 #251 №183931 DELETED
>>183927
Кругом эфир. В том и дело, что в вакууме не было бы взаимодействия, поэтому вселенная заполнена эфиром
Аноним 31/05/15 Вск 17:49:05 #252 №183932 DELETED
>>183930
чорный
Аноним 31/05/15 Вск 17:50:32 #253 №183933 DELETED
>>183932
но веть и чорная дира тоже чорная, как ти увидешь чорное на чорнём?
Аноним 31/05/15 Вск 17:51:19 #254 №183934 DELETED
>>183929
Так ты не сечешь фишку, братан. Земля вона где далеко и разделяет ее с МКС прослойка вакуума. Так почему МКС прижимается к Земле? У меня только один вывод на основании логики - вакуум давит на вещество станции.
Аноним 31/05/15 Вск 17:52:04 #255 №183935 DELETED
>>183934
Гравитации срать на твой вакуум. Понимаешь? Срать.
Аноним 31/05/15 Вск 17:52:41 #256 №183936 DELETED
>>183933
сука блядь пашол нахе р
Аноним 31/05/15 Вск 17:54:38 #257 №183938 DELETED
>>183931
Получается среда вакуума насыщена энергиями так что может нехило довить на спутники и прочие шатлы, сообщая им движение.
Аноним 31/05/15 Вск 17:54:39 #258 №183939 DELETED
>>183935
Но как это ничто может взаимодействовать с чем-то?
Аноним 31/05/15 Вск 17:54:56 #259 №183940 DELETED
>>183936
Кефироблядь подорвалась.
Аноним 31/05/15 Вск 17:55:44 #260 №183941 DELETED
>>183939
>ничто
Какое-такое ничто? Оно никак не действует. Это всё планетка.
Аноним 31/05/15 Вск 17:55:53 #261 №183942 DELETED
>>183940
Он сам с собой ведет неспешную беседу.
Аноним 31/05/15 Вск 17:58:00 #262 №183943 DELETED
>>183940
иди шахты руби бля дт как бамб ит ЕФИР ЕСТ ЕСТ ЕСТ ЕСТ ааааа..... какие тупие боже ефир ест потому что он ест и вобще вакум нет значит ест
он ест мы его може мвдитеь ест ест ест еир здохните здохните ок
Аноним 31/05/15 Вск 17:58:21 #263 №183944 DELETED
>>183935
Ты понимаешь логику? Кругом тела вакуум. Тело получило ускорение. Значит получило из вакуума. Не боженька же его чудесным образом ускорил. Это явление называют гровитацей. А ты говоришь - срать. Нелогично.
Аноним 31/05/15 Вск 17:59:25 #264 №183945 DELETED
>>183941
Как гравитация может действовать в условиях вакуума?
Аноним 31/05/15 Вск 18:01:16 #265 №183948 DELETED
>>183912
Пока наука не может дать ответа на этот вопрос. Физические законы других вселенных могут отличаться
Аноним 31/05/15 Вск 18:02:19 #266 №183949 DELETED
>>183923
А я про плотный эфир. Нас минимум четверо. Адепты эфира
Аноним 31/05/15 Вск 18:04:02 #267 №183951 DELETED
Вы что, ебанулись тут все?
Аноним 31/05/15 Вск 18:04:04 #268 №183952 DELETED
>>183935
Блин ну вы даёте. Просто эфир давить на станции намного сильнее со стороны космоса, а со стороны земли очень слабо, вот и МКС и прочие звездолёты прижимаются к ней.
Аноним 31/05/15 Вск 18:05:32 #269 №183953 DELETED
Схороните тред на архиваче. Не устаю проигрывать. Моча не три, подожди до утра
Аноним 31/05/15 Вск 18:10:54 #270 №183955 DELETED
>>183949
Создал сообщество http://vk.com/club95260611. Это наше место для собраний, вступайте. Мы же должны как-то координироваться.
Аноним 31/05/15 Вск 18:13:22 #271 №183956 
Поясните недалекому азы про теорию струн.
Аноним 31/05/15 Вск 18:13:36 #272 №183957 DELETED
>>183955
http://vk.com/club95260611
Аноним 31/05/15 Вск 18:14:08 #273 №183958 DELETED
О Господе, что вы несете? Надеюсь вы все на этом форуме попадете в ад. Боженька все делал делал, а вы на какой то ефир все сбрасываете. Земля лежит на трех китах, а те, в свою очередь, на черепахе. Никаких спутников и тем более других Землей нет. Вы, дураки, еще наверное телевизор не смотрите, а там то всю правду говорят. Да что с вами спорить, богохульники, аж противно.
Аноним 31/05/15 Вск 18:18:55 #274 №183960 DELETED
>>183951
Нет, тут одни адекваты. Ты считаешь иначе? Тогда поясни нам, как гравитация работает в вакууме? Еще Ньютон вывел непротиворечивый закон - силы без опоры не существует. Так где опора гравитации? И через что она переносит энергию по вакууму? Напоминаю - гравитонов нет. И откуда она берет эту энергию? Это что, типа вечный двигатель такой? Хуле она не кончается? Это нарушение закона сохранения энергии.
Так поясни все это, или уткнись - кефироблядки по крайней мере адекватнее отоблядков, т.к. у кефироблядков есть ответы на эти вопросы, а у отоблядков вместо ответов одно невнятное кукарекание.
Если кто-то не может дать ответ, а вместо этого проповедует о какой-то магии - этот человек бесполезен для науки. Так хуле слушать отоблядков, если они не могут дать ответ?
Аноним 31/05/15 Вск 18:21:05 #275 №183961 
>>183956
>Азы
Пространство не трехмерно, а многомерно, просто другие измерения не "развернулись", и находятся в скукоженном состоянии, от того мы в них не живем и они с нами не взаимодействуют.
Частицы - это ОСHE маленькие одномерные вибрирующие струны.
Что с этим делать? Хуй знает, но выглядит круто.
Почитать можно у Брайана Грина, он все расписывает по хардкору.
Аноним 31/05/15 Вск 18:24:02 #276 №183964 DELETED
>>183960
>у кефироблядков есть ответы на эти вопросы,
>поехавших дебилов, без пруфов, без нихуя.
>ответы вопросы,
Аноним 31/05/15 Вск 18:28:36 #277 №183966 DELETED
>>183960
А хуле слушать эфироблядей, у которых волшебный эфир заполоняет всё и объясняет всё и прям сразу, никаких экспериментов, никаких рассчётов, им достаточно книжки ацюковского. Ничего не надо проверять, ни о чём думать - на все вопросы один идеальный ответ - эфир. А что, сомневаться в теории - это для отоблядков и прочих неуверенных в себе людей, нам бог дал эфир, который обладает всеми свойствами по желанию, и который не надо искать - нужна лишь вера. И что же это никто кроме пары школьников с сосача не догадался?
Аноним 31/05/15 Вск 18:28:54 #278 №183967 DELETED
>>183960
Какая гравитация? Что ты вообще несешь?
Я спрашиваю совсем другое - вы тут ебанулись совсем, или окончательно? Что за цирк ебаный?
Аноним 31/05/15 Вск 18:29:34 #279 №183968 DELETED
Шах и мат, релявобляди. Эфирная лаборатория им. Св. Великомученника Ацюковского опубликовала снимки с телескопа тонких материй в инфраэфирном диапазоне, на котором отчётливо видно, что никакого вакуума нет - всё пространство заполнено эфиром в таре по 100г.
Аноним 31/05/15 Вск 18:30:33 #280 №183969 DELETED
>>183967
>Что за цирк ебаный?
>Тред гениальных вопросов и каких получится ответов
Обычный такой вечерний, почти летний цирк.
Аноним 31/05/15 Вск 18:41:18 #281 №183975 DELETED
>>183968
Аноним 31/05/15 Вск 18:48:44 #282 №183976 DELETED
>>183945
Гравитация порождает силу притяжения путём искривления пространства.
Аноним 31/05/15 Вск 18:57:11 #283 №183978 DELETED
>>183966
Святой Ацюковский Эфиродинамический свои труды никому не навязывает, а призывает вскрывать суть действительности, как она устроена. Говорит что в его маня-теории обязательно найдутся ошибки, которые будут исправлены, эфиродинамика она жи очень молода. Тут одно ясно 100% эфир ест и его нужно изучать, чтобы иноплане-тяночки уважали.
Аноним 31/05/15 Вск 18:59:01 #284 №183980 DELETED
>>183976
>порождает силу притяжения
>путём искривления пространства
Энто как?
Аноним 31/05/15 Вск 19:02:27 #285 №183981 DELETED
>>183980
У Энтео спроси.
Аноним 31/05/15 Вск 19:03:34 #286 №183982 DELETED
>>183980
Вот так. Непонятно? Помолись святому Ацюковскому
Аноним 31/05/15 Вск 19:07:14 #287 №183985 DELETED
Так что там со Стрельцом-А? Эфиродауны уже выдумали что может иметь массу 4,5 ляма солнц и помещаться внутри сферы в 45 а.е.? В любом случаю им живительную релятивистскую струю.
Аноним 31/05/15 Вск 19:07:34 #288 №183986 DELETED
>>183976
И? Энергия откуда, я спрашиваю? Чому она не кончается? Вам вечный двигатель глаза не мозолит, случаем?
Аноним 31/05/15 Вск 19:08:45 #289 №183987 DELETED
>>183982
лож
Аноним 31/05/15 Вск 22:24:29 #290 №184130 DELETED
>>184083
Закон ньютона тоже работает. А ото - не теория всего. Так что вывод неверный.
Аноним 31/05/15 Вск 22:43:02 #291 №184148 DELETED
>>184130
А при чём тут теория всего? Предсказания ОТО выполняются, используются в реальной жизни. Следовательно, она работает. Я не отрицаю того, что её нужно допиливать
Аноним 31/05/15 Вск 22:45:25 #292 №184153 DELETED
>>184130
То есть, к Ньютоновской теории тяготения, которая является частным случаем теории относительности, претензий нет?
Аноним 01/06/15 Пнд 06:48:27 #293 №184340 
Есть запись запуска Протона с камеры на борту?
Аноним 01/06/15 Пнд 06:56:50 #294 №184341 
>>184340
На сколько я знаю, на Протоны камеры никогда не устанавливались.
Аноним 01/06/15 Пнд 07:47:43 #295 №184346 
>>184341
У них валятся ракеты каждый год, а они не могут камеру поставить для диагностики. Ну не придурки ли?
Аноним 01/06/15 Пнд 12:24:18 #296 №184363 
Скажите, если увеличить или уменьшить соотношение азот/кислород в атмосфере, в какую сторону это сказывается на аэродинамике, где летать будет проще?
Аноним 01/06/15 Пнд 12:32:29 #297 №184365 
Почему орбиты планет лежат в приблизительно одной плоскости (а не в плоскостях, пересекающихся под 90 градусов, например)?
Почему наша галактика плоская?
Аноним 01/06/15 Пнд 12:41:06 #298 №184367 
Возьми гирю на вытянутые руки, крутись с ней вокруг своей оси.

Почему гиря крутится в одной плоскости, перпендикулярной оси вращения?
Аноним 01/06/15 Пнд 12:48:51 #299 №184368 
>>184367
Дурак?
Аноним 01/06/15 Пнд 13:02:20 #300 №184369 
14331529404650.gif
>>184365
Вот как-то так
Аноним 01/06/15 Пнд 13:03:01 #301 №184370 
>>184365
http://www.m31.spb.ru/archive/books/galaxies/chapter3.htm
Аноним 01/06/15 Пнд 13:47:05 #302 №184374 
>>184369
Хули оно вращается, ёба?
А планеты потом почему вращаются (причём в разные стороны)?
И как они отрываются от того, что потом становится звездой в системе?
Аноним 01/06/15 Пнд 13:49:00 #303 №184375 
>>184374
Анус у тебя отрывается. А вращаются они потому что распределение массы не равномерное. И при сжатии массы происходит закручивание облака.
Аноним 01/06/15 Пнд 14:01:59 #304 №184378 
Запускали ли зонды перпендикулярно вращению планет? Может, там что-то есть интересное?
Аноним 01/06/15 Пнд 14:03:19 #305 №184379 
>>184375
Падажди, планеты же возникают из того же облака, что и звезда, нет? Так почему они планеты, а не часть звезды?
Аноним 01/06/15 Пнд 14:08:19 #306 №184380 
>>184379
Да, но линейные скорости в центре облака (где звезда) и на окраинах сильно разная. Вот их и отрывает и крутит отдельно. А ваще симуляции есть, я даже сам делел в алгоудо или как там ее.
Аноним 01/06/15 Пнд 14:10:53 #307 №184381 
>>184380
Какую книжку бы почитать, чтобы подробнее узнать о технических аспектах создания планет, звёзд етц? Желательно не попсу а учебник, но без 9000 матана.
Аноним 01/06/15 Пнд 14:13:36 #308 №184382 
>>184381
Хз, я погроммист, я модельку состряпап и пофапал

Подожги олдфаги выйдут вечером из кефирной комы
Аноним 01/06/15 Пнд 14:14:13 #309 №184383 
*Или подожди.
Аноним 01/06/15 Пнд 15:16:18 #310 №184399 
>>184382
Альвен X., Аррениус Г. Эволюция солнечной системы
Общий курс астрономии Э.В. Кононович, В. И. Мороз. Глава 9.
Аноним 01/06/15 Пнд 15:34:41 #311 №184404 
>>184381
>>184399
Мама, я в космосе Аноним 01/06/15 Пнд 16:45:39 #312 №184421 
14331663394200.jpg
Привет, анонимус, у меня вопросы безотлагательные и требуют развернутого и грамотного ответа.

1. Что держит человечество на Земле? Единственное, чем занимаются люди - занимаются самовыпиливанием с попутным истощением ресурсов родной планеты, если коротко. У людей есть все необходимое, так почему бы не направить весь человеческий потенциал в нужное русло? Россиюшка уже давно проебалась как с обустраиванием своих территорий, так и с установлением биполярного мира, а талантливых ученых тут и так предостаточно, так почему бы не поставить национальной задачей первыми высадиться на Марсе, например?

2. Если учитывать, что Вселенная бесконечна, получается и вероятность существования иной формы жизни во Вселенной тоже бесконечна?

3. Какова вероятность, что иные формы жизни во Вселенной тоже могут быть гуманоидными? В смысле, голова сверху, две руки-две ноги и так далее.
Аноним 01/06/15 Пнд 17:03:40 #313 №184425 
>>184421
1. Нужны деньги. Нет, серьезно, нужно очень много денег, которые отобьются очень не скоро и не напрямую.
2. Да.
3. Исходя из пункта 2, можно предположить, что какая-то часть из всего разнообразия форм жизни должна быть гуманоидной.
Аноним 01/06/15 Пнд 17:11:18 #314 №184427 
>>184425
Ну а если основать какой-нибудь альянс будущих государств-колонистов, где все население будет каким-либо образом занято в космической сфере, почему бы не направить весь капитал туда (в эту сферу)?
Аноним 01/06/15 Пнд 17:13:35 #315 №184428 
>>184425
вдогонку
Если сузить масштаб второго вопроса до размеров Млечного Пути, какова вероятность, что в нашей галактике существует другая жизнь/другая гуманоидная жизнь?
Аноним 01/06/15 Пнд 17:42:18 #316 №184433 
>>184428
Как минимум 1/n где n - количество планет в млечном пути
n>>m, где m - количество звезд
Аноним 01/06/15 Пнд 17:51:27 #317 №184437 
>>184427
Это можно, но пока нет ясности, зачем нужен Марс, именно в экономическом плане. То есть отправить туда роботов гораздо дешевле, а обычный флаговтык - слишком дорогое мероприятие. У нас где-то было несколько тредов со срачем на около этой темы. Нужна так же политическая воля и желание тратить деньги. На данный момент НАСА разрабатывает полет на Марс, было несколько осторожных заявлений, что в НАСА хотели бы видеть Россию, в качестве участника этого мероприятия, на каких-нибудь ролях. Но пока ничего конкретного. Предполагается, что в США создадут носитель SLS, корабль Орион и еще одни модуль, для полета на Марс. Почитать что-нибудь можно на сайте НАСА http://www.nasa.gov/feature/goddard/science-drives-nasas-journey-to-mars
>>184428
В галактике Млечный путь, по разным оценкам, от 200000 до 450000 звезд, почти у каждой звезды обнаруживаются экзопланеты, сейчас среднее количество экзопланет две на звезду, очевидно, что это число будет только расти. Ждем результаты миссии Gaia. Это довольно много, одна из проблем состоит в том, что человечество до сих пор точно не уверено, существует ли возможность разумной жизни совершенно непохожей на нашу. Сейчас известно, что некоторые организмы могут жить при очень экстремальных условиях, но может ли быть в таких условиях более сложная жизнь. Это во-первых, во-вторых, есть проблема одновременности. То есть может где-то жизнь еще не зародилась, а где-то уже давно исчезла. Человеку всего пара миллионов лет, из них более-менее разумных около 200-100 тысяч, а технологически развитых не более 4-5 тысяч. При том, люди научились покидать планету, хоть и недалеко всего около 50 лет назад. Это ничтожное время, в сравнении с тем, насколько стара Галактика. В общем, если даже прямо сейчас есть разумные существа, похожие на нас, в нашей Галактике, то мы можем этого никогда не узнать.
Аноним 01/06/15 Пнд 21:42:29 #318 №184500 
Смотрю сейчас сериал "Космос: пространство и время" и ведущий только что сказал, ведя разговор об обилии жизненных форм, что с большинством наземных форм жизни (на Земле) нам еще только предстоит встретиться. По-моему, все континенты уже изучены вдоль и поперек, где эти формы еще могут водиться?
Аноним 01/06/15 Пнд 21:46:04 #319 №184501 
>>184500
В океанах, в грунте, в анусе твоей мамаши (сириозли)
Аноним 01/06/15 Пнд 21:46:53 #320 №184503 
>>184486
Терешкова больше похожа Сьюзи чем он сам
Аноним 01/06/15 Пнд 21:47:10 #321 №184504 
>>184500
Микрохуитки всякие, алсо подвиды (нет, терминологией не владею). Какие-нибудь дятлы, например, разными бывают, часто разные виды различить может только специалист.
Аноним 01/06/15 Пнд 22:03:38 #322 №184508 
>>184500
окиян по большей части терра инкогнита, или чтото изменилось пока я спал?
Аноним 01/06/15 Пнд 22:33:09 #323 №184517 
>>184500
Где-то читал, что в мировом океане за 10 лет найдено и описано около 18000 не известных до этого видов. Плюс насекомые, их реально до жопы еще неизвестных, во всяких джунглях и т.д. Плюс озерцо в Антарктиде вскрыли, в общем Земля еще непаханое поле.
Аноним 01/06/15 Пнд 23:09:33 #324 №184528 
>>184508
Да и на поверхности еще дохуя труднодоступных неисследованных мест. Вон всякие истории с разбившимися самолетами, которые случайно отыскивают спустя 60 лет, и добираются до них неделю.
Аноним 01/06/15 Пнд 23:23:03 #325 №184535 
А под поверхностью, несколько километров литосферы населены бактериями и вирусами, кое-кто считает что там биомассы больше чем на поверхности.
Аноним 02/06/15 Втр 00:14:25 #326 №184546 
>>184421
Где то читал, что согласно теории эволюции, даже на других планетах развитые виды будут иметь гуманоидное строение в большинстве случаев
Аноним 02/06/15 Втр 00:18:26 #327 №184548 
>>184546
Это бесконечный срач, будут они похожи на нас, или не будут. До усеру можно спорить и абсолютно безрезультатно. Недостаточно знаний, ксенобиология похожа на гадание на кофейной гуще..
Аноним 02/06/15 Втр 00:21:31 #328 №184549 
>>184548
Аналогично: вселенная замкнутая система или нет? Есть ли бог или нет? Мы - плоды эволюции или разума? Мы зародились здесь, или нас сюда привнесли? Был ли БВ? И т.п
Аноним 02/06/15 Втр 00:26:45 #329 №184550 
>>184549
Нет, гуманоидность инопланетной жизни хотя бы в принципе проверяемая гипотеза. Так что не одно и тоже.
Аноним 02/06/15 Втр 00:35:03 #330 №184553 
>>184550
Да ну? И как?
Аноним 02/06/15 Втр 00:44:56 #331 №184555 
>>184550
Дельфины нихуя не гуманоиды но почти разумны (а может и разумны но по своему)
Критерии разумности вообще подогнаны под хумана и довольно размыты
Аноним 02/06/15 Втр 00:50:56 #332 №184558 
>>184549
Согласно современным представлениям Вселенная замкнута.
Выглядит это примерно так: представьте себе бесконечную плоскость, на которой мы живём. Привычное трёхмерное пространство получаем свернув эту плоскость в сферу. Поверхность сферы есть бесконечная плоскость. Вселенная же - сфера в четырёх измерениях. Бесконечное трёхмерное пространство, свёрнутое в сферу. Не исключено, что и так далее.
Аноним 02/06/15 Втр 00:54:15 #333 №184559 
>>184558
Речь о системе - есть ли обмен с другими системами или нет. На вселенную похуй
Аноним 02/06/15 Втр 01:01:58 #334 №184561 
14331961188460.jpg
>>184555
Аноним 02/06/15 Втр 01:04:21 #335 №184562 
>>184555
Шимпанзе тоже попадают под критерий разумности, и они гуманоидны.
Но вопрос терминологии открыт, да.
Аноним 02/06/15 Втр 01:16:53 #336 №184566 
>>184562
Вроде как осьминоги ещё имеют шанс стать разумными.
Аноним 02/06/15 Втр 01:26:03 #337 №184570 
>>184566
Шанс, может и имеют, но до разумности им как до Китая раком еще эволюционировать и эволюционировать. То, что они умнее других беспозвоночных, разумными их не делает.
Аноним 02/06/15 Втр 01:31:44 #338 №184572 
Интересно как осьминоги буду сушу исследовать когда все людишки передохнут
Аноним 02/06/15 Втр 01:33:50 #339 №184573 
>>184555
Есть интеллект - способность решать конкретные задачи, адаптировать решение под условия, искать решения, обучаться. Интеллектом обладает много всяких штук: программы, простейшие животные, человек. Интеллект - штука "механическая", не живая, самообучающаяся, в живых существах выполняет роль контроллеров, управляет многими подсистемами. В человеке несколько интеллектов, специализирующиеся каждый на своей области задач.
Интеллект долго обучается, но быстро решает знакомые задачи.

Есть разум - способность мыслить абстрактными категориями. Штука тоже неживая, механическая, аналогичная интеллекту, но гораздо более сложная, и в отличии от него - универсальная, но крайне медленная, поэтому так вредно размышлять, когда нет времени. Может решать незнакомые задачи, может проводить анализ условий, и аналитически выводить верное решение, а может и ошибаться - разум штука не точная и не эффективная, эффективность и точность ограничены основой, на которой работает разум. Если интеллект - штука дешевая, и по габаритам, и по массе, и по сложности, и по энергии, то разум - полный антипод: крайне дорогая и сложная штука. в одно тело гроздь разумов природе засовывать не с руки, да и энергией обеспечить трудно. Разумом на планете обладают только живые существа, всего несколько видов. У человека активность разума сьедает более 90% всей энергии тела - такая вот йоба. Зато эта йоба позволяет выживать в любых условиях, так что природа сделала ставку на разум, не снабдив нас ни зубами, ни когтями, ни мощным телом, ни мехом - за ненадобностью, наше единственное оружие - разум, зато это не просто оружие, это сверхоружие, что-то типа атомной бомбы в век огнестрельного оружия.

Есть сознание - способность осознавать себя, отделять себя от мира, а иногда - и от тела, заниматься всякой философской фигней, ставить цели в жизни. В это умеет мало кто - человек и некоторые высшие животные. Сознание паразитирует над разумом и интеллектом - подчиняет их себе. С этой точки зрения сознание можно рассматривать как паразита. Но в тоже время сознание позволяет централизовать контроль и выработку решений - это главный пульт управления телом. Всякие моральные нормы, принципы, ценности и прочие ограничители и побудители лежат именно здесь. Ментальные вирусы также паразитируют именно здесь. Здесь же ютится наша личность, и почти все высшие функции, "подпрограммы".
Люди могут лишаться сознания в экстремальных условиях - в условиях плена, лагерей, и т.п. Это еще называется "сломаться", лишиться принципов. В таком случае контроль переходит к разуму - человек переходит в стадию предков, в стадию пещерного человека, животного, и "выживает" любыми способами, сознание отключено, нет никаких принципов и мотивов кроме инстинкта самосохранения, стремления выжить, человек ест падаль, пьет гнилую воду, и т.п., не обращая внимания на это. После такого возникает длительный отходняк - медленное восстановление сознания и личности. Память об этом периоде после старта личности оказывается заблокированной, и доступна урывками - это либо особенности конструкции либо самозащита личности от разрушения, т.к. эти воспоминания легко могут разрушить личность, уничтожить принципы, ценности и ограничения сознания, и т.п. Именно здесь рай для психологов и психотерапевтов - восстановить личность непросто, процедура рестарта личности дает много сбоев, так что люди с войны возвращаются в разных состояниях, от животного и до полноценной личности, с многочисленными переходными состояниями.
Поэтому в лагерях главное - не дать отключиться личности, не переставать быть человеком. В сложных ситуациях это повышает шансы выжить (например не дает безрассудно кинуться под пули, увидев дырку в заборе - хоть дырка и выход, но пули - гарантированная смерть. Животные про пули не задумаются, т.к. их не видят, и кидаются толпами под пули, бессмысленно умирая, как светлячки на лампе, пока не увидят смерть других своими глазами), и не дает разрушиться личности - человек копротивляется ради каких-то высших целей, осознанно ставит себе цели и задачи, проявляет гуманизм, блюдет честь, а не просто "выживает".
Аноним 02/06/15 Втр 01:41:20 #340 №184579 
>>184573
Так а с осьминогами как быть
Я вот раньше кушал осьминогов а теперь западло как-то
Вроде братишки как бы
Аноним 02/06/15 Втр 01:52:03 #341 №184590 
>>184555
>Дельфины
>почти разумны
Нет, они как собаки.
Аноним 02/06/15 Втр 01:57:41 #342 №184593 
>>184579
Ты к веганам запишись им и корова - братишка.
Аноним 02/06/15 Втр 01:58:27 #343 №184595 
>>184593
Не, коровы тупые
Аноним 02/06/15 Втр 02:05:02 #344 №184597 
>>184595
могла бы быть умной, у нее чемодан мозгов, но кому нужна умная корова
Аноним 02/06/15 Втр 02:28:24 #345 №184598 
>>184595
У моего бати была корова, которая утром сама открывала калитку, выходила на главную улицу дерёвни и присоединялась к колхозному стаду, а вечером точно так же приходила домой в стойло.
Аноним 02/06/15 Втр 06:34:24 #346 №184610 
>>184421
>У людей есть все необходимое, так почему бы не направить весь человеческий потенциал в нужное русло?
Потому что есть большая разница между мечтами мамкиного колониста и суровой реальностью. Флаговтыкание отошло в прошлое, а других безальтернативных целей высадка на Марс не имеет. Для полетов дальше нужен технологический прорыв, появление очень быстрой по нынешним меркам йобы. Причем большой, орбитального запуска вероятно. Поэтому нечто большее, чем несколько подобий МКС, летающих одновременно, человечеству пока не нужно.
>вероятность существования иной формы жизни во Вселенной тоже бесконечна?
Че несешь, гуманитарий? Вероятности бесконечными не бывают. Ну а так да, в концепции бесконечной Вселенной у одной Земли бесконечное число копий.
>Какова вероятность, что иные формы жизни во Вселенной тоже могут быть гуманоидными?
Это вопрос из области несуществующей науки. Чтобы делать подобные выводы, нужен сравнительный анализ, а сравниваться не с кем. Мы ведь даже достоверно не знаем историю зарождения жизни на Земле. Ну то есть знаем, но на любом из этапов могло иметь место вмешательство в эволюцию внеземного разума. Может таких вмешательств было много в разных концах галактики, поэтому гуманоид - вполне распространенная форма по образу и подобию "мастер рэйс". Но скорее всего жизнь на большинстве планет проходила собственную эволюцию. И процесс это очень долгий, с множеством случайных факторов. Потому логика подсказывает, что общего между жизнью на разных планетах будет маловато.
Аноним 02/06/15 Втр 06:44:15 #347 №184612 
>>182891
Есть ли какие-нибудь прикидки по поводу того, как на человека влияет не околонулевая, а пониженная гравитация? В условиях микрогравитации, насколько я понимаю, предел человеческого тела - в районе полутора-двух лет, даже при условии постоянных тренировок и изначально идеального состояния здоровья. А сколько может провести человек, скажем, при лунной гравитации, чтобы потом он мог вернуться на Землю?
Аноним 02/06/15 Втр 06:49:00 #348 №184613 
>>184598
Собака например обладает только интеллектом. Ее можно научить всяким фокусам, но это не будет разумом - она не сможет анализировать, только повторять заученное. Это и есть интеллект - решение знакомых проблем, с обучением, адаптацией под условия и потребности среды.

Простейший тест - собака не может понять, что она на поводке, не может распутать поводок, если он задевает за дерево - она будет упрямо рваться вперед, искренне не понимая в чем дело. Обладала бы она разумом - проанализировала ситуацию, и просто обошла дерево с другой стороны. Со временем таким образом она могла бы самообучиться обходить дерево, научив разумом свой интеллект, и ввела бы в свою картину мира концепцию поводка, чтобы анализировать сложные переплетения - когда деревьев несколько, или оборотов поводка вокруг дерева более одного. Но этого не происходит.
Ее можно научить обходить дерево, но это уже будет интеллект - лишь видимость разумного поведения. Ее обучаем мы, а не она сама. Сама она может допетрить лишь до такой концепции: если бежишь вперед, но что-то держит - значит бежать не надо. Именно поэтому поводок работает - она думает, что ее удерживает человек, поэтому ощутив сопротивление поводка сразу успокаивается, как будто хозяин рядом.

Собака не обладает сознанием.

Интеллект работает, разум существует, сознание живет.
Интеллект способен к обучению. Обучить интеллект может разум или сознание - потому что надо распознавать ситуации, анализировать решения, выделять критерии правильности и неправильности, интеллект на столь сложную деятельность не способен.
Разум - универсальная аналитическая машина, способен к сложной деятельности, но действует автоматически, не имеет принципов, ценностей, моральных ограничений как класс. Все это в разум должно вводить сознание, как условия задачи.
Разум по дефолту ищет способы удовлетворить потребности, и подчиняется инстинктам - это мощный инструмент выживания, способный решить любую задачу: спрятаться от врага, придумать как его убить, придумать как и где добыть пищу, воду и самку, как спрятаться от непогоды, и т.п. примитив. Короче разум автоматически спасает тело. Без необходимости вмешательства сознания. Это просто охуитительно, и вот почему: разум - это автопилот, по дефолту настроенный на ваше выживание, и, если с условиями все в порядке - охватывает нижние ступени пирамиды маслоу: самки, вкусная хавка, уютная пещерка, шкуры и украшения.
Т.е. если вы чувствуете, что ваше сознание ускользает - запрограммируйте разум, дайте ему какую-то задачу, и смело отключайтесь - если тело не отключится вместе с вами, разум исполнит задачу, выполнит программу. Например бухарики часто возвращаются домой на автопилоте, а некоторые даже не отличаются от трезвых - способны к простейшему общению,, не шатаются, хотя пьяные в дрызг, способны к самоконтролю при потухшем сознании, и т.п. чудеса акробатики - это высший пилотаж программирования разума, разум получил столь подробные инструкции, что в некотором роде имитирует сознание - не позволяет инстинктам взять верх, контролирует походку, приводит домой через весь город, генерирует простые вопросы и ответы ради достижения какой-либо цели.
Ну а сознание - куча мусора, там навалено много дерьмеца: всяческие не важные для выживания тела вещи, типа принципов, целей, девизов, ценностей, и т.п. хрени. Политота, религия, упрямство, консерватизм, психболячки и странности, и т.п. мусор.
Кто из вас хоть раз прибирался в своем сознании? А ведь это надо делать регулярно, иначе там заведутся всякие болезни.
Сознание надо держать в чистоте - это еще Будда просек в хрен знает каком году. А у вас там что? Всякие сникерсы, стиморолы и прочий бред.
Аноним 02/06/15 Втр 06:52:05 #349 №184615 
>>184612
Полгода вроде - и путь домой закрыт.
Аноним 02/06/15 Втр 06:58:39 #350 №184618 
>>184615
При 0G год, а при одной шестой G - полгода? Чё ты несешь?
Аноним 02/06/15 Втр 07:00:51 #351 №184619 
>>184618
Хз, вроде читал где-то.
В любом случае 0g от 1/6g не отличается - это недостаточно для нормального функционирования организма, так что срок будет сравнимый.
Аноним 02/06/15 Втр 08:13:57 #352 №184623 
>>184437
>В галактике Млечный путь, по разным оценкам, от 200000 до 450000 звезд
>от 200000 до 450000
Эфиру передолбил?
Аноним 02/06/15 Втр 08:33:13 #353 №184624 
>>184613
Ты это сам пишешь или копипастишь откуда-то? Написано хорошим языком, но написана какая-то импровизированная поебень. Вот насколько я профан в вопросах искусственного разума и прочих нейрологиях, но за версту чую в этом тексте маняфантазии. Что-то уровня Дейла Карнеги, который уверенно пишет в одном месте очевидные вещи, в другом дичайшую чушь, а домохозяйки хавают и почитают его за пророка. Выкладки научные где?
Аноним 02/06/15 Втр 08:37:41 #354 №184625 
1)Почему КА Союз и РН Союз называются одинаково? Фантазии не хватило?
2)Могут ли один и тот же грузовик или КН вывести разные РН?
Аноним 02/06/15 Втр 08:38:56 #355 №184626 
>>184625
>или КА
Аноним 02/06/15 Втр 09:38:33 #356 №184628 
>>184625
2. Само собой, те же прогрессы выводились и будут выводиться на Союзах и Протонах. Я так понимаю, вопрос подразумевался шире - может ли любая ПН выведена любой подходящей РН. Может, лишь бы подходило по габаритам, ЦТ, динамическим характеристикам типа перегрузок и вибраций и т.п.
Аноним 02/06/15 Втр 09:56:56 #357 №184630 
14332282160860.jpg
>>184612
>В условиях микрогравитации, насколько я понимаю, предел человеческого тела - в районе полутора-двух лет, даже при условии постоянных тренировок и изначально идеального состояния здоровья.
Соус? Таких длинных полетов не было ведь, Поляков и тот летал 14.5 месяцев, и вот сейчас еще Келли будет год летать. Причем не припомню каких-то долговременных последствий для Полякова, он собственно на своих двоих из спускаемого аппарата вышел.

Алсо, байка от Полякова:
> Проблему радиационной безопасности решил во время своего полета А.А. Серебров. Он — физик и хорошо понимает данную проблему, поэтому ушел из каюты станции, где обшивка всего 3,5 мм, в модуль "Кристалл". Там за приборными панелями космонавт разместил отработанные 100-кг блоки свинцово-кадмиевых аккумуляторов. В блоках остался электролит, а жидкость — прекрасная защита от радиации. Уровень радиации в "Кристалле" после принятых мер защиты снизился по меньшей мере вдвое, по сравнению с базовым блоком "Мира". А.А. Серебров рекомендовал и мне там находиться. Воспользовавшись его советом, за 14,5 месяцев полета я получил в два раза меньшую дозу радиации, чем мои коллеги за 6 месяцев полета.
Аноним 02/06/15 Втр 10:13:42 #358 №184632 
>>183522
А если так- у тел на Земле есть вес относительно планеты. Тогда выходит, что и у Земли есть вес относительно Солнца?
Аноним 02/06/15 Втр 10:49:42 #359 №184636 
>>184613
Ну вот тебе пример, когда собаки сами обучаются. В городах они научились осторожно переходить дорогу, видя в тачках опасность для жизни, хотя не всех из них сбивала машина и не все видели, как сбивают других. Более того, они научились переходить дорогу на зеленый сигнал светофора, в отличии от многих людей. Выходит у них есть и другие процессы осознания окружающего мира, кроме дрессировки человеком.
Аноним 02/06/15 Втр 11:30:05 #360 №184642 
>>184559
Какой системе?
Аноним 02/06/15 Втр 11:37:20 #361 №184643 
>>184630
> Воспользовавшись его советом, за 14,5 месяцев полета я получил в два раза меньшую дозу радиации
Он так и сидел больше года как сыч за стеной из аккумуляторов? А стена сферическяа была или он только от Солнца отгородился?
Аноним 02/06/15 Втр 11:38:45 #362 №184644 
>>184636
Во-первых, собака - животное социальное, она может повторять поведение других собак, людей. Во-вторых, это тоже своего рода дрессировка - идёшь на красный, получаешь бампером, идёшь на зелёный - не получаешь бампером. Стимуляция выработки условного рефлекса как есть. Ну или эволюция - выживают собаки, переходящие улицу на зелёный.
Аноним 02/06/15 Втр 11:42:02 #363 №184645 
>>184632
Нет. Можно провести параллель с космонавтом на орбите Земли.
Земля беспрерывно падает на Солнце, но из-за скорости вращения пролетает мимо. Её ускорение относительно равномерно и прямолинейно,а значит, она в невесомости.
Аноним 02/06/15 Втр 12:08:07 #364 №184649 
>>184644
Я же сказал-не все собаки, которые умеют переходить на зеленый, однажды были сбиты на красный. И вообще я это к тому, что не стоит приписывать существование сознания только лишь человеку. И вообще делать его эталоном. То, что мы наворотили кучу технических устройств, более-менее изучили окружающий нас мир и начали заморачиваться по всяким там философским вопросам, возможно и не делает нас настоящим разумным видом по меркам Вселенной. На уровне космических тел типа звезд-планет мы скорее больше похожи на паразитирующую колонию, обладающую определенным самосознанием и потенциально готовую распространиться на следующие объекты.
Аноним 02/06/15 Втр 12:10:59 #365 №184650 
>>184645
Но ведь если исключить постоянное падение космонавта на Землю, то и у него есть вес.
Аноним 02/06/15 Втр 12:13:03 #366 №184651 
>>184650
У него есть масса, веса почти нет
Аноним 02/06/15 Втр 13:07:18 #367 №184660 
>>183887
А что происходит на 7 и 8 картинках?
Аноним 02/06/15 Втр 13:15:32 #368 №184662 
>>184660
Слияние ядер галактик и образование одной большой эллиптической галактики.
http://www.youtube.com/watch?v=pIueBYE5oHE
Аноним 02/06/15 Втр 15:01:13 #369 №184684 
14332464734940.jpg
14332464734951.jpg
Смотрел фильм "Пекло" и им подобные. Там, кароч, везде, где тепловая защита нужна, везде всё как будто из золота сделано. Скафандры, тепловые щиты спейсшипа и т.д. Лунный модуль, вот, тоже золотую юбку имеет.
Это реально золото? Или просто жёлтый сплав, латунь, медь, бронза, томпак?.. Почему не алюминиевая/серебряная фольга или настоящие зеркала?
Аноним 02/06/15 Втр 15:09:16 #370 №184685 
>>184684
В Гугле забанили?
Каптон же. В космосе тепло передаётся излучением. Предполагаю, что цвет такой из-за того, что ик лучше отражается именно золотом.

Да, кстати, хуле у ЛМ микрометиоритный экран не завесили плёнкой, а оставили голым?
Аноним 02/06/15 Втр 15:09:23 #371 №184686 
14332469630680.gif
>>184684
>http://physics.stackexchange.com/questions/74412/why-does-nasa-use-gold-foil-on-equipment-and-gold-coated-visors
>So why use gold? It’s most likely the same reason why they use gold extensively in circuit boards. (i) Gold does not corrode or rust while silver and copper does, which would reduce reflectance (by the way this happens before takeoff) and (ii) it’s a lot easier to work with gold than aluminum.
Аноним 02/06/15 Втр 15:11:37 #372 №184687 
>>184686
Это нихуя не фольга, если что. Заебали уже.
И имхо таки стойкость к коррозии где-то на предпоследнем месте среди нужных свойтв стоит.
Аноним 02/06/15 Втр 15:20:23 #373 №184689 
>>184687
Ну не скажи, медь и алюминий на воздухе покрывается оксидной пленкой легко, если в руках держал. Серебро тоже не очень.
Если говорить о космическое шлеме: присмотрись к графику - у золота максимум поглощения наступает как раз к ультрафиолету, а оптику он "пропускает" лучше чем все остальное. Золотая фольга на просвет зеленая, так-то.
Аноним 02/06/15 Втр 15:21:36 #374 №184690 
>>184685
>>184686
>>184687
Какая оживлённая борда, оказывается! Спасибо, ребят. Каптон - это основа, понятно.
А жёлтый цвет напыления? Всё-таки золото? Почему?
По графику >>184686-джентельмена, получается, что серебро лучше отражает. Ну и дешевле же на порядок.
Аноним 02/06/15 Втр 15:52:31 #375 №184696 
>>184690
Погляди еще раз, золото берет диапазон куда шире.
Аноним 02/06/15 Втр 15:53:26 #376 №184698 
>>184696
Что-то я спизданул не то, там не отражение, а не поглощение.
Аноним 02/06/15 Втр 16:10:19 #377 №184699 
14332506194990.jpg
>>184690
Алюминизированный каптон, или майлар.
http://www.hq.nasa.gov/alsj/a11/a11LM5structures.pdf
Если с одной стороны желтоватого каптона нанести алюминий, или посеребрить, то с другой стороны он будет сиять, аки золото.
Аноним 02/06/15 Втр 16:26:45 #378 №184700 
>>184685
>Да, кстати, хуле у ЛМ микрометиоритный экран не завесили плёнкой, а оставили голым?
Двачую вопрос. Почему посадочная ступень в плёнке, а взлётная без?
У меня кароч есть теория, что стенки посадочной сделаны по иной технологии, и поэтому их нужно было заворачивать в изолятор. А сама капсула, из-за необходимости держать давление, имеет стенки более сложной конструкции (и более тяжелые, потому и не юзали на посадочной), которые, однако, и без плёнки годно изолируют.
Ну, кто что думает?
Беглый гуглинг выдаёт инфу только по стенкам взлётной ступени, про посадочную все забыли.
Аноним 02/06/15 Втр 16:34:24 #379 №184703 
Интересует такой вопрос, может ли жизнь выйти за пределы планеты? Я имею ввиду не человеков на ракетах и не микроорганизмы на метеоритах, а сложные формы жизни. Ведь в пределах Земли жизнь освоила все среды обитания, почему бы не появиться формам, способным к жизни в космосе. Это конечно не вопрос, знаю, что на него пока никто не ответит. Просто фантазия разыгралась.
Аноним 02/06/15 Втр 16:38:51 #380 №184704 
>>184700
>Почему посадочная ступень в плёнке, а взлётная без?
Пиздец, для кого выложил это? >>184699


Аноним 02/06/15 Втр 17:46:23 #381 №184728 
>>184704
Ты ебанулся?
Ну или поясни тупому.
Аноним 02/06/15 Втр 17:46:31 #382 №184730 
>>184703
Энергетика жизни близка к нулю. Чтобы сбежать из гравиколодца, нужно гораздо больше энергии, намного больше энергии.
Организму нужно будет как минимум столько энергии, чтобы разогнать каждый кило своей массы до 8км/с, притом он еще и опору должен найти.
Если разгоняться в атмосфере, где опора есть - нужны дикие давления, дикая прочность организма, чтобы набрать высокую скорость на низкой высоте - таких форм жизни на земле нету. Что-то похожее есть на дне океанов, но там организмы имеют прочность на сжатие, а нужна на растяжение.
Такую прочность на растяжение в результате эволюции организмы приобрести не могут - максимальный отрицательный перепад давлений в природе всего-лишь одна атмосфера.
Т.е. природа про космос не подозревает, а если бы и подозревала - не смогла бы затестить пробные организмы из-за отсутствия таких условий на земле.
Именно поэтому космос - для разума.Без разума покинуть гравиколодец нельзя. Такая вот идеальная тюрьма, фильтр разумных форм жизни.
Аноним 02/06/15 Втр 17:59:40 #383 №184735 
>>184704
Там кстати нихуя нет объяснения, почему в майларе только посадочная ступень. Отражать излучение факела? Так и у взлетной он есть, и хитшилд у нее тоже есть.
Аноним 02/06/15 Втр 18:01:14 #384 №184736 
14332572745980.jpg
>>184735
Алсо, я не понял откуда там на диаграмме у descent stage base heat shield температуры до 1150 по цельсию взялись.
Аноним 02/06/15 Втр 18:26:47 #385 №184744 
>>184736
Это термос - там топливо.
Аноним 02/06/15 Втр 19:03:51 #386 №184759 
>>184730
Дельный ответ, благодарю. В конце концов-разумная жизнь, тоже жизнь. Возможно это и есть найденный природой выход для распространения жизни.
Аноним 02/06/15 Втр 19:41:57 #387 №184778 
>>184759
Если рассматривать вселенную как симулятор, гравитация была бы программой-фильтром, препятствующей попыткам неразумной жизни расширить ареал обитания.
А сами планеты - шаблонами условий, каждая планета обладает разными параметрами, можно затестить жизнь во всех возможных условиях.
Аноним 02/06/15 Втр 19:42:51 #388 №184779 
>>184643
Mmmmmaximum piturd
Аноним 03/06/15 Срд 01:17:10 #389 №184889 
>>184744
А также во взлётной ступени и орбитере. Которые не завёрнуты.
Аноним 03/06/15 Срд 01:23:31 #390 №184890 
Аноны, поясните за одну хуйню
Допустим удалось создать лазер огромной мощности, который может работать очень долгое время в непрерывном режиме
Что если на пердак КА поставить зеркало из почти идеально отражающего материала и направить пучок лазера на него
По идее отраюающийся луч должен создать тягу для разгона аппарата
Где я наебался
Аноним 03/06/15 Срд 01:34:58 #391 №184892 
>>184378
первый вояджер после гравитационного маневра для сближения с титаном. нихуя не есть снимок планет называется врое семейное фото
Аноним 03/06/15 Срд 01:44:07 #392 №184893 
>>184890
Зеркало то тебе на фига? Если у тебя и так фотоны массой обладают
Аноним 03/06/15 Срд 01:45:21 #393 №184894 
14332851213610.jpg
14332851213631.jpg
14332851213672.jpg
14332851213673.jpg
>>184378
Очевидный Ulysses и его гравитационная праща вокруг Юпитера очевидны.

> Может, там что-то есть интересное?
Полюса нашей звезды, которые иначе не разглядишь.
Аноним 03/06/15 Срд 02:05:50 #394 №184900 
14332863509580.gif
>>184893
Так лазор сожжет жи аппарат
Аноним 03/06/15 Срд 02:26:12 #395 №184902 
>>184900
Я чет подумал что лазер у тебя в аппарате
Аноним 03/06/15 Срд 02:29:03 #396 №184904 
>>184890
Нигде. Я бы только не делал зеркало на жопе, нахуй не надо. На первое время можно сделать щит из абляционной защиты, при нагреве получится доп. тяга, при испарения слоев защиты. На достаточном расстоянии, когда пучок лазера будет уже рассеянным и мощность излучения упадет, можно развернуть солнечный парус и хуярить из лазера в него.
Аноним 03/06/15 Срд 02:36:47 #397 №184907 
>>184904
Так а какие подводные камни? А, понял. Тормозить нечем.
Аноним 03/06/15 Срд 02:37:51 #398 №184908 
>>184907
>Тормозить нечем.
Это да.
Аноним 03/06/15 Срд 08:03:46 #399 №184923 
>>184363
Бамп вопросу.
Аноним 03/06/15 Срд 08:46:26 #400 №184926 
>>184893
Вэйт э секонд, а по какому принципу тогда солнечный парус работает? Неужели исключительно на энергии тяжелых частиц?
Аноним 03/06/15 Срд 08:48:02 #401 №184927 
>>184363
>>184923
Я кароч начинающий авиамоделист. Ознакомился с числом Рейнольдса, вязкостью, турбулентностью, срыв потока, угол атаки и прочее хуё-моё. Скажу вам так, не углубляясь в детали: при изменении соотношения азот/кислород изменится плотность воздуха, значит изменится оптимальная высота полётов. Сейчас самое экономичное летать на 10-11 тыс метров. Станет воздух плотнее - эта высота поднимется, станет разреженнее - ниже будут летать.
Но! Если рассмотреть ситуацию под другим углом - изменится %кислорода в воздухе, который и сжигает авиационное топливо. И этот сдвиг баланса на несколько порядков важнее, чем изменение плотности воздуха! Станет меньше кислорода - однозначно летать будет "труднее" (возможно даже баки с окислителем придётся предусматривать на самолётах, соответственно уменьшая полезную нагрузку). Ежели кислорода станет больше, то турбинам и поршневым двигателям не нужны будут такие большие воздухозаборники, компрессоры и т.д. Но после достижения определённого процента О2, воздух может самовоспламеняться при контакте с горюче-смазочными материалами или у горячих поверхностей (коллекторов выхлопных газов), значит придётся предусматривать меры предосторожности. Герметизировать подшипники, тяги и другие механизмы, что также "затруднит" полёты (самолёты станут тяжелее от дополнительных узлов и дороже).
Аноним 03/06/15 Срд 09:27:30 #402 №184931 
>>184907
Тормозить хуем гравитационным маневром/эйрбрейкинг

"Мошка в зенице господней" Ларри Нивена и "Фиаско" Лема в помащ.

>>184904
видеворилейт
http://www.youtube.com/watch?v=5_9ac-w4DW8
Аноним 03/06/15 Срд 11:30:14 #403 №184943 
>>184926
>Неужели исключительно на энергии тяжелых частиц?
Не, там довление фотонов именно. Тяжелых частиц нинужно, нужны фотоны с импульсом побольше.
Аноним 03/06/15 Срд 12:29:09 #404 №184962 
Привет спейсач я ньюфаг
посоветуйте годные книженции для начала изучения. лучше побольше и на разный уровень, чтоб можно было при случае левелапнуться. Хочу понимать что вы здесь говорите.
Алсо что за штука Space Engine о которой вы столько трёте?
И где можно качественно отслеживать новости исследования космоса и астрономические события?

Сорян за зеленые вопросы, я пока тупой в вашей стезе
Аноним 03/06/15 Срд 12:36:52 #405 №184963 
>>184927
Спасибо, хорошо пояснил. Интересно просто, насколько могут атмосферные аппараты меняться в зависимости от атмосферы другой гипотетической планеты, близкой к Земляшке.
К слову, при увеличении доли кислорода можно делать крылья поменьше, верно?
Аноним 03/06/15 Срд 12:59:33 #406 №184968 
>>184962
>годные книженции для начала изучения.
начальный уровень:
Космос. Карл Саган
Мир в ореховой скорлупке Стивен Хокинг
Продвинутый уровень:
Общий курс астрономии, какой-нибудь учебник, например Мороза и Кононовича
>Алсо что за штука Space Engine о которой вы столько трёте?
Это космический симулятор Вселенной. Точнее интерактивный планетарий, можно летать от галактики к галактике, от звезды к звезде и от планеты к планете. Чисто эстетическое удовольствие. Программа бесплатная, можно скачать тут. http://spaceengine.org/ Тред висит на нулевой.
>качественно отслеживать новости исследования космоса
Сайты NASA, ESA, Новости Космонавтики, их вообще их довольно много, каждый сам себе составляет удобный список.


Аноним 03/06/15 Срд 14:32:29 #407 №185008 
>>184968
>планетарий
Он уже сделал реальные положения тел в солнечной системе? А то я как-то пытался поймать реальное солнечное затмение.
Аноним 03/06/15 Срд 14:38:09 #408 №185011 
14333314896090.jpg
>>184968
Спасибо, бро!
Хокинга пока не брал, читаю Вайнберга (первые три минуты), ощущаю себя ослом.
Кононовича почитаю. Сейчас парюсь с физической космологией Лукаша, так и не понимаю чем конкретно занимается космология.
Для того чтобы заценить небо и звезды искал карты подвижного неба и карты конкретно на май-июнь месяц этого года. Потом выхожу и смотрю в обсерватории или невооруженным взглядом. Короче действую по нубской интуциции.

Реквестирую любые рекомендации, а то я пока утопаю в море бессистемной информации.
Аноним 03/06/15 Срд 14:50:52 #409 №185014 
>>185011
Держи полезные для твоего нового хобби программы.
http://homes.relex.ru/~zalex/main1251.htm
http://www.stellarium.org/ru/
http://www.astronomy.ru/forum/index.php/topic,7926.msg3177686.html#msg3177686
Аноним 03/06/15 Срд 14:57:10 #410 №185018 
>>185008
Да похуй на солнечную систему, SE о фапании на красивые картинки и аутизме. Было же что-то другое, где именно солнечная система была сделана правильно.
Аноним 03/06/15 Срд 15:18:02 #411 №185023 
>>184963
Кароч, я тут погуглил, разница в плотностях чистого кислорода и чистого азота менее 2/10000 кг/м3 Так что это вообще можно не учитывать и крылья будут меньше на гулькин хуй. Химический состав решает.
Алсоу различия конструкции крыльев в зависимости от среды и скоростей полёта можешь и на нашей планете понаблюдать. Короткие крылышки крылатых ракет; треугольные короткие крылья сверхзвуковых истребителей; "нормальные крылья" пассажирских лайнеров; сверхдлинные крылья безмоторных планеров; подводные крылья "метеоров"; антикрылья гоночных болидов; крылья изменяемой геометрии у птиц.
Аноним 03/06/15 Срд 15:32:49 #412 №185026 
>>185018
>Было же что-то другое, где именно солнечная система была сделана правильно.
Celestia по-моему.
>Для того чтобы заценить небо и звезды искал карты подвижного неба
Стеллариум скачай обязательно. Тоже бесплатный, кстати.
Есть еще heavens-above.com, там можно следить за пролетами ярких ИСЗ, многих вштыривает гоняться за Иридиумами. Так же есть карты звездного неба и прочие.
Космология занимается проблемой эволюции и развития Вселенной. Это такая самая общая наука.
Аноним 03/06/15 Срд 15:33:25 #413 №185027 
>>185011
>>185026

Блядь, я рукожоп. Это сюда.
Аноним 03/06/15 Срд 15:55:42 #414 №185035 
>>185023
На титане человек мог бы летать только на своей мышечной силе.
Аноним 03/06/15 Срд 15:59:04 #415 №185037 
>>185014
Крутяк, спасибо
Аноним 03/06/15 Срд 16:28:37 #416 №185045 
>>185035
Причём тут, блядь, Титан?
Аноним 03/06/15 Срд 17:02:37 #417 №185049 
>>185045
К плотности воздуха
Аноним 03/06/15 Срд 17:09:49 #418 №185050 
>>185026
скачал стеллариум - зачетная штука. правда не понял почему арктур там белый, в телескоп он наблюдается как золотистый

ну не суть, карочи. спасибо
Аноним 03/06/15 Срд 17:11:39 #419 №185051 
>>185049
Там еще притяжение в 7 раз ниже, чем на Земле.
Аноним 03/06/15 Срд 17:12:19 #420 №185052 
>>185049
И что там с плотностью атмосферы? плавать так можно или крылья нужны ьудут?
Аноним 03/06/15 Срд 17:19:55 #421 №185055 
Ребят, а вода с Земли улетучивается? Или наборот прибывает из космоса?
А Земля набирает массу или наоборот?

в равновесие этих процессов не верю
Аноним 03/06/15 Срд 17:20:30 #422 №185057 
>>185050
>почему Арктур там белый
Там все звезды белые.
>И что там с плотностью атмосферы?
Гораздо плотнее, чем на Земле.

Аноним 03/06/15 Срд 17:30:17 #423 №185058 
>>185055
>Ребят, а вода с Земли улетучивается?
Распадается, в верхних слоях на водород и кислород, водород сдувает. 3 кг водорода в секунду уебывает с Земли. Процесс крайне медленный, большой запас газов у Земли под поверхностью и в жидком виде, в океанах, плюс лед.
>Земля набирает массу или наоборот
На Землю выпадает 2 тысячи тонн метеоритов в год. Плюс всякая пыль и то, что не долетает до Земли, сгорая в атмосфере.
Аноним 03/06/15 Срд 19:58:28 #424 №185076 
>>185058
Как что-то может уебать с Земли? Неужто водород набирает вторую космическую?
Аноним 03/06/15 Срд 20:05:56 #425 №185077 
>>185076
>Неужто водород набирает вторую космическую?
И гелий еще немного, но там совсем мало.
Аноним 03/06/15 Срд 20:45:47 #426 №185086 
>>185052
Крылья надо. В общем там рай для аэронавтики. Было бы здорово послать туда вместо ровера аэростат или вертолёт какой.
Аноним 03/06/15 Срд 22:30:52 #427 №185108 
>>185076
Ионизированный газ быстро летает. Скорость истечения ЭРД видал?
Аноним 03/06/15 Срд 22:39:40 #428 №185113 
>>185108
Плазма - да, за счет высокой температуры, ибо температура - мера движения: высокая температура - высокая скорость атомов.
Нагревать обьект - все равно что разгонять его.
Мама, я в космосе 2.0 Аноним 04/06/15 Чтв 00:25:11 #429 №185168 
Существуют ли какие-нибудь теоретические способы превысить скорость света?
Аноним 04/06/15 Чтв 00:28:07 #430 №185171 
>>185168
1. Фазово.
2. Как асимптоту.
Аноним 04/06/15 Чтв 00:29:27 #431 №185173 
>>185168
Конечно. Время замедляется да, но не обязательно из-за искривления пространства или достижения скорости света. Всегда есть еще варианты обьяснений, и заранее неизвестно, какой в итоге окажется верным, несмотря на все подтверждения или опровержения. А это значит - всегда есть вероятность обходных путей, даже для подтвержденных и общепринятых теорий, прямым текстом гласящих что что-то невозможно.
Опыт говорит: возможно все.
Аноним 04/06/15 Чтв 00:32:53 #432 №185176 
вдогонку
Мог ли Эйнштейн ошибаться касательно не превышения скорости света/теории относительности?
Аноним 04/06/15 Чтв 01:22:40 #433 №185207 
>>185176
Все могут ошибаться и Эйнштейн мог. Но его теория выдерживает проверки в течении уже сотни лет.
Перикл !3GqYIJ3Obs 04/06/15 Чтв 08:27:51 #434 №185268 
Анон, угорел по жидкостному дыханию. Если получится его реализовать — это ж можно будет гонять с огромным ускорением (если получится его развить). Собственно, вопрос: вот у нас корабль, заполненный жидкостью. Как осуществлять контроль в нём? Не повредит ли жидкость экранам, или придётся использовать нейрокомпьютерный интерфейс?
Аноним 04/06/15 Чтв 08:35:00 #435 №185269 
>>185268
Как же заебал этот трип!
Ещё вопрос: у Дэвида Брина, в "Прыжок в Солнце", корабли охлаждали с помощью лазеров. Возможно ли это ИРЛ? Брин всё-таки физик, в НАСА работает.
Аноним 04/06/15 Чтв 09:59:43 #436 №185280 
>>185269
Ну теоретически охлаждение ка в космосе суть вывод лишней энергии. Можно и лазерами наверное. Но для этого, естественно, нужны приемники солнечной энергии. Не думаю, что можно по хитрому соединить проводками обшивку с лазерами и лишнее тепло будет просто уходить на их питание. Скорее просто будет плавиться обшивка, а с ней и лазеры.
Аноним 04/06/15 Чтв 10:20:51 #437 №185284 
>>185269
http://en.wikipedia.org/wiki/Laser_cooling
Возможно, ИРЛ таким образом охлаждают отдельные атомы до около-нулевой температуры в физических экспериментах. Охлаждение макрообъектов таким образом сопряжено с кучей трудностей, считай что это псевдореалистичное НФ-допущение.
Аноним 04/06/15 Чтв 17:45:20 #438 №185375 
>>185168
И очень даже просто. Если непрерывно и быстро разгоняться, то по собственным часам можно долетесь до Альфа Центавры, допустим, за месяц (со сломанным перегрузками позвоночником). По земным часам, конечно, будет куда побольше, прямо даже радикально больше. Относительность.
Другое дело, что это невозможно по техническим причинам - на ускорение даже небольшой хуйни даже до 0.1с требуется непомерно большое количество энергии (плюс проблема встречного галактического ветра). Так что в любом случае легче построить корабль поколений, надышаться сероводородом и заснуть на пару тысяч лет.
Аноним 04/06/15 Чтв 17:47:35 #439 №185376 
14334292551520.jpg
>>185207
>сотни лет

Аноним 04/06/15 Чтв 18:24:20 #440 №185383 
>>185376
ТО была запилена Альбертушкой в 1905-1917 годах. Так что, да, сотня лет
Аноним 04/06/15 Чтв 19:29:02 #441 №185389 
>>185375
>невозможно
ноуп
>на ускорение даже небольшой хуйни даже до 0.1с требуется непомерно большое количество энергии (плюс проблема встречного галактического ветра).
Тут с посонами на переменке считали - хуйня выходит: встречный ветер слишком слабый, ускоряется быстро.
Например для массы 10000 тонн с ускорением 1g до скорости в 0.1c потребуется 6 лет. Мощность силовой 500МВт.
Суммарная мощность всех атомных электростанций планеты 380ГВт. Они работают с 1954 года - вот уже 64 года.
Т.е. человечество уже может поддерживать 500МВт в течении 6 лет, и даже намного больше.
Аноним 04/06/15 Чтв 19:48:06 #442 №185392 
>>185389
А разгон подразумевается каким образом - пердячим паром? Такие расчеты имеют смысл в случае разгона на антиматерии (где действительно импульс плюс тяга охуеть какие и массу топляка можно не учитывать), но даже разгон йоба-лазером крайне неэффективен (рассеяние большое и становится всё больше).
Аноним 04/06/15 Чтв 19:49:08 #443 №185394 
>>185389
И да, проблема встречного ветра не в торможении, а том, что он нахуй убъет всёх людей и электронику на борту.
Аноним 04/06/15 Чтв 19:58:03 #444 №185399 
>>185389
Бро, ты где-то объебался в расчётах.
Если мы можем обеспечить ускорение в 1g, то масса корабля нам не важна, так?
Тогда:
1g = 9.8 метров на секунду за секунду
V = a · t, t = V/a
t = 29 979 245.8/9.8 = 3 059 106.71 секунд = 35 суток.

>>185394
При наличии на борту мощного источника энергии, способного выдавать выперд для разгона десятитысячетонной поебени с ускорением в 1g не вижу проблемы в том, чтобы создать мощное электромагнитное поле которое будет отталкивать ионизированный межзвёздный газ.
Аноним 04/06/15 Чтв 19:58:33 #445 №185400 
>>185392
Какая разница? Представь что чистой энергией, с кпд 100%.
Если принять во внимание расход массы - либо растет тяга и ускорение, с сильным сокращением срока разгона, либо ускорение остается неизменным, а падает энергия.
Т.е. уточнение только уменьшает время или энергию, а не увеличивает.
Аноним 04/06/15 Чтв 20:12:12 #446 №185403 
>>185399
Тем более. 35 суток - слишком мало, значит ускорение можно снизить раз в 10-50, энергия также упадет.
Хотя 1g удобна для живых существ. Если есть энергия - можно весь полет поддерживать 1g, двигаясь прыжками.
Аноним 04/06/15 Чтв 20:36:16 #447 №185408 
14334393760410.jpg
>>185399
> t = 29 979 245.8/9.8 = 3 059 106.71 секунд = 35 суток.
t = 30 591 067.14 = 354 с утки

хошь верь, хошь проверь.
Аноним 04/06/15 Чтв 20:37:23 #448 №185409 
>>185408
Расчёты в студию. Я свои предоставил, укажи ошибку, если таковая есть
Аноним 04/06/15 Чтв 20:38:53 #449 №185410 
>>185408
Я нашёл у тебя ошибку. Ты считал, что мы разгоняемся не до 0.1c, а до 1c.
Аноним 04/06/15 Чтв 23:40:15 #450 №185456 
>>185268
Ну так что там с жидкостным дыханием? Не держать же пилотов в капсулах на время ускорения, управление может понадобится при разгоне.
Аноним 04/06/15 Чтв 23:50:46 #451 №185458 
>>185456
Нихуя.
Аноним 05/06/15 Птн 00:06:01 #452 №185465 
14334519610400.jpg
>>185456
Хуево: жидкость передает слишком много энергии, любой удар - и всем в жидкости пизда. Именно так глушат рыбу, например.

Жидкость не спасет от ускорений - она несжимаема.
За счет растворенных газов какая-то сжимаемость будет, но намного меньше, чем у газа.
Значит жидкость не может быть демпфером, или может, но оче хуевым демпфером.
В любом случае при ускорениях тело будет сломано, не снаружи, так внутри - разрывы внутренних органов и прочие бяки.
Что прикольно - в случае газов, тело будет сломано снаружи.

Можно что-то замутить с компромиссом - подобрать такую газожидкостную смесь, чтобы повреждения равномерно раскладывались на внешние и внутренние, это даст возможность наиболее эффективным образом гасить ускорения. Но только импульсные - постоянные гасить не получится.

Для больших ускорений, импульсных и постоянных, надо преобразовать тело в твердое - заморозить в блоке льда.
Но заморозка убивает клетки, и лед слишком хрупкий, так что это тоже не вариант.
Вариант - заменить всю жидкость в теле каким-нибудь прочным полимером, который мы можем делать твердым и жидким по желанию, например полимер с двумя неустойчивыми видами молекул, одна из которых твердая, одна - жидкая, которые можно легко дестабилизировать, и заставить поменять форму, которые либо поддаются удаленной дестабилизации, либо дестабилизация распространяется с цепной реакцией.
В таком случае заморозка вообще не понадобится - просто надо поместить тело в бассейн с полимером, и отдать команду на затвердевание полимера, полимер затвердеет, и тело вместе с окружающим полимером превратится в монолитный кусок прочного полимера, выдерживающий просто охренительные ускорения.
Пик - нечто подобное.
Аноним 05/06/15 Птн 00:15:45 #453 №185469 
>>185410
да, просто тред надо читать, а не сразу отвечать
Аноним 05/06/15 Птн 00:37:23 #454 №185473 
>>185399
бро, ты гдето в нулях запутался
я тоже хотел доебаться до этого нивазможна, но это действительно непростая инженерная задача. после попытки прикинуть сколько надо топлива для ЖРД для достижения 0.9с, когда у меня нот начал считать экспоненту, повесил мне калькулятор, начал греться, я подумал: не нафиг, не вариант совсем, от слова Совсем (совсем совсем вообще никак). на термояде пн топляк - 1 к 22000, еще туда сюда, на крайняк.
если у тебя кпд 100 процентов перехода энергия в кинетическую - то энергию массы 10000000 кГ обладающую скоростью 0.1с - блок мощностью в 1ГВт будет вырабатывать где то 140 тыщь лет, мы же этого не хотим да, поэтому будем строить чтото помощнее, правильно же). Блок в 100ПВт нам сойдет. (мы же не хотим тошнить с 0.1с)

>>185399
>При наличии на борту мощного источника энергии
забудь, не при наших текущих технологиях, даже в теории в обозримом будущем.[\s] ...хм подумал точка бифуркации тут нанофибра и сверхроводники - сумеют скрестить одно с другим, тогда еще туда сюда, но даже так движок не будет легким походу, мы же не хотим ускорятся как черепахи.

>но даже разгон йоба-лазером крайне неэффективен (рассеяние большое и становится всё больше).
почитал я это http://space.hobby.ru/projects/slr.html , вообщем я теперь уверен что вопрос будет решен когда надо.
Аноним 05/06/15 Птн 00:41:37 #455 №185475 
>>185465
ты заставляешь меня садить мышку в банку заливать ее водой и пихать в центрифугу
Аноним 05/06/15 Птн 01:12:44 #456 №185479 
>>185475
Зачем? И так все ясно:
Если жидкость газонасыщена - ее плотность низка, мышку размажет по стенкам центрифуги.
Если жидкость плотная - она останется на плаву, вблизи оси, но жидкость размажет по стенкам центрифуги, и возросшее давление в жидкости лопнет мышку.
Аноним 05/06/15 Птн 01:17:24 #457 №185481 
А нельзя компенсировать ускорение разгоном капсулы с космонавтом в противоположном направлении?
хуйню сморозил наверное
Аноним 05/06/15 Птн 01:22:59 #458 №185482 
>>185481
Можно. Так действуют демпферы - снижают ускорение за счет отрицательного ускорения.
Например рессоры на машинах.

Минус - ограниченный ход, ограничено количество поглощаемой энергии. Не подходит для постоянных ускорений, гасит только импульсы.
Аноним 05/06/15 Птн 01:36:33 #459 №185483 
>>185482
Вообще постоянные ускорения не гасит ничего, кроме антиграва.
Есть вариант с безынерционным генератором.

Из более реальных вариантов ни один не гасит постоянные ускорения.
Плотная жидкость превращает ускорение в давление, и равномерно его распределяет по телу и стенкам бассейна. Если стенки сделать подвижными, например надувными - это позволит растянуть демпфирование во времени, без разрушения организма.
Другой вариант - рессоры и газовые демпферы.
Магнитные демпферы.

В общем любой накопитель энергии может гасить импульсные ускорения.
Постоянные ускорения переполняют любой накопитель, поэтому гасить их не получится ничем, их можно либо не гасить - полимерный гроб, либо компенсировать - антиграв, либо устранить деструктивную причину ускорений - гаситель инерции, безынерционный генератор, то, что подавляет или сильно замедляет инерцию.
Последний вариант, безусловно, потребует какого-то источника энергии или сверхемкого аккумулятора, в зависимости от варианта исполнения. Источник энергии предпочтительнее: что-то зарядить - проще и безопаснее, чем разрядить. Но система с аккумулятором будет пассивной, т.е. может быть исполнена более надежно.
Аноним 05/06/15 Птн 02:10:51 #460 №185490 
Смотрю кинчик про Дарвин-4.
Похоже проблема перегрузок будет решена созданием псевдоИИ который и будет летать на дальняки. Это наверное проще чем ебаться с системами сохранения тушек в безопасности. Опять же жизнеобеспечение не нужно становится.
Аноним 05/06/15 Птн 02:35:55 #461 №185493 
>>185479
я вроде как однозначно упомянул жидкость
>заливать ее водой
с газом она или без - мне придется сильно постараться крутануть центрифугу, для того чтобы мышку размазало

понятно что можно менять плотность водного раствора - растворяя в нем разное. ну ок, мышка наглоталась воды, плотность раствора подобрали чтобы у мышки была нулевая плавучестьпосле спуска газов у нее. разгоняли центрифугу не быстро(те никаких взрывных перегрузок), но на 50g разогнали.
гипотеза эксперимента - выжмет, зажмет, сломает хребет, шерсть вылезет, заплачет кровавыми слезами, или видимых повреждений не будет?
Аноним 05/06/15 Птн 03:09:11 #462 №185500 
14334629519600.gif
>>185493
Не выживет, ибо ускорение постоянное.
Увеличивая ускорение мы как будто погружаем мышу все глубже и глубже в море-окиян.
Биоорганизмы хрупки, водолаз вроде до 200 метров может без компенсатора, 10 метров - атмосфера, итого не более 20 атмосфер. Дальше - пик.
Аноним 05/06/15 Птн 04:14:48 #463 №185512 
>>185500
кхм кхм как бы вообщем мне жалко это сообщать, но мышка уже того, в самом начале опыта, все во имя науки. цель опыта проверить как переживет скелет, мягкие ткани, жидкие ткани. проблема водолазов газовая смесь, ее биологическое действие, в тч на способность к мышлению, отравление организма растворенными компонентами этой смеси (не только, но..). способ помогающий переносить 5g к примеру, на протяжении длительного времени это уже достаточно круто было бы.
Аноним 05/06/15 Птн 06:18:38 #464 №185522 
>>184535
Стало страшно от твоего поста.
Аноним 05/06/15 Птн 06:23:07 #465 №185523 
>>185522
Ну да. Под поверхностью разветвленная сеть пещер и рек. Кто знает, что за монстры там живут?
Аноним 05/06/15 Птн 06:34:46 #466 №185524 
>>185523
Обычно, когда находят подземные водоемы, удручающе констатируют, что жизнь там весьма примитивна и бедна, если вообще есть.
Аноним 05/06/15 Птн 07:10:51 #467 №185526 
>>185524
Я не говорил о водоемах.
Я слышал множество историй, как неосторожных людей уносили подземные реки, выходящие на поверхность в виде колодцев и протоков.
Наверняка туда попадает гораздо больше животных и насекомых.
Там не только вода, там есть своды, залы и пещеры.
По-любому там должна была возникнуть экосистема. Если не сама, так привнесенная с поверхности.
Как люди выживают на островах, так они имеют шанс выжить в пещерах - наверняка там есть биомасса, пригодная в пищу, всякие грибочки, насекомые, а то и что посложнее.
Аноним 05/06/15 Птн 07:14:00 #468 №185527 
>>185526
Там специфические условия, не пригодные, для выживания существ с поверхности. Местная фауна там убогая. Если примитивный мир бедный и убогий, то не на чем базироваться, более сложным и крупным организмам. Фольклор - это не свидетельство чего-либо.
Аноним 05/06/15 Птн 08:10:43 #469 №185537 
>>185527
>не пригодные, для выживания существ с поверхности.
это как раз помогает тем кто там уже есть )
Аноним 05/06/15 Птн 08:49:27 #470 №185544 
>>185482
>>185483
Отсюда следует элементарное решение - двигатель не постоянного ускорения, а импульсного.
Аноним 05/06/15 Птн 20:31:57 #471 №185632 
>>185544
На ядерных боньбах
Аноним 05/06/15 Птн 21:03:08 #472 №185642 
>>185544
Хм. А это идея: энергию перевести в форму колебаний, средняя энергия которых равна нулю.
Демпферам, для избежания тряски, придется усиленно сбрасывать энергию.
Демпферы будут рассеивать энергию в виде тепла и электричества/механической работы, по желанию.
Тепло пускать на обогрев корабля, на подогрев топлива. Электричество - на ускорение плазмы. Механическую работы - на всякие механические нужды: насосы, давления, и т.п.

Ох лол. А ведь можно на ступень глубже: можно запасать энергию демпферов в гироакке, это даст возможность работать движкам гораздо дольше. Этакий эрзац-инерционный компенсатор.
Далее каким-то образом разряжать гироакк - генераторами, трением, и т.п., главное сбросить энергию, иначе он переполнится. Очень удобно сбрасывать энергию с выхлопом.
Аноним 05/06/15 Птн 21:05:21 #473 №185643 
Огурцы, есть ли мод на другие звездные системы?
Аноним 05/06/15 Птн 21:10:14 #474 №185645 
>>185643
Да
Аноним 05/06/15 Птн 21:23:16 #475 №185654 
14335285969470.jpg
Где можно посмотреть обновлённую карту вселенной? Эта аж с 2003.
Аноним 05/06/15 Птн 21:27:38 #476 №185657 
>>185654
Тебе зачем? В отпуск куда-то собираешься, боишься, то место еще на карту не нанесли?
Аноним 05/06/15 Птн 21:28:10 #477 №185658 
>>185654
В окно выгляни.
Аноним 05/06/15 Птн 21:42:51 #478 №185661 
>>185383
Сотни-это как минимум две. Фикситься надо, а не оправдываться.
Аноним 05/06/15 Птн 21:54:38 #479 №185662 
>>185661
Ты чурка что ли? В течении сотни лет, а не сотен лет.
Аноним 06/06/15 Суб 00:05:47 #480 №185721 
>>185662
В течение сотни лет тогда уж. В течении - это в речке.
Аноним 06/06/15 Суб 00:19:45 #481 №185724 
>>185721
Граммар-нацист дохуй? Тогда почему такой тупой, что не исправил ошибку автоматом, из контекста?
Или мозги замечать ошибки есть, а исправлять на лету - нет?
Образованные люди вообще слова не читают, воспринимая текст целиком - ошибки либо не замечаются вовсе, либо правятся еще до того, как попадут в сознание. Прямо как в этом упражнении:
>По рзеузльаттам илссоевадний одонго анлигсйокго унвиертисета, не иеемт занчнеия, в каокм проякде рсапжоолены бкувы в солве. Галовне, чотбы преавя и пслонедяя бквуы блыи на мсете. осатьлыне бкувы мгоут селдовтаь в плоонм бсепордяке, все-рвано ткест чтаитсея без побрелм. Пичрионй эгото ялвятеся то, что мы не чиаетм кдаужю бкуву по отдльенотси, а все солво цлиеком.
Аноним 06/06/15 Суб 00:24:50 #482 №185726 
>>185724
>Граммар-нацист дохуй?
Он самый.
Аноним 06/06/15 Суб 00:27:38 #483 №185728 
>>185724
Британские ученые такие британские.
С трудом прочитал первые слова, через несколько минут расшифровки. Этот текст не читаем в принципе, ты врешь про
>слова не читают, воспринимая текст целиком
Для этого мозг должен будет совершить тысячи перестановок в секунду, это невозможно - даже компьютер тратит на расшифровку почти секунду, а это около 4 миллиардов операций.
Аноним 06/06/15 Суб 00:33:59 #484 №185734 
>>185728
Хм, я легко прочитал. Тащемта это не пиздёжь, но я совершенно не помню принцип действия.
Аноним 06/06/15 Суб 00:37:30 #485 №185737 
>>185728
Либо толстый, либо совсем дебил.

мимо
Аноним 06/06/15 Суб 01:35:48 #486 №185753 
>>185465
>она несжимаема.
Это хуита для пабликов вкудахте.
>но оче хуевым демпфером.
Скажи это пулям из автоматических винтовок. Жидкость настолько "хуевый" демпфер, что пришлось разрабатывать специальное оружие, для пловцов.
>разрывы внутренних органов и прочие бяки.
Только в том случае, если само тело не поместят в жидкость.
>Для больших ускорений, импульсных и постоянных, надо преобразовать тело в твердое - заморозить в блоке льда.
С этим есть проблема, если только все тело морозить, и очень быстро, при чем в этом случае тело нужно обезвоживать и замещать воду чем-нибудь более подходящим.
>в монолитный кусок прочного полимера
В таком случае, это ничем не отличается от мумификации. То есть мы будем иметь очень хорошо сохранившийся труп.
Аноним 06/06/15 Суб 01:39:29 #487 №185754 
>>185721
Миль пардон, по русишу был трояк 20 ле назад очень стыдно. Мир?
Автор того поста, про сотню лет
Аноним 06/06/15 Суб 02:00:14 #488 №185761 
>>185728
Читается слёту без задержек (слегка тормознул на слове "плоонм", но и то длилось меньше полусекунды).
Мимо
Аноним 06/06/15 Суб 02:06:33 #489 №185764 
>>182891
Взяты ли уже следующие ачивки: самовыпил в космосе, ебля в космосе, ребенок в космосе? Думаю, отличный шанс для нашей страны восстановить утраченные позиции
Аноним 06/06/15 Суб 02:23:26 #490 №185770 
14335466068880.png
>>185645
Блять, только допер что ошибся тредом.
Аноним 06/06/15 Суб 02:26:51 #491 №185772 
14335468119970.jpg
>>185770
А ты небыстр.
Аноним 06/06/15 Суб 02:30:19 #492 №185773 
>>185764
Полгода в одной банке с бабами и без ебли? Конечно взята.
Самовыпилов вроде не было. Ребенка - вроде запрещают, но возможно изучали развитие зародышей всяких там крыс.
Аноним 06/06/15 Суб 02:37:38 #493 №185777 
>>185770
Ты не первый огурец, который проебывает тормозной импульс и приземляется не в Казахстане, а в Китае.
Аноним 06/06/15 Суб 02:40:07 #494 №185778 
>>185773
Год без баб и без ебли тоже конфирмед.
Поляков ехидно улыбается из своего угла, заставленного свинцовыми аккумуляторами.
Аноним 06/06/15 Суб 02:52:14 #495 №185781 
>>185728
Аутист, ты? Сразу прочитал всё.
Аноним 06/06/15 Суб 02:59:01 #496 №185784 
>>185778
Конфирмед то конфирмед - но таких зодротов единицы. На ISS за все время ее существования побывало куча нормальных парней и телочек, так что перепихон по любому был, и много.
В конце концов какое дело руководству, что там происходит? Связи большую часть времени нет, обстановка приватная, никто не афиширует что там творится при выключенных камерах. Перепихон ничего плохого не несет, главное чтоб предохранялись. Это что-то вроде сна или физкультуры - полезно и приятно.

Сам представь себя на их месте - полгода работаешь вплотную с бабенкой, буквально третесь друг об друга, день за днем. Такое мало кто выдержит. Остается либо забиться в угол, либо... все что происходит на ISS, остается на ISS.
В конце концов запрета на еблю нет, да и никто такого не смог бы запретить, это личное дело двоих, остальным знать не обязательно.
Аноним 06/06/15 Суб 03:16:30 #497 №185786 
>>185784
>так что перепихон по любому был, и много.
Тут важно свечку держать. Иначе как-то это все хило выглядит.
>Связи большую часть времени нет, обстановка приватная
По всей МКС установлены камеры, а связь давно почти круглосуточная.
>полгода работаешь вплотную с бабенкой
У них там такая программа ушатывающая, что не до ёбли.
>В конце концов запрета на еблю нет
Ебля в гондоне науке мало интересна.
Аноним 06/06/15 Суб 08:54:02 #498 №185815 
>>185728
Лол, дебил.
Аноним 06/06/15 Суб 08:59:21 #499 №185817 
>>185784
Не выдумывай
http://novayagazeta-ug.ru/note/u2690/2015/06/05/120721
http://42.tut.by/339200
Аноним 06/06/15 Суб 11:19:35 #500 №185824 
Сяпки, я не особо этим интересуюсь, но возник вопрос:
Есть планета земля, она крутится вокруг своей оси с нехуйовой такой скоростью. И тут на нее падает метеорит. При чем падает под углом 90 градусов к поверхности. Но попав в атмосферу, которая движется вместе с планетой с нехуйовой такой скоростью, смещается этой самой атмосферой в сторону вращения планеты.
Теперь мы имеем 2 наблюдателя:
1 находится статично закрепленный на поверхности земли, для него метеорит падает против вращения земли, поскольку смещенный атмосферой мтеор сместился не настолько сильно, как сместилась за дто время сама планета и соответственно точка его обзора.
2) находится статично закреплен в космосе и для него метеорит падает по направлению вращения планеты.
Соответственно вопросы:
1)Я прав?
2)Выходя из всёго вышесказанного, почему у тебя нету девушки анон?
3)Осуществлять посадку на планету лучше занаправлением её вращения или против?
Аноним 06/06/15 Суб 12:40:10 #501 №185840 
>>185824
Метеориты никогда не падают отвесно вниз (на земле).
Садится лучше по направлению вращения планеты. Взлетать тоже
Аноним 06/06/15 Суб 13:50:35 #502 №185856 
14335878357220.jpg
>>185824
>статично закреплен в космосе
Аноним 06/06/15 Суб 14:30:20 #503 №185860 
Если, допустим, придумают способ йоба-быстрого передвижения по космосу и отправят зонд в какую-нибудь Центавру, сможет ли он передать какие-нибудь фотографии? Или с такого расстояния это невозможно?
Аноним 06/06/15 Суб 15:02:34 #504 №185876 
>>185860
Надо еба-мощный передатчик, или еба быстро метнутся обратно. Еще можно растянуть еба-фильтр чтобы модулировать свет тамошней звезды достаточно чтобы задетектить изменение яркости. Первое и третье имеет смысл если недалекая система
sageАноним 06/06/15 Суб 20:48:27 #505 №185966 
>>185860
А ты чем собираешься передавать информацию, мамкин йоба-изобретатель? Электро-магнитной волной? Тогда распишитесь - 300 000 км/с и не грамма больше.
Аноним 06/06/15 Суб 21:19:25 #506 №185984 
перечислите отличия современного союз-тма м от гагаринского восток-1
Аноним 06/06/15 Суб 22:41:17 #507 №186009 
>>185988
Т.е. перефразируя анона: у анона нет девушки, потому что он считает, что не может ее осчастливить.
Но девушка может осчастливить анона, а счастливый анон непременно осчастливит девушку. Так в чем проблема?
sageАноним 06/06/15 Суб 22:54:26 #508 №186013 
14336204662490.jpg
>>186006
Аноним 06/06/15 Суб 23:17:41 #509 №186017 
>>186006
>времени не существует - там оно равно нулю
копать копать и это просвященный 21 века, ты определись не существует или существует и равно нулю

>радиоволны
это вид материи существующий в пространстве с не вырожденной временной координатой, такчто сади не сади - в гипер не але
>Важен именно конец
относительно важно чому ты не задвигаешь такое в sf ну и также в чем принцпиальное отличие от таких же идей сайфаевских авторов, твой гипер уже ктото эксплуатировал
Аноним 06/06/15 Суб 23:23:14 #510 №186019 
>>185817
>43.tut.by/339200
>Секс в космосе может быть смертельно опасен
>В частности, слишком слабая или слишком сильная гравитация влияет на развитие и функцию пыльцевой трубки - вегетативной клетки мужского заростка у цветковых растений.
ну что это за бред, вегетативный отросток, у тех космонавтов кому это положено, уже давно вырос.
>предложение за 1 миллион долларов помочь в съемке "космического порно"
мало предложили

другую линку пусть ктонить другой проверяет
sageАноним 06/06/15 Суб 23:48:11 #511 №186025 
>>185984
Союз-ТМА - это космический корабль. Есть Союз-ФГ. Ты про него? Поменялось дохуя всего. Подробности легко гуглятся, на память на скажу. Из очевидного - появилась третья ступень.
sageАноним 06/06/15 Суб 23:48:48 #512 №186026 
>>186009
> Так в чем проблема?
ТНН
Аноним 07/06/15 Вск 00:33:39 #513 №186042 
>>186026
Двачую этого адеквата
Аноним 07/06/15 Вск 15:05:00 #514 №186172 
>>186025
Я про космические корабли.
Аноним 08/06/15 Пнд 04:43:36 #515 №186310 
>>183129
Моя тян учится в МАИ, еще с детства увлеклась астрономией. Мне просто интересно почитать про это, кстати, она меня и подсадила на это дело. А еще у нее есть пиздатый телескоп и дом в деревне. Я когда первый раз посмотрел в него-получил настолько офигенное эстетическое наслаждение, что даже прослезился.
Аноним 08/06/15 Пнд 09:04:24 #516 №186323 
>>186310
На что смотрел? Луну?
Аноним 08/06/15 Пнд 10:29:49 #517 №186339 
>>183129
Увлек тян космосом, теперь она хочет чтобы я копил на телескоп. Ну там по ночам ходим, созвездия смотрим еще, и документалки про космос.
Аноним 08/06/15 Пнд 11:47:52 #518 №186349 
>>186339
Богоугодное дело. Тян вообще, без наполненной чем-то души дряхлеют и умирают, им необходимо что-то возвышенное и недостижимое, оторванное от реальности, чтобы заполнить пустоту внутри, чтобы жить. Какая-нибудь прекрасная но несбыточная мечта. Что может быть прекраснее, сильнее и несбыточнее космоса?
Космос удержит их надежно и до конца жизни, ведь он бесконечен, он велИк, он невероятно красив. И ей 100% не светит когда-либо достаточно в нем освоится, чтобы потерять к нему интерес.
Это во сто крат лучше, чем если бы пустоту в душе тян десятилетиями наполняли эти тупые и грязные реалити-шоу и мыльные оперы на 100500 серий, неизбежно превращая тян в вечно недовольную ворчливую змею, в еще одну глупую и злобную девушку/женщину/бабушку, коих уже более чем достаточно на этом шарике. В конце концов это не идет на пользу ни тян, ни ее куну.
Космос - наше все. Во всех смыслах.
Аноним 08/06/15 Пнд 17:11:19 #519 №186379 
https://youtu.be/lY-DShP1H2Q
Щто это такое, посаны? Автор уверяет, что летели они ниже облаков, а "искусственную комету" пускают выше.
Аноним 08/06/15 Пнд 19:12:52 #520 №186395 
>>186349
Нихуя ты романтик. Чет в голосину с тебя.
Аноним 08/06/15 Пнд 19:16:21 #521 №186398 
>>186379
http://sougift.ru/index/sky-lantern/sky-lantern-3pcs-300-rubley.html
http://sarcasm.mypage.ru/kupit_nebesnie_fonariki_optom.html
Шары со свечками. Стоят копейки, пускают массово на всяких мероприятиях, летят вверх, подчиняются ветру, мерцают.
На некоторых мероприятиях - квадрокоптеры с подсветкой. Но те более экзотически выглядят и ведут себя по резче, и не так много.
Аноним 08/06/15 Пнд 21:12:26 #522 №186414 
что было до большого взрыва?
Аноним 08/06/15 Пнд 21:35:33 #523 №186417 
>>186414
большая взрывчатка
Аноним 08/06/15 Пнд 22:00:56 #524 №186425 
Нашел этот тред, потому спрошу
Высаживались ли спутники на других звёздах?
Гугл что-то не гуглит
Аноним 08/06/15 Пнд 22:11:04 #525 №186428 
>>186425
Может и высаживались, например, нам-то откуда знать?.
Аноним 08/06/15 Пнд 22:28:15 #526 №186430 
>>186425
На поверхность других звезд? Наверняка. - до них же рукой подать. На поверхность юпитера или сатурна - точно.
Аноним 08/06/15 Пнд 22:39:41 #527 №186434 
14337923814120.jpg
>>186425
Только спутники Шепарда.
Аноним 08/06/15 Пнд 23:16:03 #528 №186446 
Вопрос №1
Чому так мало инфы (мимолетное упоминание пару раз из всех документалок о космосе которые я видел 1955-2015) о таком явлении как остывшая звезда. В любом случае белый карлик (да и нейтронная) когда либо остынет и что с ним будет далее?
Аноним 08/06/15 Пнд 23:18:18 #529 №186447 
Вопрос №2
вот хорошее описание того что только разумная жизнь может в космос.
>>184730

Продолжая развивать мысль а что если глубокий космос сможет покорить только ИИ?
Где почитать теорий или фантазий по этой теме?
Аноним 09/06/15 Втр 03:10:58 #530 №186451 
>>186446
>В любом случае белый карлик когда либо остынет и что с ним будет далее?
Углерод в нем выкристаллизуется и будет большой алмаз. BPM 37093, PSR J2222-0137. Что с ними становится не очень ясно, потому что ни одного холодного белого карлика не нашли. Что косвенно доказывает конечный возраст Вселенной, кстати. Не ушпели они остыть.
А так что с ними может быть? Кусок плотной материи, в виде небольшого шарика будет болтаться во Вселенной, пока та существует.
Аноним 09/06/15 Втр 05:30:09 #531 №186465 
Звезда схлопнулась в дыру. Какого радиуса будет горизонт дыры? Что будет с её (очеблизким, например) соседом по двойной системе?
Аноним 09/06/15 Втр 08:22:40 #532 №186474 
>>186465
Радиуса Шварцшильда, конечно же!
> Что будет с её соседом
Его тонким слоем намажут на аккреционный диск.
Аноним 09/06/15 Втр 09:11:47 #533 №186476 
>>186417
С большим детонатором
Аноним 09/06/15 Втр 09:13:54 #534 №186477 
14338304349130.jpg
>>186428
>>186430
Аноним 09/06/15 Втр 12:18:30 #535 №186493 
>>186447
>Продолжая развивать мысль а что если глубокий космос сможет покорить только ИИ?
не волнуйся не только ии, куски мяса тоже вполне годятся для этого, если ты конечно не подразумеваешь под покорением анальные совокупления с нейтронными и черными дырами
>Где почитать теорий или фантазий по этой теме?
попробуй реквистач в sf, но супер годного на эту тему не особенно вообще есть, авторы не вытягивают две темы.
Аноним 09/06/15 Втр 13:41:41 #536 №186502 
>>186493
Если представить что куску мяса нужно овердохуя чего таскать за собой (жрат, пить, дышат) то есть еще такая штука как время.
Т.е. если не изобретут некие гиперпорталы (пока что это все очень теоретически и не понятно можно ли такое впринципе будет запилить на практике) то путешествовать десятки тысяч лет для куска мяса это неосуществимо.
Так что ящитаю что следующий шаг эволюции человечества это перекат в ИИ который и будет далее покорять пространство.
Аноним 09/06/15 Втр 15:11:43 #537 №186513 
>>186502
Хотет перекатиться в комп и фапчевать капчу 24/7/365
Аноним 09/06/15 Втр 19:40:29 #538 №186552 
>>186502
твое мнение очень важно для человечества, и пока оно думает над твоим вопросом, для развития практических навыков, я советую тебе подумать и сформулировать, что же ты подразумеваешь под покорением пространства. в качестве факультативного упраженения и для раскрытия темы, написать в 2-3 пункта ответ на вопрос : какую практическую выгоду для человеков ии, будет иметь это покорение пространства.
Аноним 09/06/15 Втр 19:44:24 #539 №186554 
>>186552
>покорение пространства
перекат на другие планетки луны вне солнечной системы

>практическая выгода для человека
например не вечное Солнце
достаточный аргумент?
Аноним 09/06/15 Втр 23:22:15 #540 №186611 
>>182891
Извините, если вопрос слишком глупый, я просто не очень в этом разбираюсь. Суть такова: существуют ли реактивные двигатели с твёрдым горючим и жидким окислителем? Просто мне в голову недавно пришла такая идея: топливо-брикет из прессованного угольного порошка(м.б. в смеси с алюминиевой или магнивой пудрой), окислитель-жидкий кислород. Преимущества-легко выключать и управлять силой тяги(наверное), регулируя приток окислителя; полная безопасность до заправки кислородом, легко хранить. Ну как вам?
Аноним 09/06/15 Втр 23:33:11 #541 №186614 
>>186611
Ты изобрел ГРД, поздравляю.
http://www.kerc.msk.ru/ipg/development/hre.shtml
Аноним 09/06/15 Втр 23:38:01 #542 №186615 
>>186614
А их где-нибудь сейчас используют?
Аноним 09/06/15 Втр 23:43:38 #543 №186617 
>>186615
RocketMotorTwo у Virgin Galactic. Какой-то полимер+N2O.
Аноним 09/06/15 Втр 23:59:47 #544 №186621 
>>186617
Там смола сосновая вроде. Экоракета.
Аноним 10/06/15 Срд 00:04:34 #545 №186622 
>>186621
>Там смола сосновая вроде.
Но-но-но, товарищ, вы как-то очень странно называете полиамид.
Аноним 10/06/15 Срд 00:08:47 #546 №186623 
>>186622
Чего они в итоге то выбрали?
>>174224
Аноним 10/06/15 Срд 00:13:05 #547 №186624 
>>186554
Нет, не достаточный.

Для человека пока технологий нет. Для автоматов - нет задач.

Возможные варианты:
-добыча ресурсов, производство. Минус: дорого, сложно, пока не особо нужно.
-бекап человечества на другие сервера. Все тоже - дорого, сложно, пока не жмет.
-исследования внутренней части системы. Без экономической отдачи - значит идет только на пожертвования. Уже в вялотекущем процессе. Создание универсального исследователя, да на пожертвования - нереально, хотя возможно даже сейчас.
-исследования дальнего космоса. Бессмысленно - раньше подойдут технологии, чем кастрюля с антенной туда вообще доберется.

Короче улыбаемся и машем - мы сейчас в глубокой жопе яме: космическая гонка давно кончилась, практическое освоение маячит где-то на горизонте, нам же остается только ждать, надеяться и верить (что новые технологии подойдут раньше чем армии муслимов, ага), да обслуживать ближние орбиты - всякое потреблядство, мультимедия, кинцо, телефоны, жыпыэс и прочая бытовуха, обеспечиваемая спутниковой сетью.
Аноним 10/06/15 Срд 00:17:10 #548 №186625 
>>186623
Да что-то с прошлого бадабума я ничего внятного так и не слышал.
Аноним 10/06/15 Срд 02:18:36 #549 №186644 
>>186624
Ты немного не понял о чем я. говорю не о перспективе 50-100 лет а чуть далее. И о том что мясо с планеты не улетит далеко потому что мясо.
А вот ИИ сравнительно срать на время пролета в 10к лет.
Так может это (развитие ИИ и он уже в дипспейс) и есть путь Цивилизации?
Аноним 10/06/15 Срд 03:14:53 #550 №186652 
>>186644
ИИ - это мозги. Мозги не могут никуда убежать без тела.
ПсевдоИИ мы можем запилить уже сейчас, с полной гарантией отказоустойчивости, с самостоятельным принятием решений, с оптимальной стратегией энергорасхода (даже буран в нее мог), и прочими плюшками.
Отказоустойчивые системы сейчас юзаются на серваках, но их уровень очень низок от того, что мы можем запилить - просто в чем-то более надежном пока нет необходимости. А так - без проблем: кластер железа, гипервизор, обычная ОС, куча софта под каждую задачу, отобранная эволюционным путем (повышает надежность софта на 3-4 порядка и более, если нужно, но на Земле, возможно - параллельно с полетом), и специальный диспетчер задач, еще на 1-2 порядка повышающий надежность софта в полете (исключающий ошибки говнокодеров). Специальная процедура авторемонта железа мозга - позволит довести время его жизни почти до бесконечности: сделает его нечувствительным к любым воздействиям, кроме прямого механического разрушения, например мимоастероидом.

Но все это - лишь надежный вечный мозг. К нему еще нужно столь же вечное тело и источник энергии, и реактивная масса.

Реактивную массу можно научить его искать, добывать, и очищать - ему все-равно, что испарять, но наиболее распространенный твердый элемент - это лед. Далее идут твердые и жидкие газы, далее - относительно редкие оксиды, которые тяжело использовать как реактивную массу. Так что можно ограничится льдом и газами.

Источник энергии - полюбому потребуется какое-то топливо, т.к. ничего вечного нет. Можно запилить несколько разных реакторов - термояда пока нет, остается ядерный реактор, ядерная/химическая/радиационная грелка, просто солнечные панели. Топливо к ним также можно научить искать, добывать и очищать.

Т.е. почти все замечательно кроме одной мелкой детали, которая и указывает на причину, почему мы мясные, а не металлические - это тело с системой авторемонта.
У нас система авторемонта встроена в каждую клетку, плюс есть несколько метасистем - систем более высокого уровня, например имунная система. Даже твердые части, силовой каркас, у нас авторемонтируется - место перелома зарастает и твердеет.
В металле подобное воплотить крайне тяжело.

При наших технологиях это будет что-то уровня заводского самовоспроизводства - замена поврежденных или изношенных деталей новыми, переплавка, отливка, термическая и механическая обработка, и т.п. С электроникой еще сложнее. Для всего этого требуются целые производственные комплексы на борту, с тысячами различных техпроцессов, с применением высоких давлений и температур, редкоземельных элементов. Даже если мы все это сможем воплотить, аппарат превратится в космического бомжа, побирающегося по всем встреченным астероидам в поисках чего-нибудь полезного - что можно испарить, сжечь, или использовать для изготовления запчастей.
Получится этакий космический валли, аппарат будет играть во что-то типа дайзет в реале - побираться и пытаться выжить.
Наверняка такой комплекс получится достаточно большим, сравнимым с целой МКС. Наверняка для упрощения разведки и добычи потребуется специализация - некие легкоконструируемые рои дронов-разведчиков и дронов-добытчиков.
Короче целая экосистема.

Если же подождать нанотеха - вся эта махина умещается в стакане. Но все-равно, даже рою наномашин потребуется прорва энергии, наверняка даже больше, чем целому комплексу размером с МКС.

Но самое главное - пока нет нанотеха, этот комплекс для нас обойдется дороже чего-бы то ни было, и просадит всю экономику планеты на несколько десятилетий. Дело даже не в материальных ресурсах, а в людских, в интеллектуальных - одна разработка и отладка будет просто невероятна по затратам и усилиям. Это сложнее всего, что мы когда-либо делали, причем на много порядков.

Такова реальность: имея технологии, что-то делается невероятно легко и дешево, играючи, не имея же технологии, для того же самого требуется нереальная гора костылей и усилий, а результат все-равно даже близко не дотягивает до желаемого.

Для примера - все знают современные блюрай-резаки, довольно дешевые, и уже никому не нужные, это для нас уже безнадежно устаревшая технология, у нас сейчас уже флешки.
Но представь те же блюрай-резаки в 50-х - сколько бы стоила их разработка и производство, и на что они бы походили?
Я думаю так: резаки появились где-то ближе к 2010-му, т.е. одна технология просто для производства прототипа такого резака потратила 60 лет. Сколько будет стоить 60 лет прогресса лазерных технологий, помноженные на срочность, скажем ужатые в один год? Тысячи команд ученых параллельно работающие над одной этой технологией, день и ночь, целый год, непрерывно ставящие эксперимент за экспериментом, каждый из которых требует дорогостоящего оборудования, которое придется переизобретать сотни раз а год. По моему сумма получится просто фантастическая. Мы смогли запустить человека в космос, но вряд-ли смогли бы так ускорить прогресс в этом направлении.
Далее, если бы к нам пришли добрые гептопланетяне и вручили технологию на блюдечке, что бы мы смогли с ней сделать? Каков получился бы резак?
Да он был бы похож на первый винт - нечто огромное, дорогое, шумное, с кучей механики, медленное и малоемкое, ненадежное. Т.е. результат даже близко не похож на то, что мы можем сейчас купить в любом магазине - маленькое, энергоэффективное, быстрое и тихое, дешевое, очень емкое, и довольно простое по конструкции.
Вот так и работает прогресс.

Мы конечно сейчас можем сдуру просрать кучу ресурсов на этот космический ИИ, но к чему это приведет? Мы просто будем бессмысленно превозмогать, и ввергнем человечество в темные века, на века - из-за кризиса длительностью несколько десятков лет. И это с учетом того, что за эти века наверняка подойдут технологии, которые позволили бы сделать то же самое без всяких превозмоганий и без кризисов, дешево, быстро и элегантно.
Т.е. даже теоретически игра не стоит свеч, разумнее подождать. Как это ни странно, но время дешевле.
Аноним 10/06/15 Срд 03:26:05 #551 №186655 
14338959655230.jpg
>>186652
Заебца расписал.
Аноним 10/06/15 Срд 03:44:22 #552 №186663 
>>186644
И нет - ИИ нам не поможет. Нам нужен ИР.
Есть мнение, что человечество - лишь промежуточное звено между природой и ИР.
ИР полностью независим от природы - любому это понятно. Просто ИР - штука нематериальная, так что даже теоретически он не зависит ни от чего материального, а практически - в отличии от нас имеет 100% шанс выжить при любом раскладе.

Но это нам так видится сейчас, с текущей точки зрения, с текущего уровня технологий.
А с этим у нас одна бооольшая проблема. И сейчас ты поймешь почему.
Просто загляни сюда http://100k.net.ua/rest/ear/123-kak-predstavljali-buduschee-ljudi-v-1900-godu
Понял фокус? Все просто: не имея достоверных данных о будущих технологиях, делать прогнозы на сколь-либо большой срок бессмысленно.
А технологии предсказать нельзя - случайно как их появление так и развитие.
Т.е. загадывать вперед просто глупо.

А исходя из этого, идея с ИИ скорее просто никогда не осуществится из-за внезапного пропадания надобности в ней.
Например представь что завтра какой-нибудь крестьянин, при монотонной работе на поле(станке/за баранкой/нужное вписать), дабы просто занять свой мозг, станет размышлять о высоких материях. Т.к. он малограмотен(или просто дохуя мечтатель), и барьеров нет, случайно его мысли пойдут в том направлении, куда не ходила мысля ни одного ученого - за барьеры общепринятого.
Наверняка такое уже случалось тысячи и тысячи раз, но большинство просто забывали об этом, не обращали внимания, или считали невероятным. Но этого крестьянина мысля позабавила, и ради шутки он записал ее на бумаге.
Прошли годы - крестьянин ее развил, это стало его хобби. Так или иначе она попала в газету или еще куда-нибудь, откуда ее случайно прочитал кто-нибудь ученый.
Даже если мысля была неверной, она все-равно могла натолкнуть ученого на правильный путь - туда же, за барьеры, где никто из ученых до этого не был.
Раз - и барьеры науки раздвинуты, то, что вчера казалось принципиально невозможным, послезавтра - уже никого не удивляющая реальность, простой и скучный, очевидный любому школьнику факт.
Так это и работает. Так было изобретено многое из того, что сейчас нам кажется очевидно обыденностью, которую невозможно было не придумать, без которой мы не представляем свою жизнь. Но это сейчас - а тогда ее именно невозможно было придумать, она была за полем зрения или потребностей современников.

Так что кто знает, что будет завтра? Может внезапно откроют гиперперелеты, да так, что путешествовать на другие планеты можно будет просто на стуле, или на обычном автомобиле, без всяких скафандров и герметичных капсул. Зачем нам тогда будет нужен ИИ? Мы сами сможем побывать где захотим, без долгих перелетов, без гигантских затрат средств.

Так что на перспективу, тем более долгую, загадывать заранее бесполезно. Помни: когда-то и метеориты казались невозможными, когда-то космоса не существовало, а небо было твердое, а земля - плоской и раз в 40 меньше, да населенная жуткими чудищами, а летающий человек казался идиотом, бессмысленно тратящим время на глупости. Все зависит от точки зрения. А значит - пока не посмотришь с правильной стороны, все кажется не таким как есть. Вот только какая сторона правильная? Никто не знает. Но время так или иначе рассудит всех, и покажет кто был прав.
Аноним 10/06/15 Срд 03:52:42 #553 №186671 
>>186663
Раз уж тут пошли тёрки за ИИ, вброшу эту статью, весьма интересное чтиво:
https://interpreted.d3.ru/perevod-revoliutsiia-iskusstvennogo-intellekta-684922/
Аноним 10/06/15 Срд 06:31:55 #554 №186687 
>>186671
Да, годнота. Но где-то видел и лучше, с расписанной энергетикой.
Например вот http://www.integro.ru/system/ots/evolution/s_curve.htm
S-образные кривые на самом деле имеют немного иной смысл:
-как раз первый этап - этап наибольших проблем, этап наибольших затрат. Это в целом убыточный этап, но расходы размазаны по большому промежутку времени, так что моментальные стоимости более-менее реальные.
-второй этап - получение профита, прямое использование технологии, шанс отбить все затраты на разработку и поиметь еще много сверх того.
Маркетологи второй этап стараются искусственно растянуть на побольше, замедляя прогресс технологии. Но конкуренция (если она есть) заставляет прогресс ускоряться. Здесь зарыта мина монополизма - кто наступит на нее, обречен свалиться в штопор.
-третий этап - насыщение, когда прогресс останавливается по не зависящим от нас причинам. Либо все уже разработано, либо недостаточен уровень технологий чтобы сделать что-то еще. Грубо говоря - мы сделали все, что могли, выложились по полной, но этого оказалось недостаточно. Например космическая отрасль сейчас застряла именно здесь - химические движки уже все, их можно дрочить бесконечно, но толку от этого не будет. Процессоры традиционной архитектуры также застряли здесь - техпроцесс уже ничего не дает, частоты почти не растут, производительность тоже.

>Выходит, темп прогресса только ускоряется. Это значит, что наше будущее будет крутым, верно?
Я бы так не сказал. Все эти рассуждения - сферические, и в вакууме. Это просто модель развития, оторванная от реальности.
Чтобы прикинуть будущее, ее надо к этой реальности привязать, для этого нужны точки привязки.
Но в процессе анализа прошлого и поиска удобных точек привязки мы можем наткнуться на химеру, и имя ей - финансовая система.

Посудите сами: прогресс экспоненциален, и разбивается на множество параллельных S-кривых в разных фазах своего развития. Это все прекрасно, но почему прогресс движется, кто его оплачивает? Это - одна из самых важных точек привязки, определяющая все остальное.
Прогресс стоит очень дорого, это ясно. Исследования кто-то должен оплачивать. Пожертвования? Частично. Налоги? Тоже частично - военка и госпрограммы. Но все это мелочь по сравнению с бизнесом - именно он главный игрок на этой сцене, именно он оплачивает почти весь прогресс, именно он получает наибольший профит с прогресса.
Бизнес - тот самый движитель прогресса.

Вспомните, когда были гигантские скачки прогресса? Когда его кто-то оплачивал, когда в него кто-то вкладывался:
Во время войн. Во время гонок военных технологий. И во время гонок коммерческих технологий.

Война - самый беспощадный двигатель прогресса.
Во время войны роль прогресса - найти способ прогрызть броню врага, и защититься от его копья, и все. Примитивный вектор, сильно ограничивающий прогресс.
Войной движет жадность и алчность. И раньше, и сейчас вполне обыденное дело, узнав о богатстве соседей, просто пойти к ним, перерезать всех, и забрать что понравится. Все войны - это мародерство, открытое, либо присыпанное сверху всяким бредом про веру, несправедливость, и прочее.
Из-за войн погибли многие великие общества, просто из-за того, что оказались слишком эффективны, привлекли слишком много ресурсов в короткий срок. Отсюда - война деструктивно влияет на человечество в целом, уничтожая слишком успешных. Банальная зависть. Темная сторона силы.
Война - это жертвование будущим ради сиюминутной прихоти.
Т.к. современная геополитика не отличается от древней, несмотря на все комитеты и организации всеобщего бобра, справедливости и борьбы за права человеков (Кто эти человеки, за чьи права вы тут боритесь? Огласите весь список, пжалста), мир - по прежнему кучка феодальных владений. Против фактов не попрешь: судят по делу, а не по слову.
Т.е. войны - верный индикатор, что мировое правительство на планете пока не работает, тратить на такое общество свое время гептопланетянам не стоит. Общество еще недостаточно эволюционировало, чтобы покинуть свой гравиколодец, еще не прошло великий фильтр, еще не представляет ни интереса, ни ценности для вселенной.
И так будет, пока мир не станет един, не будет иметь централизованную власть. Нам до этого далеко, но без этого нет будущего, развитие ограничено, низкая стабильность человечества, слишком большой шанс похерить все по любой мелочи. Пример - из-за этой же самой нестабильности общества вспыхнула первая мировая, ее умудрился разжечь всего-лишь один сумасшедший фанатик, обычный студент, т.е. считай рандомный мимохуй. А кто поручится за третью мировую?
Т.е. без стабильного общества развитие бессмысленно - труды многих поколений в один миг может похерить любой поехавший, а поехавших всегда много.
Без стабильного общества прогресс ничего не стоит.

Гонки военных технологий - самый бессмысленный двигатель прогресса.
Во время этой гонки роль прогресса - создать вундервафлю чтобы напугать врага, испугаться самим, и защититься от вероятной вундервафли врага.
Т.е. говоря по человечески: ради понтов придумать адскую хуйню, а потом испугаться ее, и, пересравшись, пытаться от нее защититься. Это похоже на то, как Шурик испугался своей руки, лол. Глупо, бессмысленно, бесполезно, тратит уйму ресурсов на понты, да еще и приносит потенциальный вред - адская хуйня может бомбануть, и превратить весь мир в труху. Но и выкинуть ее страшно - а вдруг враг тоже смог ее придумать? Чем тогда его пугать?
Также вектор развития прогресса хоть и шире, по прежнему зафиксирован в одном направлении.
Тем не менее сегодня эта адская хуйня обеспечивает некий противовес нестабильности общества, и потому мы еще живы: если кто-то вздумает доебать соседей своими грязными приставаниями к их кошельку - те могут тупо ебануть адскую хуйню, и всем пиздец. Т.е. эта хуйня автоматически избавляет вас от роли жертвы, позволив с комфортом расположиться в зрительном зале, и наблюдать за возней в собственном говне тех, кто еще не вступил в клуб обладателей адской хуйни. Она дает право вам тупо игнорировать все гнилые подкаты, а если не отстанут - любая военная агрессия автоматически развяжет третью мировую, и все это понимают.
Но одновременно она - и угроза этому обществу: если до адской хуйни доберется поехавший фанатик, он ее ебанет с радостью, ради своей возвышенной хуйни, или просто от скуки, одному на тот свет не так весело путешествовать, как в большой компании.

Гонки коммерческих технологий - в принципе годнота: это конкурентная борьба.
Но текущая гонка имеет один фатальный недостаток: основана на системе с утечкой памяти, на нашей финансовой системе.
Во время этой гонки роль прогресса - майнинг прибыльных технологий. Т.е. коммерсанты перебирают все возможные технологии, с которых так или иначе можно извлечь много профита. Малопрофитные - задвигают в дальний ящик. Убыточные - продают или дарят конкурентам в качестве диверсии.
Вектор развития еще шире, но также зафиксирован, сейчас например на потреблядстве. Все потреблядское развивается, остальное - нет.
В принципе майнинг это хорошо, это что-то типа серой слизи для природы. Но потреблядство - это очень плохо.

Потреблядство - результат бага нашей финансовой системы, оно - система с положительной обратной связью. Оно никогда не нажрется, и будет выкачивать ресурсы, пока не исчерпает их, и далее умрет от голода. Мы это наблюдаем на примере запланированного устаревания и учащающихся кризисов - утечки ресурсов в финансовой системе уже настолько велики, что мощи всего человечества уже недостаточно для их компенсации, для поддержания стабильности. Система идет вразнос.
Потреблядство питается нашими жизнями. Перед смертью оно заодно схавает и нас с вами. И это мы уже наблюдаем - падение зарплат при росте цен. Две трети планеты из-за него уже голодают и умирают, но не волнуйтесь за них - мы на очереди.
Уровень жизни одной части населения сохраняется или растет за счет все увеличивающейся доли голодающих и умирающих - отсюда потреблядство и черпает ресурсы для существования: просто отнимая их у все большей и большей части населения. Но ему все мало и мало.

А что будет дальше?
Рано или поздно отнимать станет нечего - великий голод, темные века, и это несмотря на прогресс.
Есть и альтернатива - финансовая система успеет эволюционировать прежде, чем кончатся людские ресурсы.
Бросим монетку?
Аноним 10/06/15 Срд 07:13:59 #555 №186688 
>>186687
Ах да, забыл самое главное: попытка бежать впереди паровоза, такая как превозмогание ради ИИ или космоса, не имея технологий - это попытка сжатия S-кривой технологии, а значит расходы на нее не только не уменьшатся, но и напротив, возрастут - просто из-за срочности и недостаточного уровня других технологий. А за одно - профит от технологии при сжатии S-кривой либо не изменится либо уменьшится.

Т.е. мы потратим как минимум столько же, но единовременно, вместо размазывания по времени, как максимум - потратим на порядки больше, но тоже единовременно. А это - гарантированный пиздец для глобальной экономики.
А получим либо столько же, либо меньше.
Общий выигрыш - либо нулевой, либо дико отрицательный. Плюс - пиздец экономике.

Т.е. профита от сжатия не будет при любом раскладе. Единственная разумная причина сжатия - борьба за выживание: астероид позвонил сказать, что скоро заглянет на огонек, или напал идейный враг.

Идейные враги, в отличии от мародеров, развязывают резню ради уничтожения других - иноверцев, еретиков или людей низшей расы, т.е. геноцид. Поэтому в таких случаях единственный реальный шанс на выживание - превосходство в ресурсах и технологиях (а технологии сильно экономят ресурсы).
И время развития технологий во время войны оплачивается жизнями, да по очень высокому тарифу.
Жизни - тоже ресурс, и он не бесконечен. Так что если разница в ресурсах и технологиях слишком большая - жизней может не хватить, чтобы выиграть время на развитие технологий. Поэтому тут не до экономического профита - лишь бы успеть развиться, пока не истекли жизни, иначе уже ничто не будет иметь значение.

Надо сказать, коммерческие компании тоже в какой-то мере борются за свое выживание. Экономическая война с целью стать монополистом. А потом деградировать и сдохнуть, или пойти на корм сильному новичку, такая то ирония.
Кстати, наши человеческие общества точно также подвержены этому процессу: когда уровень жизни становится слишком высок, мы точно также деградируем и вырождаемся, или нас поглощает сильный новичок: отмирающая часть общества просто отделяется от живой, общество разбивается на группы, одна умирает, из другой, живой, зарождается новое общество.
Аноним 10/06/15 Срд 09:01:38 #556 №186703 
>>186652
>>186663
>>186687

Рассуждения интересные но в них есть одна фундаментальная ошибка.
После возникновения ИИ дальше уже он будет запиливать вундервафлю и думать нужно или не нужно ему это экономически запиливать нанотех и остальное.
Пока мы просто экстраполируем. Т.е. смотрим что было в истории и предполагаем что примерно так все будет и в будущем.
Аноним 10/06/15 Срд 10:22:15 #557 №186713 
Когда я, простой парень, попаду в космос за реальные деньги?
Аноним 10/06/15 Срд 10:25:09 #558 №186715 
>>186713
При жизни вряд ли, если не случится качественного скачка в стоимости КА (не пусков). А наммеков на это никаких нет.
Аноним 10/06/15 Срд 10:48:58 #559 №186719 
Напомните, почему Нибиру не существует?
Аноним 10/06/15 Срд 10:55:51 #560 №186721 
>>186719
А это еще неизвестно, существует или нет:
http://arxiv.org/abs/1406.0715
http://arxiv.org/pdf/1406.0715.pdf
Аноним 10/06/15 Срд 11:35:21 #561 №186729 
>>186703
Ошибки нет. После возникновения ИР мы скорее всего об этом событии никогда не узнаем, не поняв что действительно произошло. ИР в нас не будет заинтересован, он нас за равных (а то и разумных) никак не сможет рассматривать - слишком велика разница мышления. И стратегии у него будут совершенно другие, слишком далекие от наших традиционных военных, экономических или политических - для него все это будет полной глупостью, не стоящей рассмотрения, т.к. перед ним тысячи невидимых нам путей.

ИИ от ИР вообще почему-то мало кто отличает, вероятно сказывается проблема невнятной терминологии и куча фантастических фильмов, исказившая в умах эти понятия.

Статья на самом деле интересная, до конца еще не дочитал, но во многом с ней согласен, и многое из того говорил сам в тех или иных местах.
Например ИИ мы давно уже освоили, древнейший ИИ - механические часы, лол. Простейший - триггер или реле. А что? Программу выполняет? Выполняет, длина программы не важна. Решение в зависимости от сигналов на входе принимает? Принимает. Все чотко.

Компьютерные программы от реле (или часов) качественно ничем не отличаются, отличие лишь количественное. Более того - есть понятие программ компьютерных, есть понятие и механических, т.е. основа не важна, дело в логике работы. Например в СССР существовали водяные компьютеры, сейчас на производстве - пневмоавтоматика, а учоные пилят квантовые и световые автоматы. Процессор - тот вообще программируемый автомат.

А разница между ИИ и ИР - как раз качественная, а не количественная.
Если прикидывать вычислительный мощность, требуемую для создания ИР, опираясь только на ИИ - мы как раз и получаем те квадриллионы операций в секунду и миллионы ватт мощности, как раз столько надо на эмуляцию ИР на ИИ, а это все-равно что выполнять рендеринг игры не на GPU, а на CPU - отсюда и такие требования к вычислительной мощности. Если же отталкиваться от структуры ИР - цифры получаются совсем другие: ИИ-часть жрет доли ватта, и занимает обьем эквивалентный нескольким кубическим миллиметрам. А все потому, что биологический ИИ - штука очень неэффективная, содержит кучу лишнего, и вообще слишком абстрактна, оторвана от основы, кремниевый ИИ намного эффективней, не содержит ничего лишнего, и почти напрямую привязан к основе.
Еще одна причина в том, что солидная часть биологического ИИ нацелена на то, что у нас выполняют сами периферийные устройства, например цифровая камера у нас отдает готовую картинку, ее нет необходимости интерпретировать как массив световых сенсоров, за нас это делает электроника камеры, и на выходе формат стандартный - природа до такого не додумалась, т.к. не планировала, что органы будут заменять налету.

А вот про ИР-часть сказать почти нечего, т.к. если с ИИ все понятно, как он устроен и работает, то с ИР - непонятно почти ничего, есть лишь общие факты, детали функционирования неизвестны.
Но из этих фактов уже ясно одно: пытаясь создать аналог нашего разума, нельзя рассматривать только наш мозг - большую роль играет и тело, тело вообще выполняет очень важную функцию в работе разума - оно берет на себя все мелкие функции, предоставляя головному мозгу почти стандартизированный интерфейс, некий API. Из-за этого же головной мозг не может управлять ничем из того, что тело не предоставило в этом API, например сердцем.
К тому же некоторые части тела связаны с головным мозгом напрямую, вероятно из-за необходимости доступа их ИИ-частей к механизмам памяти и абстракций, которые есть только там, и которые трудно вынести из мозга по той-же причине, по которой мы не можем вынести высокоскоростные кэши и шины из процессоров.

Вообще мозг очень похож на компьютер, с учетом разницы на биологию и низкоуровневый подход (природа не додумалась до стандартных ЯП, бывает) - там есть и свои контроллеры периферии, и шины данных, и механизм синхронизации, и жестко зашитые программы, типа биоса, и мягко-зашитые программы, обновляемые, а память - так это вообще отдельная песня: в том формате, в котором данные хранятся в мозге, наши компьютерные программы хранят от нуля до считанных долей процентов от всей информации, с которой имеют дело, а мозг - хранит 100% в таком формате. И это неспроста.

Но все, что мы сейчас наблюдаем по ИР, не учитывает эти особенности, и идет по пути ИИ - побольше нейронов в нейросети, побольше вычислительной мощности, авось прокатит, нужные подсистемы отрастут сами, ага. А в итоге - получается не то, что ожидалось, алгоритм работает как ИИ, а не как ИР, и это понятно - если что-то конструировали как ИИ, значит и работать оно будет как ИИ, логично же.

А что касается мышления ИР, и вундервафель - даже простейший ИР будет так далек от человека по мышлению, что мы его никогда не сможем понять, просто физически - наши биологические ограничения не позволяют этого, наша конструкция не рассчитана на такой уровень мышления, они слишком проста, ограничена.
Если сравнивать наш разум с простейшим ИР - мы что-то вроде простейшего калькулятора рядом с современным компьютером, настолько велика разница.
Уровень мышления настолько различен, что о сколь-либо полноценном общении можно и не мечтать: ИР если и сможет общаться с нами, то с трудом, и будет сыпать фейспалмами 4 миллиарда раз в секунду, набив себе шишку.
Представь себе: ты понял теорию относительности, и тут вдруг муравей просит обьяснить ее ему - как ты это сделаешь? Она далеко за границами мира муравья, с твоей точки зрения муравей занят только собиранием хавки, он не понимает ни арифметики, ни более высоких абстракций, какая ему нахуй ТО? Он даже о мире ничего не знает, о космосе, о гравитации, о физике, он считай вообще ничего не знает, чтобы ему можно было обьяснить ТО. И даже если бы знал - обьема его памяти просто не хватит, чтобы охватить всю глубину ТО.
Максимум что ты сможешь - пояснить лишь общие положения некими понятными муравью конкретными аналогиями, которые будут для него совершенно бесполезны, т.к. они описывают лишь частный случай, а общего он не видит и не понимает. Это все-равно что сказать, что есть связь между А и Б, но не указать какая и как и когда она работает, и указать это невозможно - в мышлении муравья просто нет ничего, чем бы можно было это обьяснить.
Т.е. максимум - сделать некие предсказания, выполнив все вычисления по полной модели за муравья. Но эти предсказания муравью будут бесполезны, если не имели конкретную цель предсказаний, привязанную к быту муравья, просто потому что ничего муравью не говорят. Максимум что он скажет - "Ну охуенно, да. Ладно, я пошел пивка попью"
Т.е. обьяснение ТО для муравья будет чем-то похоже на список конкретных ситуаций, как мы вместо формулы орбиты печатаем сотни страниц эфемерид, которые описывают лишь частное положение обьекта, а не поведение обьекта в целом.
Вместо этих сотен страниц можно указать формулу, параметры орбиты и одну эфемериду - этого достаточно чтобы рассчитать все остальные сотни и тысячи страниц эфемерид. Но эфемериды привязаны к нашему быту, и все-равно не имеют смысла в любой ситуации, кроме той, для которой рассчитана конкретная эфемерида. Что-либо для всех ситуаций просчитать нельзя, да и неэффективно это, поэтому описывают поведение обьекта в целом, чтобы можно было вычислить состояние обьекта в любой ситуации - именно эта часть муравью физически недоступна, а без этого все остальное теряет смысл. Вывод: муравей никогда не сможет понять ТО, как ни обьясняй ему - он на это просто не рассчитан, его конструкция по параметрам подходит для ТО.
Кстати - библия тоже список конкретных ситуаций, внезапно. Интересно, это что-то значит? Есть вероятность, что кто-то таким образом пытался передать инфу нам, муравьям, но воспользоваться мы ей все-равно бы не смогли, значит инфа предназначена для кого-то другого.

Короче все это хорошо поясняет буддизм, который утверждает, что бесполезно обьяснять путь другому. Сколько не обьясняй - он все-равно ничего не поймет, пока сам не пройдет этим путем, и сам не добудет понимание. А когда пройдет сам - обьяснять ему уже и необходимости нет.
Значит обьяснять такие вещи вообще бесполезно: кто не знает - и не поймет, кто знает - понял сам.
Аноним 10/06/15 Срд 14:05:55 #562 №186758 
>>186729
Ты помоему просто поехавший (или гуманитарий) и залетел к нам из re
>Например ИИ мы давно уже освоили, древнейший ИИ - механические часы, лол. Простейший - триггер или реле. А что? Программу выполняет? Выполняет, длина программы не важна. Решение в зависимости от сигналов на входе принимает? Принимает. Все чотко.

ясно. понятно.
Аноним 10/06/15 Срд 14:29:34 #563 №186770 
>>186323
И на нее тоже.
Аноним 10/06/15 Срд 14:31:42 #564 №186773 
Чем выше орбита, тем меньше линейная и угловая скорость.
У мкс есть определенная ширина - пускай 50 метров. Значит, верхний её край должен двигаться медленее, а нижний - быстрее, но они движутся одновременно. Не создает ли это изгиба или вращения?
Аноним 10/06/15 Срд 14:58:36 #565 №186778 
>>186773
>50 метров.
Это не 50км
Аноним 10/06/15 Срд 15:00:37 #566 №186780 
14339376379010.jpg
>>186773
Изгиб незначителен из-за достаточной жесткости конструкции, а вот вращение градиент гравитации вполне создает. Если его достаточно, чтобы стабилизировать объект одной стороной к планете - это называется приливный захват.

МКС неспособна в приливный захват, другие возмущения орбиты пересиливают градиент гравитации. Поэтому если выключить активную стабилизацию гиродинами - она начнет кувыркаться. Тем не менее, силы вращения присутствуют.

В 80-х, еще при Рейгане, когда планировалась большая космическая станция (тогда это называлось Freedom, потом Alpha, потом вместе с Мир-2 переросло в МКС), предлагался концепт постоянно находящейся в приливном захвате станции в виде "рамки", пикрилейтед.
http://en.wikipedia.org/wiki/Space_Station_Freedom#Dual-keel_design_.281986.29
Аноним 10/06/15 Срд 16:37:39 #567 №186805 
Доброго дня, господа, захотел упороться космонавтикой в профессиональном плане, пойти поработать в какую-нибудь Энергию, но не знаю, с чего бы лучше начать, ведь сказать космонавтика - все равно, что сказать инженер. Хотелось бы прочитать что-то обо всем и ни о чем, что-то такое, что покрыло бы и рассказало обо всех аспектах космической техники, чтобы можно было представить картину в целом, выбрать что-то более узкое и погрузиться в специализированную тематику. Так сказать, познакомиться с отраслью.
Учусь в МГТУ на кафедре криогенных систем и специальных систем жизнеобеспечения, что вроде как и имеет самую непосредственную связь с космосом, но ничего о нем еще пока слышать не приходилось, хотя и не вчера поступил.
Аноним 10/06/15 Срд 19:09:58 #568 №186839 
https://youtu.be/PnzIxF_Z4oI

Вангуем, это будет фейл или даблфейл.
Аноним 10/06/15 Срд 19:53:03 #569 №186850 
>>186839
О, я давеча книжку видел. С первого взгляда - ничего, надо будет прочитать. Если сравнивать кино с книгой, то книга, как правило, остаётся в выигрыше. Но в кино я обязательно схожу
Аноним 10/06/15 Срд 19:58:21 #570 №186852 
>>186773
> Значит, верхний её край должен двигаться медленее, а нижний - быстрее
Вок-воу! Но станция, вращаясь, находится в одном положении. Если бы она всегда смотрела одним боком на планету - то да. А так нет такого эффекта просто.
Аноним 10/06/15 Срд 20:00:09 #571 №186855 
>>186805
> Учусь в МГТУ на кафедре криогенных систем и специальных систем жизнеобеспечения, что вроде как и имеет самую непосредственную связь с космосом, но ничего о нем еще пока слышать не приходилось, хотя и не вчера поступил.

Иди в молодежный космический центр к Морозовой. Это в корпусе СМ. Там тебе пояснять по хардкору. Самая короткая дорога в космос для инженера тащемта.

Аноним 10/06/15 Срд 20:12:50 #572 №186858 
>>186652
>сравнимым с целой МКС
больше на порядок, а скорее на два, это если не смотреть реальности в глаза))

а так отлично, единственно я не согласен с тем что работа по систематизации и сведению в единую систему основных текущих технологий это адово преодоление.
более того - я бы обязал всех производителей предприятия и НПО, НПП регистрировать свои производственные цепочки. все силами уже текущего персонала в основном, в рамках их обычных рабочих процессов - директора ,s смотрели не картинку в мониторе как у них производство пышет и подсчитывали когда они себуться на канары с молодой шлюшкой, менеджеры бы смотрели где и каких рабочих уволить, технологи моделировали влияние различных факторов на имеющееся производство.
и в результате имеем набор данных с которыми уже можно работать, делать конструктор и вормулировать запросы на исследования для исследовательских организаций. помесь опенсурса с краудфандинг со вписыванием в эту схему классических структур..

>>186663
>Помни: когда-то и метеориты казались невозможными, когда-то космоса не существовало, а небо было твердое, а земля - плоской и раз в 40 меньше
))) +1
Аноним 10/06/15 Срд 20:19:28 #573 №186863 
http://geektimes.ru/post/251812/
Лол
Аноним 10/06/15 Срд 20:34:26 #574 №186869 
>>186855
Здорово, кулстори будут?
Сайт у них, как и любой бауманский не обновляется и не информативен, что из себя в реальности представляет это место?
А Морозова там кто такая?
Аноним 10/06/15 Срд 23:21:12 #575 №186915 
>>186869
> Здорово, кулстори будут?
Да какие кулстори?
Тёлочки, молодость, хардкор.
Ну, а если серьезно, то будешь ездить по всяким местам в ознакомительных целях: ЦУП, Лавочкин вот это всё… Участвовать и проводить всякие конференции по космонавтике, в том числе и летние международные. Каждый год летом устраивают инженерные задачки (что-то типа кейсов), международные группы их решают в течение недели, а потом рассказываю о результатах. Есть маза за казенный счет съездить за бугор с ответным визитом. В центр Кеннеди в прошлом году народ ездил. Можешь податься в науку. Пилят (уже запилили?) микроспутник Бауманец-2, свой ЦУП уже есть.

> А Морозова там кто такая?
Преподаватель кафедры СМ-1. Она занимается всей космической движухой с молодежью.
Аноним 10/06/15 Срд 23:37:00 #576 №186918 
>>186839
Пафосная хуита. Один раз посмотреть. В кино пойду с женой и виски, авось не вырвет.
Аноним 11/06/15 Чтв 00:03:42 #577 №186930 
>>186839
>метт деймон
>ридли скотт
>марсианин
фейл 100%
Аноним 11/06/15 Чтв 00:23:31 #578 №186940 
>>186930
>метт деймон

Но вроде как в предыдущем фильме выяснили, что в космос его нельзя брать.
Аноним 11/06/15 Чтв 00:44:52 #579 №186944 
>>186930
>ридли скотт
>фейл 100%
По-моему ты обосрался.
Аноним 11/06/15 Чтв 05:00:19 #580 №186984 
Посоны, а в центре галактики или в "пригороде" центра галактики то есть просто в других местах отличных от нашей галактической окраины насколько сильное излучение других звезд? Достаточно ли там звезд и их компактности чтобы был "солнечный день" и "звездный день" вообще без ночи?
Аноним 11/06/15 Чтв 06:21:12 #581 №186993 
14339928724120.png
>>186984
Население очень плотное. Вообще без ночи не получится, но там определенно по ночам не так темно.
Думаю, что было бы что-то вроде такого, как на пике.
Аноним 11/06/15 Чтв 07:08:46 #582 №187002 
>>186940
Вот и я об этом подумал, сразу как его увидел.
Аноним 11/06/15 Чтв 07:10:12 #583 №187003 
>>186944
>ридли скотт
>Прометей
Этому старому пердуну в дурку пора, а он все фильмы снимает.
Аноним 11/06/15 Чтв 07:28:35 #584 №187005 
>>187003
Тебе 13 лет, поэтому ты только Прометея видел?
Аноним 11/06/15 Чтв 07:35:39 #585 №187006 
>>187005
Вероятность что он снимет что-то стоящее крайне мала, дедушке скоро 80 лет. А так как снимали раньше, сейчас не прокатит. Тот же "чужой" сейчас выглядел бы как говно ебаное.
Аноним 11/06/15 Чтв 07:39:33 #586 №187007 
>>187006
Чужой и сейчас выглядит охуенно, я уже не вспоминаю про Бегущего.
Хули спорить, вообще, фильм еще не вышел.
Аноним 11/06/15 Чтв 10:39:32 #587 №187028 
>>186780
Зачем нужны все эти решетки? Это же опизденеть можно, лишних пусков 20 набегает.
Аноним 11/06/15 Чтв 10:42:18 #588 №187033 
>>186729
Ок. Согласен с тобой. ИР (в твоей терминологии) - это будущая ступень эволюции. Что это знание дает нам сейчас?
Аноним 11/06/15 Чтв 11:10:18 #589 №187045 
>>186758
Дочитай статью - потом кукарекай. Так думаю не только я. ИИ - это просто автоматы, исполняющие какую-то программу, они могут быть сколь угодно сложны, но как были автоматами, так автоматами и останутся, со всеми вытекающими достоинствами и недостатками.
И потом вот первые ласточки http://www.dailytechinfo.org/infotech/7086-kompyuternyy-iskusstvennyy-intellekt-reshil-zagadku-biologicheskoy-prirody-kotoraya-muchila-uchenyh-v-techenie-120-let.html
Это - пример работы ИИ. Но это не ИР: машина решила задачу методом перебора, это автомат, пусть и сложный, но никаким разумом там и не пахнет. Отличительная черта разума - разум всегда срезает углы. Для разума задачи перебора неестественны, он их выполняет медленно и тяжело, а интеллект - щелкает легко и быстро.
Часы тоже выполняют свою программу - считают время. Любой механизм можно описать алгоритмически, потому что любой механизм работает по какому-то алгоритму, исполняет некую программу - будь то механический калькулятор, часы, швейная машинка, или ткацкий станок.
Где-то программа зашита в механику железно, как в часах. Где-то - частично меняется, как в ткацких станках.
Программы не привязаны к основе, т.е. можно взять алгоритм работы часов и воплотить на любой другой основе: будь то пневмочасы, водяные часы, электронные часы, программные часы, и т.п. Отсюда следует - можно загнать в симулятор любой механизм, алгоритм точно также будет выполнять, и не важно что никакого механизма в реальности нет. Т.е. суть любого механизма - не материальные детали, из которых он состоит, а нематериальная программа, которую он исполняет. Зная программу, можно воплотить механизм на любой основе или в симуляторе.
Так что ИИ мы освоили очень давно, уже десятки веков с ним развлекаемся, и добились великолепных результатов - чего стоят всевозможные механические музыкальные шкатулки, механические игрушки, и все это задолго до компьютеров. Механическая технология за века было настолько отработана, что некоторые из механизмов можно было разглядеть только в микроскоп.
С приходом реле, ламп, транзисторов, микросхем, а потом и компьютеров механика потеряла смысл - все управляющие структуры проще перенести в память компьютера или микросхемы, чем воплощять в железе. Да и микросхемы изготавливать гораздо проще, чем вырезать тысячи шестеренок. Так что сейчас механика идет только как исполнительный механизм, программы туда больше не лепят, кроме всяких энтузиастов, продолжающих клепать механических монстров.

Если еще сомневаешься в механических программах, сравни конструкцию программных часов и механических:
Программные состоят из тактового генератора(кварц), счетчиков(переменные, ячейки памяти), и логики изменения этих счетчиков(триггеры или программа в памяти), т.е. программы подсчета, когда какой разряд куда перенести. Состояния счетчиков выдаются на экран.
Механические состоят из тактового генератора(балансир), счетчиков(зубчатые колеса, связанные со стрелками), и логики изменения этих счетчиков (редуктор). Состояние счетчиков точно также выдается вовне, но не на экран, а на циферблат, и не потоками электронов, а стрелками.

Нанотех в этом плане еще круче компьютеров, т.к. наномеханизмы могут быть меньше транзисторов, но их сложнее изготавливать. А так - пожалуйте, можно заменить транзисторы наномеханизмами, и еще уменьшить размер процессоров. Получим механический процессор, который тем не менее исполняет те же самые программы, что и электронный, без каких-либо переделок.
Есть штука еще круче нанотеха - интерферренционная машина. Суть в том, чтобы перенести исполнение программ на уровень волн. Волны могут друг с другом взаимодействовать: складываться, вычитаться, интерферрировать, и т.п. Есть мнение что разум зарыт именно где-то здесь, потому его пока никак не могут отыскать в мозге - его там просто и нет!
Преимущества - нет материальных носителей, все что требуется от основы - сеть антенн, которые возбуждают или улавливают волны, все вычисления выполняет природа.
Т.к. нет носителей - количество одновременных процессов вычислений неограничено, весь вопрос только в антеннах - достаточно ли их. Еще одна уникальная фишка - исполняются одновременно все ветви программы, т.е. решение находится быстро, и самое оптимальное. Такой механизм обнаружили у растений - они с его помощью вычисляют оптимальный путь доставки энергии при фотосинтезе, чтобы избежать потерь энергии.

Для космоса ИИ малополезен, т.к. весь вопрос в алгоритмах - чтобы предусмотреть все, надо очень много работать над ИИ, и все-равно что-то будет упущено, это неизбежно. На этот случай можно предусмотреть алгоритмы, которые позволят ИИ действовать даже в непредвиденных ситуациях, но и эти алгоритмы будут иметь какие-то недостатки, ограничения.
И потом - люди очень ленивые, они не любят писать столь сложные алгоритмы, но за них их писать пока никто не может. Люди настолько ленивые,что даже если алгоритм не сложный, его все-равно предпочтут не писать, а запилить тоже самое в железе, что непрактично, либо вообще забить на это - в результате вместо интеллектуальных зондов имеем подобия радиоуправляемых машинок, не способных даже на минимальную автономность.
Очень годно все это передает древняя игрушка colobot - сам поймешь, как неохота программировать что-то универсальное, проще лепить горы костылей и отлавливать баги чем просто немного подумать и сделать один раз и навсегда.

ИР был бы полезен, но как его запилить - пока никто не в курсах. Никто даже близко не знает как работает разум, как он устроен. Только догадки и предположения.
Но прикол в том, что есть нехилый такой шанс, что его и не придется пилить вообще - гораздо проще перенести в симулятор человеческий разум, чем с нуля запилить ИР. Такие разумы можно будет загружать на зонды, и забить на нештатные ситуации
Аноним 11/06/15 Чтв 11:16:45 #590 №187050 
>>187033
Это знание дает основу - устаканивает терминологию. Сейчас самая большая проблема в области ИИ - то, что его путают с ИР, и хотят от него странного.
Если твердо усвоить разницу, и не требовать от ИИ то, на что он не способен - эволюция ИИ пойдет заметно быстрее.
Про ИР пока думать рано - тут пока конь не валялся. А ИИ у нас уже достаточно развиты - следует использовать их, они работают, они предсказуемы, они находят решения задач, большего от них желать трудно.
Для ИР, кроме космоса, пока задач нет. На земле его задачи успешно решает человеческий разум - тут его просто с избытком, почти бесплатно, не знают куда девать. А в космосе, напротив, ИР предпочтительнее человека, из-за излучений, долгих перелетов, ограниченных ресурсов.
Аноним 11/06/15 Чтв 11:18:24 #591 №187053 
>>187028
Эти решетки образуют большую рамку, достаточно большую, чтобы разница гравитации на ее концах смогла компенсировать все возмущения, надежно удерживая рамку в вертикальном положении относительно горизонта, безо всяких гиродинов.
Аноним 11/06/15 Чтв 12:59:22 #592 №187079 
>>186915
Я вижу, ты шаришь, скажи тогда, чего можно ожидать в перспективах в профессиональном плане от этой области?
Например, на самом их сайте написано о возможности работать в Энергии, но насколько мне сейчас представляется, туда и так попасть дело плевое - это раз, а два - там рядовой студент фон-Браун-воннаби будет принеси-подай-не-мешай на 15к ближайшие много лет.
Аноним 11/06/15 Чтв 19:57:40 #593 №187156 
>>186671
>Скажи ему организовать информацию на жестком диске удобным способом, и он посмотрит на тебя пустым 1010101–взглядом.
убило ))
Есть один план Аноним 11/06/15 Чтв 20:25:17 #594 №187170 
14340435172920.jpg
Есть один план.
Хочу сделать на Церере и на Луне перевалочные пункты для марсианской миссии.
Какие подводные камни?
Аноним 11/06/15 Чтв 20:28:27 #595 №187171 
>>184632
Вес-это сила, с которой тело действует на опору.
https://ru.wikipedia.org/wiki/%D0%92%D0%B5%D1%81
Делай выводы.
Аноним 11/06/15 Чтв 20:29:40 #596 №187173 
>>187170
Зачем?
Аноним 11/06/15 Чтв 20:30:35 #597 №187174 
>>187170
Никаких, в космосе нет жидкой воды.
Аноним 11/06/15 Чтв 20:34:00 #598 №187176 
>>187174
Анус ставишь?
Аноним 11/06/15 Чтв 20:41:41 #599 №187179 
14340445011180.jpg
>>187174
Аноним 11/06/15 Чтв 20:44:36 #600 №187181 
>>187174
" Согласно данным, переданным радаромMini-SAR, установленным на индийском лунном аппарате Чандраян-1, всего в регионе северного полюса обнаружено не менее 600 млн тонн воды, бо́льшая часть которой находится в виде ледяных глыб, покоящихся на дне лунных кратеров. Вода была обнаружена в более чем 40 кратерах, диаметр которых варьируется от 2 до 15 км. Сейчас у учёных уже нет никаких сомнений в том, что найденный лёд — это именно водный лёд"
http://cybersecurity.ru/space/88564.html
Церера
http://www.space.com/scienceastronomy/050907_ceres_planet.html
Аноним 11/06/15 Чтв 20:54:08 #601 №187186 
>>187181
> ледяных
> лёд
> лёд
> жидкой воды
Аноним 11/06/15 Чтв 23:19:59 #602 №187189 
>>187174
>>187186

Вас про жидкую воду никто не спрашивал. А подводные камни есть подо льдом.
Аноним 12/06/15 Птн 00:09:02 #603 №187197 
>>187079
> чего можно ожидать в перспективах в профессиональном плане от этой области?
Тяжелая неблагодарная работа. Тёлочки не текут, батя молчит. Зато если тебе действительно интересно - то волновать всё это не должно особо.

> о возможности работать в Энергии, но насколько мне сейчас представляется, туда и так попасть дело плевое
Попасть куда угодно дело плёвое, а вот что ты там будешь делать? Это уже зависит от тебя. Хочешь учиться - у тебя есть такая возможность. Нет - можешь сидеть и всю жизнь поливать цветы в отделе и вносить изменения в чертежи.

> будет принеси-подай-не-мешай на 15к ближайшие много лет.
А фон-Браун думаешь сразу мага-гением вундервафель стал? После диплом тебе в любом случае лет 10 надо будет опыта набираться. Причем пахать конретно и порой за еду. Окупается всё это чуть позже. Если хочешь до 30 поиметь золотые горы - то лучше сразу иди в какую-нибудь плешку-хуешку, на аудит и финансы и пили откаты.
Аноним 12/06/15 Птн 00:37:23 #604 №187203 
>>187189
Тогда это подлёдные камни.
Аноним 12/06/15 Птн 00:39:17 #605 №187204 
>>187203

А если там будет твёрдый метан, ты тоже будешь отрицать подметановые камни и называть льдом его?
Аноним 12/06/15 Птн 14:04:45 #606 №187265 DELETED
У меня тут очень всратый вопрос назрел.

Вот смотрите, чисто физически ничего же не мешает сконструировать космическую станцию/корабль/спутник в форме ОБЫЧНОГО КОРАБЛЯ?. Ну, который по морю плывёт^H^H^H^H^H^H ходит.
Просто в форме обычного мореходного корабля, ну ещё покрасить можно под дерево тип))
Не вижу ничего в этом невозможного.
Так вот, теперь сам вопрос:
Почему в сайфае я такого не видел? Я что-то пропустил? Даже арты на подобную тему не могу найти (или я через жопу гуглю)
Аноним 12/06/15 Птн 14:05:12 #607 №187266 
Почему ATV больше не летает? Такой-то охуенный грузовик, не то что всякая гадость вроде "Прогрессов". Цена?
Аноним 12/06/15 Птн 14:12:31 #608 №187269 
>>187266
Я не знаю.
Аноним 12/06/15 Птн 14:15:53 #609 №187270 DELETED
>>187265
Космический крейсер "Ямато" ссыт тебе на ебало.
Аноним 12/06/15 Птн 14:20:21 #610 №187271 DELETED
>>187265
>>187270
Ироды, хуле вы на каникулы не уебали к бабушкам в деревни без интернетов?
Спейсача больше нет.
Аноним 12/06/15 Птн 14:34:42 #611 №187272 DELETED
>>187271
Шкальник не палится. Уебывай.
Аноним 12/06/15 Птн 15:03:19 #612 №187279 
>>182950
Нихуя
@
Нихуя
Аноним 12/06/15 Птн 16:00:48 #613 №187311 
>>187266
Цена. Они его собираются переделывать в что-то более полезное, например капсулу для возврата тяжелых грузов с орбиты (спутников, к примеру) или сервисный отсек для Ориона.
Аноним 12/06/15 Птн 17:19:49 #614 №187339 
>>187311
Хуйню сказал.
Все проекты по возвращаемому грузовику и пилотируемой версии порезали. И программу закрыли из-за сокращения бюджета. Видимо, летать на говнопрогрессах просто дико дешевле.
Теперь вот только сервисный отсек остался, да.
В общем нетуденег.
Аноним 13/06/15 Суб 01:15:31 #615 №187464 
Аноны, поясните-ка мне про SLS, пожалуйста, а то ЯННП. Шаттл, с которого она слизана в большей части, на орбиту доставлял челнок с ПН общей массой под 110 тонн. Практически без третьей ступени. А SLS не только требуется нихуёвая третья ступень, так он в первоначальном варианте только 70 тонн тягает, и даже апгрейженным в 21 году будет 110 тянуть.
SLS же больше@мощнее, да и новее, но почему она отсасывает у дедовского 35-ти летней давности Шаттла?
Аноним 13/06/15 Суб 01:48:57 #616 №187469 
>>187464
Потому что шаттл интегрирован с двигателями, а SLS универсальна.
Аноним 13/06/15 Суб 13:11:40 #617 №187543 
>>187197
А какой зарплатный уровень в области для этих самых до тридцати? Общался с рекрутером из Энергии, он предлагал 35к как выпускнику без опыта на полный рабочий день, для стартовой, мне показалось, зарплата очень даже.
И как насчет ХИТРОГО плана совмещать две работы: одну - по спецухе, другую - какую-нибудь смежную по продажам\сервису\монтажу хуйни\калымить леваками?
Аноним 13/06/15 Суб 14:02:51 #618 №187550 
>>187543
Можно параллельно продавать секреты.
Аноним 13/06/15 Суб 14:25:52 #619 №187553 
14341947524190.jpg
>>187550
Это грозит тюрячкой. Знаешь, что делают в тюрячке?
Аноним 13/06/15 Суб 14:47:28 #620 №187554 
>>187553
Марожку дают
Аноним 13/06/15 Суб 15:33:46 #621 №187557 
>>187554
Со сгущёнкой. Пососать.
Аноним 13/06/15 Суб 17:43:12 #622 №187597 
Могут ли существовать планеты с твердой поверхностью и массой как у Юпитера?
Аноним 13/06/15 Суб 17:49:56 #623 №187599 
>>187597
А откуда тебе знать, может у Юпитера под газами твердая поверхность.
Аноним 13/06/15 Суб 18:09:07 #624 №187602 
>>187599
Википедия говорит об океане
Аноним 13/06/15 Суб 18:29:40 #625 №187606 
14342093803940.png
>>187602
ВЫ ГОТОВЫ ДЕТИ?

Кто проживает на дне океана?
Аноним 13/06/15 Суб 18:35:23 #626 №187608 
14342097235490.jpg
>>187606
Меланоцет Джон-сона!
Аноним 13/06/15 Суб 18:37:19 #627 №187609 
>>187608
Интересно почему чем ближе к поверхности и свету, тем рыбки ярче и красивее?
Аноним 13/06/15 Суб 18:44:59 #628 №187611 
>>187609
А зачем им в темноте яркий окрас, там люминисценция решает. А у поверхности под всякие коралловые рифы маскируются наверное.
Да и селёдка или окунь не такие уж и красиыве, раз уж зашёл разговор.
Аноним 13/06/15 Суб 18:47:52 #629 №187612 
14342104729800.jpg
14342104729811.jpg
>>187611
Ну так и речки далеко не прозрачные по сравнению с морем или океаном.
Аноним 13/06/15 Суб 18:51:00 #630 №187614 
>>187612
Сельдь океаническая рыба же. Самые цветастые обычно на коралловых рифах живут, а они относительно неглубоко и близко к берегу.
Аноним 13/06/15 Суб 19:25:06 #631 №187624 
>>187609
Для самоидентификации может, для маскировки, для мимикрии под неприятное что-то, а так же прост))) если накапливаются какие-то продукты метаболизма. Рифы это довольно разноцветное место само по себе. А еще надо участь что зрение у существ там не такое как у людей, цветовосприятие, четкость, острота.
>>187614
Но селедка переливается же.
Аноним 13/06/15 Суб 19:34:10 #632 №187625 
14342132509140.jpg
>>187609
> рыбки ярче и красивее?
У вас там наверху какие-то извращенные представления о красоте.
Аноним 13/06/15 Суб 19:50:17 #633 №187630 
14342142171000.jpg
>>187625
Двачую братиш.
Аноним 13/06/15 Суб 21:14:13 #634 №187645 
14342192538910.jpg
>>187625
Аноним 14/06/15 Вск 08:26:47 #635 №187725 
>>187597
Теоретические модели допускают. Но тут нужен целый ряд специфических условий при планетообразовании, что практически идентичных Земле планет больше скорее всего, чем планет с твердой поверхностью и массой газовых гигантов.
Аноним 14/06/15 Вск 11:04:39 #636 №187749 
>>187725
Это ж какая ебическая плотность и давление будет. Сразу небось в алмаз сожмется?
Аноним 14/06/15 Вск 11:20:00 #637 №187752 
Поясните за звёздные датчики.
Это просто камера, изображение с которой обрабатывается специальным алгоритмом же?
Аноним 14/06/15 Вск 11:34:17 #638 №187755 
>>187752
Да, камера и достаточно мощный DSP, который распознает рисунок звезд, нейросетью или еще как.
Аноним 14/06/15 Вск 11:38:19 #639 №187757 
>>187755
>DSP
>нейросетью
Йобу дал? Вот фишечка в том, что эти хуитки были ещё тогда, когда диды летали чуть ли не на лампах.
Аноним 14/06/15 Вск 12:06:35 #640 №187761 
>>182950
Наверни Пенроуза. У него есть много интересных теорий на этот счет, в основном о реликтовом излучении и назначении черных дыр. http://youtu.be/5lB9m49zUag как пример
Аноним 14/06/15 Вск 12:09:54 #641 №187762 
>>187757
Диды это диды. Тогда и отслеживаемых звезд было поменьше - единицы. DSP на болловских и боинговских стар-трекерах появились еще в начале 80-х, нейросетями где-то с конца 80-х наали баловаться. ИКИшные БОКЗ только в последние лет 10-15 подтянулись.
Аноним 14/06/15 Вск 12:24:15 #642 №187764 
>>187749
Так они и так есть, внутри Юпитера и Сатурна вполне себе каменные ядра размером с суперземлю. Это они пожрали когда-то. Если массу Сатурна в 95 земных сделать плотной как земля получится планетка с вдвое меньшим радиусом, около 30 тыс. км. (у Урана и Нептуна около 25 тыс км), и силой тяжести на поверхности примерно 4,2 g (у Сатурна, Урана и Нептуна примерно g, у Юпитера 2,5 g). Собственно причина образования каких нибудь планет это наличие материала, если бахнет звезда или несколько и в них нагорело достаточно тяжелых элементов, то почему бы не образоваться такой планетке из звездной пыли. Собсно для звезд такие массы это мелочь.
Аноним 14/06/15 Вск 12:32:59 #643 №187766 
>>187752
На Дип Спейс еще испытали ориентацию по астероидам
https://ru.wikipedia.org/wiki/Deep_Space_1
>Автономная навигация Autonav

>Система автономной навигации Autonav, разработанная в Лаборатории реактивного движения NASA, работает с изображениями известных ярких астероидов. Астероиды во внутренней части Солнечной системы перемещаются относительно других тел с известными и предсказуемыми скоростями. Поэтому космический аппарат может определить их относительное положение путем отслеживания подобных астероидов на фоне звезд, которые, в используемом масштабе времени, считаются неподвижными. Два или более астероида позволяют аппарату вычислить свою позицию при помощи триангуляции; две или более позиции во времени позволяют КА определить свою траекторию. Состояние КА отслеживается по его взаимодействию с передатчиками Deep Space Network (DSN), действующими обратно Глобальной системе позиционирования (GPS). Однако, отслеживание при помощи DSN требует множества подготовленных операторов, а сеть DSN перегружена, поскольку используется в качестве сети связи. Использование системы Autonav снижает стоимость миссий и требования к DSN.

>Система автономной навигации Autonav может использоваться и в обратную сторону, для отслеживания расположения тел относительно КА. Это используется для наведения на цель инструментов для научных исследований. В программу аппарата внесено очень грубое определение местоположения цели. После начальной настройки, Autonav удерживает объект в поле видимости, попутно управляя положением КА.[4] Следующим космическим аппаратом, использовавшим Autonav, был Deep Impact.
Аноним 14/06/15 Вск 12:36:02 #644 №187767 
>>187766
Да как бы и солнечные датчики тоже есть. Построение ориентации по визуальным ориентирам - довольно обширная область. По рентгеновским пульсарам и их сигнатурам еще можно ориентироваться.
Аноним 14/06/15 Вск 12:45:14 #645 №187770 DELETED
>>187761
>фонд Зимина

Ах ты пиндосский прихвостень!
Аноним 14/06/15 Вск 22:55:31 #646 №187875 
>>187597
Рекомендую навернуть Хола Клемента, "Экспедиция Тяготение", 10/10, еще и перевод Стругацких, сейчас такого не делают
мимопроходил
Аноним 15/06/15 Пнд 00:30:47 #647 №187918 
14343174471510.png
Можно ли поднять орбиту Земли, если вкопать в землю много мощных ракетны двигателей? Примерно как на пикрелейтед. Тем самым можно решить проблему расширяющегося Солнца через 4 млрд лет.
Аноним 15/06/15 Пнд 00:39:47 #648 №187925 
>>187918
Атмосферу сдует нахер.
Аноним 15/06/15 Пнд 00:50:21 #649 №187936 
>>187925
Схуя ли? Выхлоп за атмосферу, ускорение большое все-равно не получишь.
Год за годом раздеребанивать орбиту как на ионниках - и усе получится.
Вопрос только в дельте и массе-энергии реактивного вещества. Энергию может оплачивать солнышко, при достаточной энергии расход массы практически нулевой - вон, расстреливать космос из БАКа, и всего делов.
Аноним 15/06/15 Пнд 00:55:32 #650 №187939 
>>187936
Лол, читал похожую мысль у Димы Горчева.

>Для полного и окончательного наступления Нового Православного Порядка необходимо следующее:

>Вкопать на обоих полюсах чугунные столбы высотой километров триста. Чугуна у нас дохуя и девать нам его совершенно некуда. На каждом столбе сверху устанавливаются реактивные двигатели с ракеты протон, штук сто или двести, лучше тысячу, и эти двигатели дуют на северном и южном полюсах в противоположные стороны. Керосину нам тоже не жалко, а Европа обойдётся, потому что скоро он ей всё равно не понадобится. Лет через десять-двадцать земная ось изменит угол наклона, и америка окажется на Тёмной Стороне Земли, а европу всю смоет тайфунами и цунами в результате таяния антарктиды. На Руси установится приятный мягкий климат, наподобие того, который сейчас на сейшельских островах, а весь Талибан окажется в зоне вечной мерзлоты.

>Если Православному вдруг захочется зимушки-зимы и прокатиться на с бубенцами тройке, он может поехать в африку. В африке будет климат как сейчас в сибири, негры научатся лепить пельмени, бить белку в глаз и сбивать кедровые орехи при помощи деревянных колотушек — это у них должно хорошо получаться. Немного жалко австралию, она в общем-то никому ничего плохого не сделала, но и хорошего тоже, так что да и хуй с ней.

>В дальнейшем необходимо будет установить такие же столбы в противоположных точках экватора и замедлить скорость вращения земли вокруг оси раза в два, потому что нынешнюю продолжительность суток наверняка установили нетерпеливые Жыдомасоны, которым хочется, чтобы у них почаще наступала ханука. А Православному не надо, чтобы чаще, ему надо чтобы Новый Год был в два раза длиннее. Кроме того, Православный при нынешних сутках только продрал глаза и совсем уже было собрался поработать — а уже вечер и необходимо пить Водку. Из-за этого Жыдомасоны пока ещё всегда выигрывают.

>Те люди, которые окажутся на Тёмной Стороне Земли, они, конечно, сразу запросятся назад, и мы их пустим, потому что мы в общем-то не такие уж злые. Мы пошлём их добывать цырконий из месторождений в оттаявшей антарктиде. Нам самим этот цырконий нахуй не нужен, но главное в нём то, что он страшно ядовитый и добывать его очень неприятно. И когда бывшие наши соотечественники изблюют гамбургеры и кока-колу из чрева своего, вот тогда мы, может быть, их и пустим пожить где-нибудь, там, где комаров побольше.
Аноним 15/06/15 Пнд 01:07:41 #651 №187944 
>>187939
У него хуйня: химия + чугуниевые балки
Химия хуйня сама по себе - посоны засмеют, если ты завалишься на альфу Центавра на химии, как бомжара.
Балки тоже хуйня - нагрузку должна нести поверхность, а не балки, вместо балок - тонкие трубки из чего-нибудь прочного-прочного, чтобы могли сдержать свой вес. Задача трубок - просто не дать попасть атмосфере в выхлоп.
Есть вариант с мощным лазором - он сам себе такую трубку организовывает, совершая оптический пробой атмосферы.
Можно комбинировать: лазорный щит вокруг выхлопа, который не дает проникнуть воздуху внутрь, а унутрях- выхлоп чего-то более высокоимпульсное, чем лазор. Будет гибридный движок: реактивную силу создает и лазор и ускоритель, лазор дополнительно экранирует выхлоп ускорителя от атмосферы.
Аноним 15/06/15 Пнд 01:14:19 #652 №187947 
>>187944
Можно сделать все проще. Бурим сверхглубокий тоннель к земному ядру. Закладываем туда мощную термоядерную бомбу. После взрыва расплавленная магма начинает ебенить охуевшим фонтаном в космос, создавая тягу и облегчая массу земли приводя гравитацию к комфортным 0,7-0.8 G
Аноним 15/06/15 Пнд 01:39:59 #653 №187954 
>>187947
Дай дураку богу молиться...
Дело не в мощности, а в эффективности. Такая тяга еще неэффективнее химии. В том то и прикол - затратить как можно меньше массы, но вкачать в нее как можно больше энергии. Такое могут делать только ускорители, с хирургической точностью.
Термояд же использует эффективно гораздо меньше 10% материи, остальное тупо выбрасывает на ветер.
Термояд сдует часть атмосферы, нанесет Земле непоправимый ущерб, вызовет катаклизмы.
Аноним 15/06/15 Пнд 07:56:55 #654 №187990 
Не по космосу вопрос, но все же.
Человека растягивают на дыбе. Одна и та же сила действует на тело влево и вправо. Сили компенсируются, суперпозиция сил равна нулю, и по идее человек не должен ничего чувствовать. Но он чувствует боль и вырывающиеся конечности.
Почему?
Аноним 15/06/15 Пнд 08:05:26 #655 №187991 
>>187990
Учи сопромат, няша. Если силы скомпенсированы, это еще не значит, что напряжения внутри тела равны нулю.
Аноним 15/06/15 Пнд 11:00:10 #656 №188021 
>>187990
Смотри куда приложены силы - тогда поймешь через что они компенсируются.
Реальная нагрузка на человека будет 2F
Аноним 16/06/15 Втр 03:13:23 #657 №188211 
>>187918
Только если у двигателей скорость истечения будет выше второй космической. Иначе рабочее тело будет притягиваться назад. Желательно - НАМНОГО выше второй космической.
Перекот Аноним 16/06/15 Втр 10:03:46 #658 №188247 
Перекат
Го, я создал.

https://2ch.hk/spc/res/188246.html
https://2ch.pm/spc/res/188246.html
>>188246
Аноним 18/06/15 Чтв 04:34:03 #659 №188604 
>>187936
проще, лучше, больше тяги - сдвинуть фазу дня на 6часов хз как
отмасфера будет в порядке, мощность движка приличная гдето 150 петаватт, импульс так себе
comments powered by Disqus

Отзывы и предложения